HESI Pharmacology Exam Practice

Pataasin ang iyong marka sa homework at exams ngayon gamit ang Quizwiz!

A nurse is caring for a patient who has been administered metaraminol. Which of the following changes should the nurse report to the doctor immediately? 1. Consistent fall in BP 2. Rise in blood glucose levels 3. Decrease in gastric motility 4. Increase in HR

1

A nurse observes leakage of norepinephrine from the IV line. Which of the following interventions should the nurse perform to minimize tissue injury? 1. Discontinue old IV line immediately 2. Do not add another IV line unless instructed 3. Move the head of the bed to an elevated position 4. Mix alkaline solutions with norepinephrine infusions

1

A patient is experiencing insomnia during epinephrine therapy. Which of the following interventions should the nurse perform while caring for the patient? Select all that apply 1. Identify circumstances that disturb sleep 2. Draw curtains over windows 3. Provide bedtime back rub to the patient 4. Give frequent sips of tea and coffee 5. Administer drugs only during daytime

1, 2, 3

Select the terms that describe drugs that stimulate the sympathetic branch of the ANS. Select all that apply 1. Sympathomimetic 2. Sympatholytic 3. Adrenergic 4. Cholinergic

1, 3

A nurse is required to administer dopamine to a patient. Which of the following nursing interventions should the nurse perform when caring for the patient? Select all that apply 1. Administer dopamine only via IV route 2. Mix dopamine with alkaline solutions before administering 3. Use an electronic infusion pump to administer these drugs 4. Monitor BP every 30 mins 5. Inspect needle site and surrounding tissues at frequent intervals

1, 3, 5

173.) A nurse reviews the medication history of a client admitted to the hospital and notes that the client is taking leflunomide (Arava). During data collection, the nurse asks which question to determine medication effectiveness? 1. "Do you have any joint pain?" 2. "Are you having any diarrhea?" 3. "Do you have frequent headaches?" 4. "Are you experiencing heartburn?"

1. "Do you have any joint pain?" Rationale: Leflunomide is an immunosuppressive agent and has an anti-inflammatory action. The medication provides symptomatic relief of rheumatoid arthritis. Diarrhea can occur as a side effect of the medication. The other options are unrelated to medication effectiveness.

127.) The nurse provides medication instructions to an older hypertensive client who is taking 20 mg of lisinopril (Prinivil, Zestril) orally daily. The nurse evaluates the need for further teaching when the client states which of the following? 1. "I can skip a dose once a week." 2. "I need to change my position slowly." 3. "I take the pill after breakfast each day." 4. "If I get a bad headache, I should call my doctor immediately."

1. "I can skip a dose once a week." Rationale: Lisinopril is an antihypertensive angiotensin-converting enzyme (ACE) inhibitor. The usual dosage range is 20 to 40 mg per day. Adverse effects include headache, dizziness, fatigue, orthostatic hypotension, tachycardia, and angioedema. Specific client teaching points include taking one pill a day, not stopping the medication without consulting the health care provider (HCP), and monitoring for side effects and adverse reactions. The client should notify the HCP if side effects occur.

31.) A community health nurse visits a client at home. Prednisone 10 mg orally daily has been prescribed for the client and the nurse reinforces teaching for the client about the medication. Which statement, if made by the client, indicates that further teaching is necessary? 1. "I can take aspirin or my antihistamine if I need it." 2. "I need to take the medication every day at the same time." 3. "I need to avoid coffee, tea, cola, and chocolate in my diet." 4. "If I gain more than 5 pounds a week, I will call my doctor."

1. "I can take aspirin or my antihistamine if I need it." Rationale: Aspirin and other over-the-counter medications should not be taken unless the client consults with the health care provider (HCP). The client needs to take the medication at the same time every day and should be instructed not to stop the medication. A slight weight gain as a result of an improved appetite is expected, but after the dosage is stabilized, a weight gain of 5 lb or more weekly should be reported to the HCP. Caffeine-containing foods and fluids need to be avoided because they may contribute to steroid-ulcer development.

179.) A nurse provides medication instructions to a client who had a kidney transplant about therapy with cyclosporine (Sandimmune). Which statement by the client indicates a need for further instruction? 1. "I need to obtain a yearly influenza vaccine." 2. "I need to have dental checkups every 3 months." 3. "I need to self-monitor my blood pressure at home." 4. "I need to call the health care provider (HCP) if my urine volume decreases or my urine becomes cloudy."

1. "I need to obtain a yearly influenza vaccine." Rationale: Cyclosporine is an immunosuppressant medication. Because of the medication's effects, the client should not receive any vaccinations without first consulting the HCP. The client should report decreased urine output or cloudy urine, which could indicate kidney rejection or infection, respectively. The client must be able to self-monitor blood pressure to check for the side effect of hypertension. The client needs meticulous oral care and dental cleaning every 3 months to help prevent gingival hyperplasia.

77.) Phenytoin (Dilantin), 100 mg orally three times daily, has been prescribed for a client for seizure control. The nurse reinforces instructions regarding the medication to the client. Which statement by the client indicates an understanding of the instructions? 1. "I will use a soft toothbrush to brush my teeth." 2. "It's all right to break the capsules to make it easier for me to swallow them." 3. "If I forget to take my medication, I can wait until the next dose and eliminate that dose." 4. "If my throat becomes sore, it's a normal effect of the medication and it's nothing to be concerned about."

1. "I will use a soft toothbrush to brush my teeth." Rationale: Phenytoin (Dilantin) is an anticonvulsant. Gingival hyperplasia, bleeding, swelling, and tenderness of the gums can occur with the use of this medication. The client needs to be taught good oral hygiene, gum massage, and the need for regular dentist visits. The client should not skip medication doses, because this could precipitate a seizure. Capsules should not be chewed or broken and they must be swallowed. The client needs to be instructed to report a sore throat, fever, glandular swelling, or any skin reaction, because this indicates hematological toxicity.

115.) A client received 20 units of NPH insulin subcutaneously at 8:00 AM. The nurse should check the client for a potential hypoglycemic reaction at what time? 1. 5:00 PM 2. 10:00 AM 3. 11:00 AM 4. 11:00 PM

1. 5:00 PM Rationale: NPH is intermediate-acting insulin. Its onset of action is 1 to 2½ hours, it peaks in 4 to 12 hours, and its duration of action is 24 hours. Hypoglycemic reactions most likely occur during peak time.

10.) The clinic nurse is performing an admission assessment on a client. The nurse notes that the client is taking azelaic acid (Azelex). Because of the medication prescription, the nurse would suspect that the client is being treated for: 1. Acne 2. Eczema 3. Hair loss 4. Herpes simplex

1. Acne Rationale: Azelaic acid is a topical medication used to treat mild to moderate acne. The acid appears to work by suppressing the growth of Propionibacterium acnes and decreasing the proliferation of keratinocytes. Options 2, 3, and 4 are incorrect.

26.) Glimepiride (Amaryl) is prescribed for a client with diabetes mellitus. A nurse reinforces instructions for the client and tells the client to avoid which of the following while taking this medication? 1. Alcohol 2. Organ meats 3. Whole-grain cereals 4. Carbonated beverages

1. Alcohol Rationale: When alcohol is combined with glimepiride (Amaryl), a disulfiram-like reaction may occur. This syndrome includes flushing, palpitations, and nausea. Alcohol can also potentiate the hypoglycemic effects of the medication. Clients need to be instructed to avoid alcohol consumption while taking this medication. The items in options 2, 3, and 4 do not need to be avoided.

142.) A health care provider has written a prescription for ranitidine (Zantac), once daily. The nurse should schedule the medication for which of the following times? 1. At bedtime 2. After lunch 3. With supper 4. Before breakfast

1. At bedtime Rationale: A single daily dose of ranitidine is usually scheduled to be given at bedtime. This allows for a prolonged effect, and the greatest protection of the gastric mucosa. **recall that ranitidine suppresses secretions of gastric acids**

202.) A nurse is collecting data from a client about medications being taken, and the client tells the nurse that he is taking herbal supplements for the treatment of varicose veins. The nurse understands that the client is most likely taking which of the following? 1. Bilberry 2. Ginseng 3. Feverfew 4. Evening primrose

1. Bilberry Rationale: Bilberry is an herbal supplement that has been used to treat varicose veins. This supplement has also been used to treat cataracts, retinopathy, diabetes mellitus, and peripheral vascular disease. Ginseng has been used to improve memory performance and decrease blood glucose levels in type 2 diabetes mellitus. Feverfew is used to prevent migraine headaches and to treat rheumatoid arthritis. Evening primrose is used to treat eczema and skin irritation.

156.) A nurse is reviewing the laboratory results for a client receiving tacrolimus (Prograf). Which laboratory result would indicate to the nurse that the client is experiencing an adverse effect of the medication? 1. Blood glucose of 200 mg/dL 2. Potassium level of 3.8 mEq/L 3. Platelet count of 300,000 cells/mm3 4. White blood cell count of 6000 cells/mm3

1. Blood glucose of 200 mg/dL Rationale: A blood glucose level of 200 mg/dL is elevated above the normal range of 70 to 110 mg/dL and suggests an adverse effect. Other adverse effects include neurotoxicity evidenced by headache, tremor, insomnia; gastrointestinal (GI) effects such as diarrhea, nausea, and vomiting; hypertension; and hyperkalemia.

33.) The home health care nurse is visiting a client who was recently diagnosed with type 2 diabetes mellitus. The client is prescribed repaglinide (Prandin) and metformin (Glucophage) and asks the nurse to explain these medications. The nurse should reinforce which instructions to the client? Select all that apply. 1. Diarrhea can occur secondary to the metformin. 2. The repaglinide is not taken if a meal is skipped. 3. The repaglinide is taken 30 minutes before eating. 4. Candy or another simple sugar is carried and used to treat mild hypoglycemia episodes. 5. Metformin increases hepatic glucose production to prevent hypoglycemia associated with repaglinide. 6. Muscle pain is an expected side effect of metformin and may be treated with acetaminophen (Tylenol).

1. Diarrhea can occur secondary to the metformin. 2. The repaglinide is not taken if a meal is skipped. 3. The repaglinide is taken 30 minutes before eating. 4. Candy or another simple sugar is carried and used to treat mild hypoglycemia episodes. Rationale: Repaglinide is a rapid-acting oral hypoglycemic agent that stimulates pancreatic insulin secretion that should be taken before meals, and that should be withheld if the client does not eat. Hypoglycemia is a side effect of repaglinide and the client should always be prepared by carrying a simple sugar with her or him at all times. Metformin is an oral hypoglycemic given in combination with repaglinide and works by decreasing hepatic glucose production. A common side effect of metformin is diarrhea. Muscle pain may occur as an adverse effect from metformin but it might signify a more serious condition that warrants health care provider notification, not the use of acetaminophen.

116.) A nurse administers a dose of scopolamine (Transderm-Scop) to a postoperative client. The nurse tells the client to expect which of the following side effects of this medication? 1. Dry mouth 2. Diaphoresis 3. Excessive urination 4. Pupillary constriction

1. Dry mouth Rationale: Scopolamine is an anticholinergic medication for the prevention of nausea and vomiting that causes the frequent side effects of dry mouth, urinary retention, decreased sweating, and dilation of the pupils. The other options describe the opposite effects of cholinergic-blocking agents and therefore are incorrect.

112.) A hospitalized client is started on phenelzine sulfate (Nardil) for the treatment of depression. The nurse instructs the client to avoid consuming which foods while taking this medication? Select all that apply. 1. Figs 2. Yogurt 3. Crackers 4. Aged cheese 5 Tossed salad 6. Oatmeal cookies

1. Figs 2. Yogurt 4. Aged cheese Rationale: Phenelzine sulfate (Nardil) is a monoamine oxidase inhibitor(MAOI). The client should avoid taking in foods that are high in tyramine. Use of these foods could trigger a potentially fatal hypertensive crisis. Some foods to avoid include yogurt, aged cheeses, smoked or processed meats, red wines, and fruits such as avocados, raisins, and figs.

93.) The client who is human immunodeficiency virus seropositive has been taking stavudine (d4t, Zerit). The nurse monitors which of the following most closely while the client is taking this medication? 1. Gait 2. Appetite 3. Level of consciousness 4. Hemoglobin and hematocrit blood levels

1. Gait Rationale: Stavudine (d4t, Zerit) is an antiretroviral used to manage human immunodeficiency virus infection in clients who do not respond to or who cannot tolerate conventional therapy. The medication can cause peripheral neuropathy, and the nurse should monitor the client's gait closely and ask the client about paresthesia. Options 2, 3, and 4 are unrelated to the use of the medication.

91.) Cyclobenzaprine (Flexeril) is prescribed for a client to treat muscle spasms, and the nurse is reviewing the client's record. Which of the following disorders, if noted in the client's record, would indicate a need to contact the health care provider regarding the administration of this medication? 1. Glaucoma 2. Emphysema 3. Hyperthyroidism 4. Diabetes mellitus

1. Glaucoma Rationale: Because this medication has anticholinergic effects, it should be used with caution in clients with a history of urinary retention, angle-closure glaucoma, and increased intraocular pressure. Cyclobenzaprine hydrochloride should be used only for short-term 2- to 3-week therapy.

125.) A nurse is preparing to administer digoxin (Lanoxin), 0.125 mg orally, to a client with heart failure. Which vital sign is most important for the nurse to check before administering the medication? 1. Heart rate 2. Temperature 3. Respirations 4. Blood pressure

1. Heart rate Rationale: Digoxin is a cardiac glycoside that is used to treat heart failure and acts by increasing the force of myocardial contraction. Because bradycardia may be a clinical sign of toxicity, the nurse counts the apical heart rate for 1 full minute before administering the medication. If the pulse rate is less than 60 beats/minute in an adult client, the nurse would withhold the medication and report the pulse rate to the registered nurse, who would then contact the health care provider.

158.) A client with chronic renal failure is receiving epoetin alfa (Epogen, Procrit). Which laboratory result would indicate a therapeutic effect of the medication? 1. Hematocrit of 32% 2. Platelet count of 400,000 cells/mm3 3. White blood cell count of 6000 cells/mm3 4. Blood urea nitrogen (BUN) level of 15 mg/dL

1. Hematocrit of 32% Rationale: Epoetin alfa is used to reverse anemia associated with chronic renal failure. A therapeutic effect is seen when the hematocrit is between 30% and 33%. The laboratory tests noted in the other options are unrelated to the use of this medication.

The client diagnosed with angina is prescribed NitroBid and tells the nurse, "I don't understand why I can't take my Viagra. I need to take it so that I can make love to my wife." Which statement is the nurse's best response? 1. "If you take the medications together, you may get very low blood pressure." 2. "You are worried your wife will be concerned if you cannot make love." 3. "If you wait at least 8 hours after taking your NTG, you can take your Viagra." 4. "You should get clarification with your HCP about your taking Viagra."

1. Life-threatening hypotension can result with concurrent use of NTG and Viagra.

43.) A histamine (H2)-receptor antagonist will be prescribed for a client. The nurse understands that which medications are H2-receptor antagonists? Select all that apply. 1. Nizatidine (Axid) 2. Ranitidine (Zantac) 3. Famotidine (Pepcid) 4. Cimetidine (Tagamet) 5. Esomeprazole (Nexium) 6. Lansoprazole (Prevacid)

1. Nizatidine (Axid) 2. Ranitidine (Zantac) 3. Famotidine (Pepcid) 4. Cimetidine (Tagamet) Rationale: H2-receptor antagonists suppress secretion of gastric acid, alleviate symptoms of heartburn, and assist in preventing complications of peptic ulcer disease. These medications also suppress gastric acid secretions and are used in active ulcer disease, erosive esophagitis, and pathological hypersecretory conditions. The other medications listed are proton pump inhibitors. H2-receptor antagonists medication names end with -dine. Proton pump inhibitors medication names end with -zole.

12.) A nurse is caring for a client who is receiving an intravenous (IV) infusion of an antineoplastic medication. During the infusion, the client complains of pain at the insertion site. During an inspection of the site, the nurse notes redness and swelling and that the rate of infusion of the medication has slowed. The nurse should take which appropriate action? 1. Notify the registered nurse. 2. Administer pain medication to reduce the discomfort. 3. Apply ice and maintain the infusion rate, as prescribed. 4. Elevate the extremity of the IV site, and slow the infusion.

1. Notify the registered nurse. Rationale: When antineoplastic medications (Chemotheraputic Agents) are administered via IV, great care must be taken to prevent the medication from escaping into the tissues surrounding the injection site, because pain, tissue damage, and necrosis can result. The nurse monitors for signs of extravasation, such as redness or swelling at the insertion site and a decreased infusion rate. If extravasation occurs, the registered nurse needs to be notified; he or she will then contact the health care provider.

18.) The nurse is reviewing the history and physical examination of a client who will be receiving asparaginase (Elspar), an antineoplastic agent. The nurse consults with the registered nurse regarding the administration of the medication if which of the following is documented in the client's history? 1. Pancreatitis 2. Diabetes mellitus 3. Myocardial infarction 4. Chronic obstructive pulmonary disease

1. Pancreatitis Rationale: Asparaginase (Elspar) is contraindicated if hypersensitivity exists, in pancreatitis, or if the client has a history of pancreatitis. The medication impairs pancreatic function and pancreatic function tests should be performed before therapy begins and when a week or more has elapsed between administration of the doses. The client needs to be monitored for signs of pancreatitis, which include nausea, vomiting, and abdominal pain. The conditions noted in options 2, 3, and 4 are not contraindicated with this medication.

171.) A nurse is preparing to administer furosemide (Lasix) to a client with a diagnosis of heart failure. The most important laboratory test result for the nurse to check before administering this medication is: 1. Potassium level 2. Creatinine level 3. Cholesterol level 4. Blood urea nitrogen

1. Potassium level Rationale: Furosemide is a loop diuretic. The medication causes a decrease in the client's electrolytes, especially potassium, sodium, and chloride. Administering furosemide to a client with low electrolyte levels could precipitate ventricular dysrhythmias. Options 2 and 4 reflect renal function. The cholesterol level is unrelated to the administration of this medication.

30.) A client with diabetes mellitus visits a health care clinic. The client's diabetes mellitus previously had been well controlled with glyburide (DiaBeta) daily, but recently the fasting blood glucose level has been 180 to 200 mg/dL. Which medication, if added to the client's regimen, may have contributed to the hyperglycemia? 1. Prednisone 2. Phenelzine (Nardil) 3. Atenolol (Tenormin) 4. Allopurinol (Zyloprim)

1. Prednisone Rationale: Prednisone may decrease the effect of oral hypoglycemics, insulin, diuretics, and potassium supplements. Option 2, a monoamine oxidase inhibitor, and option 3, a β-blocker, have their own intrinsic hypoglycemic activity. Option 4 decreases urinary excretion of sulfonylurea agents, causing increased levels of the oral agents, which can lead to hypoglycemia.

152.) Intravenous heparin therapy is prescribed for a client. While implementing this prescription, a nurse ensures that which of the following medications is available on the nursing unit? 1. Protamine sulfate 2. Potassium chloride 3. Phytonadione (vitamin K ) 4. Aminocaproic acid (Amicar)

1. Protamine sulfate Rationale: The antidote to heparin is protamine sulfate; it should be readily available for use if excessive bleeding or hemorrhage occurs. Potassium chloride is administered for a potassium deficit. Vitamin K is an antidote for warfarin sodium. Aminocaproic acid is the antidote for thrombolytic therapy.

102.) A client with human immunodeficiency virus is taking nevirapine (Viramune). The nurse should monitor for which adverse effects of the medication? Select all that apply. 1. Rash 2. Hepatotoxicity 3. Hyperglycemia 4. Peripheral neuropathy 5. Reduced bone mineral density

1. Rash 2. Hepatotoxicity Rationale: Nevirapine (Viramune) is a non-nucleoside reverse transcriptase inhibitors (NRTI) that is used to treat HIV infection. It is used in combination with other antiretroviral medications to treat HIV. Adverse effects include rash, Stevens-Johnson syndrome, hepatitis, and increased transaminase levels. Hyperglycemia, peripheral neuropathy, and reduced bone density are not adverse effects of this medication.

72.) Cinoxacin (Cinobac), a urinary antiseptic, is prescribed for the client. The nurse reviews the client's medical record and should contact the health care provider (HCP) regarding which documented finding to verify the prescription? Refer to chart. 1. Renal insufficiency 2. Chest x-ray: normal 3. Blood glucose, 102 mg/dL 4. Folic acid (vitamin B6) 0.5 mg, orally daily

1. Renal insufficiency Rationale: Cinoxacin should be administered with caution in clients with renal impairment. The dosage should be reduced, and failure to do so could result in accumulation of cinoxacin to toxic levels. Therefore the nurse would verify the prescription if the client had a documented history of renal insufficiency. The laboratory and diagnostic test results are normal findings. Folic acid (vitamin B6) may be prescribed for a client with renal insufficiency to prevent anemia.

90.) A nurse is reviewing the record of a client who has been prescribed baclofen (Lioresal). Which of the following disorders, if noted in the client's history, would alert the nurse to contact the health care provider? 1. Seizure disorders 2. Hyperthyroidism 3. Diabetes mellitus 4. Coronary artery disease

1. Seizure disorders Rationale: Clients with seizure disorders may have a lowered seizure threshold when baclofen is administered. Concurrent therapy may require an increase in the anticonvulsive medication. The disorders in options 2, 3, and 4 are not a concern when the client is taking baclofen.

53.) Rifabutin (Mycobutin) is prescribed for a client with active Mycobacterium avium complex (MAC) disease and tuberculosis. The nurse monitors for which side effects of the medication? Select all that apply. 1. Signs of hepatitis 2. Flu-like syndrome 3. Low neutrophil count 4. Vitamin B6 deficiency 5. Ocular pain or blurred vision 6. Tingling and numbness of the fingers

1. Signs of hepatitis 2. Flu-like syndrome 3. Low neutrophil count 5. Ocular pain or blurred vision Rationale: Rifabutin (Mycobutin) may be prescribed for a client with active MAC disease and tuberculosis. It inhibits mycobacterial DNA-dependent RNA polymerase and suppresses protein synthesis. Side effects include rash, gastrointestinal disturbances, neutropenia (low neutrophil count), red-orange body secretions, uveitis (blurred vision and eye pain), myositis, arthralgia, hepatitis, chest pain with dyspnea, and flu-like syndrome. Vitamin B6 deficiency and numbness and tingling in the extremities are associated with the use of isoniazid (INH). Ethambutol (Myambutol) also causes peripheral neuritis.

149.) A client taking fexofenadine (Allegra) is scheduled for allergy skin testing and tells the nurse in the health care provider's office that a dose was taken this morning. The nurse determines that: 1. The client should reschedule the appointment. 2. A lower dose of allergen will need to be injected. 3. A higher dose of allergen will need to be injected. 4. The client should have the skin test read a day later than usual.

1. The client should reschedule the appointment. Rationale: Fexofenadine is an antihistamine, which provides relief of symptoms caused by allergy. Antihistamines should be discontinued for at least 3 days (72 hours) before allergy skin testing to avoid false-negative readings. This client should have the appointment rescheduled for 3 days after discontinuing the medication.

213.) A client is admitted to the hospital with complaints of back spasms. The client states, "I have been taking two or three aspirin every 4 hours for the past week and it hasn't helped my back." Aspirin intoxication is suspected. Which of the following complaints would indicate aspirin intoxication? 1. Tinnitus 2. Constipation 3. Photosensitivity 4. Abdominal cramps

1. Tinnitus Rationale: Mild intoxication with acetylsalicylic acid (aspirin) is called salicylism and is commonly experienced when the daily dosage is higher than 4 g. Tinnitus (ringing in the ears) is the most frequently occurring effect noted with intoxication. Hyperventilation may occur because salicylate stimulates the respiratory center. Fever may result because salicylate interferes with the metabolic pathways involved with oxygen consumption and heat production. Options 2, 3, and 4 are incorrect.

3.) Salicylic acid is prescribed for a client with a diagnosis of psoriasis. The nurse monitors the client, knowing that which of the following would indicate the presence of systemic toxicity from this medication? 1. Tinnitus 2. Diarrhea 3. Constipation 4. Decreased respirations

1. Tinnitus Rationale: Salicylic acid is absorbed readily through the skin, and systemic toxicity (salicylism) can result. Symptoms include tinnitus, dizziness, hyperpnea, and psychological disturbances. Constipation and diarrhea are not associated with salicylism.

110.) A client taking lithium carbonate (Lithobid) reports vomiting, abdominal pain, diarrhea, blurred vision, tinnitus, and tremors. The lithium level is checked as a part of the routine follow-up and the level is 3.0 mEq/L. The nurse knows that this level is: 1. Toxic 2. Normal 3. Slightly above normal 4. Excessively below normal

1. Toxic Rationale: The therapeutic serum level of lithium is 0.6 to 1.2 mEq/L. A level of 3 mEq/L indicates toxicity.

239.) Which of the following precautions will the nurse specifically take during the administration of ribavirin (Virazole) to a child with respiratory syncytial virus (RSV)? 1. Wearing goggles 2. Wearing a gown 3. Wearing a gown and a mask 4. Handwashing before administration

1. Wearing goggles Rationale: Some caregivers experience headaches, burning nasal passages and eyes, and crystallization of soft contact lenses as a result of administration of ribavirin. Specific to this medication is the use of goggles. A gown is not necessary. A mask may be worn. Handwashing is to be performed before and after any child contact.

195.) A nurse is caring for a client who is taking metoprolol (Lopressor). The nurse measures the client's blood pressure (BP) and apical pulse (AP) immediately before administration. The client's BP is 122/78 mm/Hg and the AP is 58 beats/min. Based on this data, which of the following is the appropriate action? 1. Withhold the medication. 2. Notify the registered nurse immediately. 3. Administer the medication as prescribed. 4. Administer half of the prescribed medication.

1. Withhold the medication. Rationale: Metoprolol (Lopressor) is classified as a beta-adrenergic blocker and is used in the treatment of hypertension, angina, and myocardial infarction. Baseline nursing assessments include measurement of BP and AP immediately before administration. If the systolic BP is below 90 mm/Hg and the AP is below 60 beats/min, the nurse should withhold the medication and document this action. Although the registered nurse should be informed of the client's vital signs, it is not necessary to do so immediately. The medication should not be administered because the data is outside of the prescribed parameters for this medication. The nurse should not administer half of the medication, or alter any dosages at any point in time.

234.) A hospitalized client is started on phenelzine sulfate (Nardil) for the treatment of depression. At lunchtime, a tray is delivered to the client. Which food item on the tray will the nurse remove? 1. Yogurt 2. Crackers 3. Tossed salad 4. Oatmeal cookies

1. Yogurt Rationale: Phenelzine sulfate is a monoamine oxidase inhibitor (MAOI). The client should avoid taking in foods that are high in tyramine. These foods could trigger a potentially fatal hypertensive crisis. Foods to avoid include yogurt, aged cheeses, smoked or processed meats, red wines, and fruits such as avocados, raisins, or figs.

176.) A nurse notes that a client is taking lansoprazole (Prevacid). On data collection, the nurse asks which question to determine medication effectiveness? 1. "Has your appetite increased?" 2. "Are you experiencing any heartburn?" 3. "Do you have any problems with vision?" 4. "Do you experience any leg pain when walking?"

2. "Are you experiencing any heartburn?" Rationale: Lansoprazole is a gastric acid pump inhibitor used to treat gastric and duodenal ulcers, erosive esophagitis, and hypersecretory conditions. It also is used to treat gastroesophageal reflux disease (GERD). It is not used to treat visual problems, problems with appetite, or leg pain. **NOTE: "-zole" refers to gastric acid pump inhibitors**

183.) A client who received a kidney transplant is taking azathioprine (Imuran), and the nurse provides instructions about the medication. Which statement by the client indicates a need for further instructions? 1. "I need to watch for signs of infection." 2. "I need to discontinue the medication after 14 days of use." 3. "I can take the medication with meals to minimize nausea." 4. "I need to call the health care provider (HCP) if more than one dose is missed."

2. "I need to discontinue the medication after 14 days of use." Rationale: Azathioprine is an immunosuppressant medication that is taken for life. Because of the effects of the medication, the client must watch for signs of infection, which are reported immediately to the HCP. The client should also call the HCP if more than one dose is missed. The medication may be taken with meals to minimize nausea.

78.) A client is taking phenytoin (Dilantin) for seizure control and a sample for a serum drug level is drawn. Which of the following indicates a therapeutic serum drug range? 1. 5 to 10 mcg/mL 2. 10 to 20 mcg/mL 3. 20 to 30 mcg/mL 4. 30 to 40 mcg/mL

2. 10 to 20 mcg/mL Rationale: The therapeutic serum drug level range for phenytoin (Dilantin) is 10 to 20 mcg/mL. ** A helpful hint may be to remember that the theophylline therapeutic range and the acetaminophen (Tylenol) therapeutic range are the same as the phenytoin (Dilantin) therapeutic range.**

29.) A client is taking Humulin NPH insulin daily every morning. The nurse reinforces instructions for the client and tells the client that the most likely time for a hypoglycemic reaction to occur is: 1. 2 to 4 hours after administration 2. 4 to 12 hours after administration 3. 16 to 18 hours after administration 4. 18 to 24 hours after administration

2. 4 to 12 hours after administration Rationale: Humulin NPH is an intermediate-acting insulin. The onset of action is 1.5 hours, it peaks in 4 to 12 hours, and its duration of action is 24 hours. Hypoglycemic reactions most likely occur during peak time.

Which data indicates to the nurse that simvastatin, an HMG-CoA reductase inhibitor, is effective? 1. The client's blood pressure is 132/80. 2. The client's cholesterol level is 180 mg/dL. 3. The client's LDL is 180 mg/dL. 4. The client's HDL is 35 mg/dL.

2. A cholesterol level less than 200 mg/dL is desirable and indicates the medication is effective.

The client diagnosed with a myocardial infarction is receiving thrombolytic therapy. Which data warrants immediate intervention by the nurse? 1. The client's telemetry has reperfusion dysrhythmias. 2. The client is oozing blood from the IV site. 3. The client is alert and oriented to date, time, and place. 4. The client has no signs of infiltration at the insertion site.

2. Any bleeding from the IV site, gums, rectum, or vagina should be reported to the HCP. The HCP may not be able to take intervention to prevent the bleeding during therapy, but it warrants notifying the HCP.

9.) The nurse is applying a topical corticosteroid to a client with eczema. The nurse would monitor for the potential for increased systemic absorption of the medication if the medication were being applied to which of the following body areas? 1. Back 2. Axilla 3. Soles of the feet 4. Palms of the hands

2. Axilla Rationale: Topical corticosteroids can be absorbed into the systemic circulation. Absorption is higher from regions where the skin is especially permeable (scalp, axilla, face, eyelids, neck, perineum, genitalia), and lower from regions in which permeability is poor (back, palms, soles).

199.) A nurse is applying a topical glucocorticoid to a client with eczema. The nurse monitors for systemic absorption of the medication if the medication is being applied to which of the following body areas? 1. Back 2. Axilla 3. Soles of the feet 4. Palms of the hands

2. Axilla Rationale: Topical glucocorticoids can be absorbed into the systemic circulation. Absorption is higher from regions where the skin is especially permeable (scalp, axillae, face, eyelids, neck, perineum, genitalia), and lower from regions where penetrability is poor (back, palms, soles). **Eliminate options 3 and 4 because these body areas are similar in terms of skin characteristics**

123.) A nurse is planning to administer amlodipine (Norvasc) to a client. The nurse plans to check which of the following before giving the medication? 1. Respiratory rate 2. Blood pressure and heart rate 3. Heart rate and respiratory rate 4. Level of consciousness and blood pressure

2. Blood pressure and heart rate Rationale: Amlodipine is a calcium channel blocker. This medication decreases the rate and force of cardiac contraction. Before administering a calcium channel blocking agent, the nurse should check the blood pressure and heart rate, which could both decrease in response to the action of this medication. This action will help to prevent or identify early problems related to decreased cardiac contractility, heart rate, and conduction. **amlodipine is a calcium channel blocker, and this group of medications decreases the rate and force of cardiac contraction. This in turn lowers the pulse rate and blood pressure.**

224.) Neuroleptic malignant syndrome is suspected in a client who is taking chlorpromazine. Which medication would the nurse prepare in anticipation of being prescribed to treat this adverse effect related to the use of chlorpromazine? 1. Protamine sulfate 2. Bromocriptine (Parlodel) 3. Phytonadione (vitamin K) 4. Enalapril maleate (Vasotec)

2. Bromocriptine (Parlodel) Rationale: Bromocriptine is an antiparkinsonian prolactin inhibitor used in the treatment of neuroleptic malignant syndrome. Vitamin K is the antidote for warfarin (Coumadin) overdose. Protamine sulfate is the antidote for heparin overdose. Enalapril maleate is an antihypertensive used in the treatment of hypertension.

20.) The client with metastatic breast cancer is receiving tamoxifen. The nurse specifically monitors which laboratory value while the client is taking this medication? 1. Glucose level 2. Calcium level 3. Potassium level 4. Prothrombin time

2. Calcium level Rationale: Tamoxifen may increase calcium, cholesterol, and triglyceride levels. Before the initiation of therapy, a complete blood count, platelet count, and serum calcium levels should be assessed. These blood levels, along with cholesterol and triglyceride levels, should be monitored periodically during therapy. The nurse should assess for hypercalcemia while the client is taking this medication. Signs of hypercalcemia include increased urine volume, excessive thirst, nausea, vomiting, constipation, hypotonicity of muscles, and deep bone and flank pain.

146.) A client has begun therapy with theophylline (Theo-24). The nurse tells the client to limit the intake of which of the following while taking this medication? 1. Oranges and pineapple 2. Coffee, cola, and chocolate 3. Oysters, lobster, and shrimp 4. Cottage cheese, cream cheese, and dairy creamers

2. Coffee, cola, and chocolate Rationale: Theophylline is a xanthine bronchodilator. The nurse teaches the client to limit the intake of xanthine-containing foods while taking this medication. These include coffee, cola, and chocolate.

84.) Baclofen (Lioresal) is prescribed for the client with multiple sclerosis. The nurse assists in planning care, knowing that the primary therapeutic effect of this medication is which of the following? 1. Increased muscle tone 2. Decreased muscle spasms 3. Increased range of motion 4. Decreased local pain and tenderness

2. Decreased muscle spasms Rationale: Baclofen is a skeletal muscle relaxant and central nervous system depressant and acts at the spinal cord level to decrease the frequency and amplitude of muscle spasms in clients with spinal cord injuries or diseases and in clients with multiple sclerosis. Options 1, 3, and 4 are incorrect.

209.) A client with multiple sclerosis is receiving diazepam (Valium), a centrally acting skeletal muscle relaxant. Which of the following would indicate that the client is experiencing a side effect related to this medication? 1. Headache 2. Drowsiness 3. Urinary retention 4. Increased salivation

2. Drowsiness Rationale: Incoordination and drowsiness are common side effects resulting from this medication. Options 1, 3, and 4 are incorrect.

133.) A nurse is monitoring a client receiving desmopressin acetate (DDAVP) for adverse effects to the medication. Which of the following indicates the presence of an adverse effect? 1. Insomnia 2. Drowsiness 3. Weight loss 4. Increased urination

2. Drowsiness Rationale: Water intoxication (overhydration) or hyponatremia is an adverse effect to desmopressin. Early signs include drowsiness, listlessness, and headache. Decreased urination, rapid weight gain, confusion, seizures, and coma also may occur in overhydration. **Recall that this medication is used to treat diabetes insipidus to eliminate weight loss and increased urination.**

240.) A client with Parkinson's disease has been prescribed benztropine (Cogentin). The nurse monitors for which gastrointestinal (GI) side effect of this medication? 1. Diarrhea 2. Dry mouth 3. Increased appetite 4. Hyperactive bowel sounds

2. Dry mouth Rationale: Common GI side effects of benztropine therapy include constipation and dry mouth. Other GI side effects include nausea and ileus. These effects are the result of the anticholinergic properties of the medication. **Eliminate options 1 and 4 because they are comparable or alike. Recall that the medication is an anticholinergic, which causes dry mouth**

108.) A nurse is performing a follow-up teaching session with a client discharged 1 month ago who is taking fluoxetine (Prozac). What information would be important for the nurse to gather regarding the adverse effects related to the medication? 1. Cardiovascular symptoms 2. Gastrointestinal dysfunctions 3. Problems with mouth dryness 4. Problems with excessive sweating

2. Gastrointestinal dysfunctions Rationale: The most common adverse effects related to fluoxetine include central nervous system (CNS) and gastrointestinal (GI) system dysfunction. This medication affects the GI system by causing nausea and vomiting, cramping, and diarrhea. Options 1, 3, and 4 are not adverse effects of this medication.

136.) A nurse performs an admission assessment on a client who visits a health care clinic for the first time. The client tells the nurse that propylthiouracil (PTU) is taken daily. The nurse continues to collect data from the client, suspecting that the client has a history of: 1. Myxedema 2. Graves' disease 3. Addison's disease 4. Cushing's syndrome

2. Graves' disease Rationale: PTU inhibits thyroid hormone synthesis and is used to treat hyperthyroidism, or Graves' disease. Myxedema indicates hypothyroidism. Cushing's syndrome and Addison's disease are disorders related to adrenal function.

131.) The nurse is reinforcing medication instructions to a client with breast cancer who is receiving cyclophosphamide (Neosar). The nurse tells the client to: 1. Take the medication with food. 2. Increase fluid intake to 2000 to 3000 mL daily. 3. Decrease sodium intake while taking the medication. 4. Increase potassium intake while taking the medication.

2. Increase fluid intake to 2000 to 3000 mL daily. Rationale: Hemorrhagic cystitis is a toxic effect that can occur with the use of cyclophosphamide. The client needs to be instructed to drink copious amounts of fluid during the administration of this medication. Clients also should monitor urine output for hematuria. The medication should be taken on an empty stomach, unless gastrointestinal (GI) upset occurs. Hyperkalemia can result from the use of the medication; therefore the client would not be told to increase potassium intake. The client would not be instructed to alter sodium intake.

162.) Carbamazepine (Tegretol) is prescribed for a client with a diagnosis of psychomotor seizures. The nurse reviews the client's health history, knowing that this medication is contraindicated if which of the following disorders is present? 1. Headaches 2. Liver disease 3. Hypothyroidism 4. Diabetes mellitus

2. Liver disease Rationale: Carbamazepine (Tegretol) is contraindicated in liver disease, and liver function tests are routinely prescribed for baseline purposes and are monitored during therapy. It is also contraindicated if the client has a history of blood dyscrasias. It is not contraindicated in the conditions noted in the incorrect options.

89.) A nurse is reviewing the laboratory studies on a client receiving dantrolene sodium (Dantrium). Which laboratory test would identify an adverse effect associated with the administration of this medication? 1. Creatinine 2. Liver function tests 3. Blood urea nitrogen 4. Hematological function tests

2. Liver function tests Rationale: Dose-related liver damage is the most serious adverse effect of dantrolene. To reduce the risk of liver damage, liver function tests should be performed before treatment and periodically throughout the treatment course. It is administered in the lowest effective dosage for the shortest time necessary. **Eliminate options 1 and 3 because these tests both assess kidney function.**

204.) A client receives a dose of edrophonium (Enlon). The client shows improvement in muscle strength for a period of time following the injection. The nurse interprets that this finding is compatible with: 1. Multiple sclerosis 2. Myasthenia gravis 3. Muscular dystrophy 4. Amyotrophic lateral sclerosis

2. Myasthenia gravis Rationale: Myasthenia gravis can often be diagnosed based on clinical signs and symptoms. The diagnosis can be confirmed by injecting the client with a dose of edrophonium . This medication inhibits the breakdown of an enzyme in the neuromuscular junction, so more acetylcholine binds to receptors. If the muscle is strengthened for 3 to 5 minutes after this injection, it confirms a diagnosis of myasthenia gravis. Another medication, neostigmine (Prostigmin), also may be used because its effect lasts for 1 to 2 hours, providing a better analysis. For either medication, atropine sulfate should be available as the antidote.

169.) Insulin glargine (Lantus) is prescribed for a client with diabetes mellitus. The nurse tells the client that it is best to take the insulin: 1. 1 hour after each meal 2. Once daily, at the same time each day 3. 15 minutes before breakfast, lunch, and dinner 4. Before each meal, on the basis of the blood glucose level

2. Once daily, at the same time each day Rationale: Insulin glargine is a long-acting recombinant DNA human insulin used to treat type 1 and type 2 diabetes mellitus. It has a 24-hour duration of action and is administered once a day, at the same time each day.

228.) A client receiving an anxiolytic medication complains that he feels very "faint" when he tries to get out of bed in the morning. The nurse recognizes this complaint as a symptom of: 1. Cardiac dysrhythmias 2. Postural hypotension 3. Psychosomatic symptoms 4. Respiratory insufficiency

2. Postural hypotension Rationale: Anxiolytic medications can cause postural hypotension. The client needs to be taught to rise to a sitting position and get out of bed slowly because of this adverse effect related to the medication. Options 1, 3, and 4 are unrelated to the use of this medication.

50.) A nurse has given a client taking ethambutol (Myambutol) information about the medication. The nurse determines that the client understands the instructions if the client states that he or she will immediately report: 1. Impaired sense of hearing 2. Problems with visual acuity 3. Gastrointestinal (GI) side effects 4. Orange-red discoloration of body secretions

2. Problems with visual acuity Rationale: Ethambutol causes optic neuritis, which decreases visual acuity and the ability to discriminate between the colors red and green. This poses a potential safety hazard when a client is driving a motor vehicle. The client is taught to report this symptom immediately. The client is also taught to take the medication with food if GI upset occurs. Impaired hearing results from antitubercular therapy with streptomycin. Orange-red discoloration of secretions occurs with rifampin (Rifadin).

25.) A home care nurse visits a client recently diagnosed with diabetes mellitus who is taking Humulin NPH insulin daily. The client asks the nurse how to store the unopened vials of insulin. The nurse tells the client to: 1. Freeze the insulin. 2. Refrigerate the insulin. 3. Store the insulin in a dark, dry place. 4. Keep the insulin at room temperature.

2. Refrigerate the insulin. Rationale: Insulin in unopened vials should be stored under refrigeration until needed. Vials should not be frozen. When stored unopened under refrigeration, insulin can be used up to the expiration date on the vial. Options 1, 3, and 4 are incorrect.

40.) The client who chronically uses nonsteroidal anti-inflammatory drugs has been taking misoprostol (Cytotec). The nurse determines that the medication is having the intended therapeutic effect if which of the following is noted? 1. Resolved diarrhea 2. Relief of epigastric pain 3. Decreased platelet count 4. Decreased white blood cell count

2. Relief of epigastric pain Rationale: The client who chronically uses nonsteroidal anti-inflammatory drugs (NSAIDs) is prone to gastric mucosal injury. Misoprostol is a gastric protectant and is given specifically to prevent this occurrence. Diarrhea can be a side effect of the medication, but is not an intended effect. Options 3 and 4 are incorrect.

168.) Colcrys (colchicine) is prescribed for a client with a diagnosis of gout. The nurse reviews the client's medical history in the health record, knowing that the medication would be contraindicated in which disorder? 1. Myxedema 2. Renal failure 3. Hypothyroidism 4. Diabetes mellitus

2. Renal failure Rationale: Colchicine is contraindicated in clients with severe gastrointestinal, renal, hepatic or cardiac disorders, or with blood dyscrasias. Clients with impaired renal function may exhibit myopathy and neuropathy manifested as generalized weakness. This medication should be used with caution in clients with impaired hepatic function, older clients, and debilitated clients. **Note that options 1, 3, and 4 are all endocrine-related disorders: Myxedema=Hypothyroidism**

48.) A client is to begin a 6-month course of therapy with isoniazid (INH). A nurse plans to teach the client to: 1. Drink alcohol in small amounts only. 2. Report yellow eyes or skin immediately. 3. Increase intake of Swiss or aged cheeses. 4. Avoid vitamin supplements during therapy.

2. Report yellow eyes or skin immediately. Rationale: INH is hepatotoxic, and therefore the client is taught to report signs and symptoms of hepatitis immediately (which include yellow skin and sclera). For the same reason, alcohol should be avoided during therapy. The client should avoid intake of Swiss cheese, fish such as tuna, and foods containing tyramine because they may cause a reaction characterized by redness and itching of the skin, flushing, sweating, tachycardia, headache, or lightheadedness. The client can avoid developing peripheral neuritis by increasing the intake of pyridoxine (vitamin B6) during the course of INH therapy for TB.

23.) A client who has been newly diagnosed with diabetes mellitus has been stabilized with daily insulin injections. Which information should the nurse teach when carrying out plans for discharge? 1. Keep insulin vials refrigerated at all times. 2. Rotate the insulin injection sites systematically. 3. Increase the amount of insulin before unusual exercise. 4. Monitor the urine acetone level to determine the insulin dosage.

2. Rotate the insulin injection sites systematically. Rationale: Insulin dosages should not be adjusted or increased before unusual exercise. If acetone is found in the urine, it may possibly indicate the need for additional insulin. To minimize the discomfort associated with insulin injections, the insulin should be administered at room temperature. Injection sites should be systematically rotated from one area to another. The client should be instructed to give injections in one area, about 1 inch apart, until the whole area has been used and then to change to another site. This prevents dramatic changes in daily insulin absorption.

216.) A nurse is caring for a client with gout who is taking Colcrys (colchicine). The client has been instructed to restrict the diet to low-purine foods. Which of the following foods should the nurse instruct the client to avoid while taking this medication? 1. Spinach 2. Scallops 3. Potatoes 4. Ice cream

2. Scallops Rationale: Colchicine is a medication used for clients with gout to inhibit the reabsorption of uric acid by the kidney and promote excretion of uric acid in the urine. Uric acid is produced when purine is catabolized. Clients are instructed to modify their diet and limit excessive purine intake. High-purine foods to avoid or limit include organ meats, roe, sardines, scallops, anchovies, broth, mincemeat, herring, shrimp, mackerel, gravy, and yeast.

114.) A postoperative client requests medication for flatulence (gas pains). Which medication from the following PRN list should the nurse administer to this client? 1. Ondansetron (Zofran) 2. Simethicone (Mylicon) 3. Acetaminophen (Tylenol) 4. Magnesium hydroxide (milk of magnesia, MOM)

2. Simethicone (Mylicon) Rationale: Simethicone is an antiflatulent used in the relief of pain caused by excessive gas in the gastrointestinal tract. Ondansetron is used to treat postoperative nausea and vomiting. Acetaminophen is a nonopioid analgesic. Magnesium hydroxide is an antacid and laxative.

172.) A nurse provides dietary instructions to a client who will be taking warfarin sodium (Coumadin). The nurse tells the client to avoid which food item? 1. Grapes 2. Spinach 3. Watermelon 4. Cottage cheese

2. Spinach Rationale: Warfarin sodium is an anticoagulant. Anticoagulant medications act by antagonizing the action of vitamin K, which is needed for clotting. When a client is taking an anticoagulant, foods high in vitamin K often are omitted from the diet. Vitamin K-rich foods include green, leafy vegetables, fish, liver, coffee, and tea.

165.) The client has been on treatment for rheumatoid arthritis for 3 weeks. During the administration of etanercept (Enbrel), it is most important for the nurse to assess: 1. The injection site for itching and edema 2. The white blood cell counts and platelet counts 3. Whether the client is experiencing fatigue and joint pain 4. A metallic taste in the mouth and a loss of appetite

2. The white blood cell counts and platelet counts Rationale: Infection and pancytopenia are adverse effects of etanercept (Enbrel). Laboratory studies are performed before and during treatment. The appearance of abnormal white blood cell counts and abnormal platelet counts can alert the nurse to a potential life-threatening infection. Injection site itching is a common occurrence following administration of the medication. In early treatment, residual fatigue and joint pain may still be apparent. A metallic taste and loss of appetite are not common signs of side effects of this medication.

51.) Cycloserine (Seromycin) is added to the medication regimen for a client with tuberculosis. Which of the following would the nurse include in the client-teaching plan regarding this medication? 1. To take the medication before meals 2. To return to the clinic weekly for serum drug-level testing 3. It is not necessary to call the health care provider (HCP) if a skin rash occurs. 4. It is not necessary to restrict alcohol intake with this medication.

2. To return to the clinic weekly for serum drug-level testing Rationale: Cycloserine (Seromycin) is an antitubercular medication that requires weekly serum drug level determinations to monitor for the potential of neurotoxicity. Serum drug levels lower than 30 mcg/mL reduce the incidence of neurotoxicity. The medication must be taken after meals to prevent gastrointestinal irritation. The client must be instructed to notify the HCP if a skin rash or signs of central nervous system toxicity are noted. Alcohol must be avoided because it increases the risk of seizure activity.

82.) A client is receiving meperidine hydrochloride (Demerol) for pain. Which of the following are side effects of this medication. Select all that apply. 1. Diarrhea 2. Tremors 3. Drowsiness 4. Hypotension 5. Urinary frequency 6. Increased respiratory rate

2. Tremors 3. Drowsiness 4. Hypotension Rationale: Meperidine hydrochloride is an opioid analgesic. Side effects include respiratory depression, drowsiness, hypotension, constipation, urinary retention, nausea, vomiting, and tremors.

135.) A nurse reinforces medication instructions to a client who is taking levothyroxine (Synthroid). The nurse instructs the client to notify the health care provider (HCP) if which of the following occurs? 1. Fatigue 2. Tremors 3. Cold intolerance 4. Excessively dry skin

2. Tremors Rationale: Excessive doses of levothyroxine (Synthroid) can produce signs and symptoms of hyperthyroidism. These include tachycardia, chest pain, tremors, nervousness, insomnia, hyperthermia, heat intolerance, and sweating. The client should be instructed to notify the HCP if these occur. Options 1, 3, and 4 are signs of hypothyroidism.

14.) The client with acute myelocytic leukemia is being treated with busulfan (Myleran). Which laboratory value would the nurse specifically monitor during treatment with this medication? 1. Clotting time 2. Uric acid level 3. Potassium level 4. Blood glucose level

2. Uric acid level Rationale: Busulfan (Myleran) can cause an increase in the uric acid level. Hyperuricemia can produce uric acid nephropathy, renal stones, and acute renal failure. Options 1, 3, and 4 are not specifically related to this medication.

68.) Bethanechol chloride (Urecholine) is prescribed for a client with urinary retention. Which disorder would be a contraindication to the administration of this medication? 1. Gastric atony 2. Urinary strictures 3. Neurogenic atony 4. Gastroesophageal reflux

2. Urinary strictures Rationale: Bethanechol chloride (Urecholine) can be harmful to clients with urinary tract obstruction or weakness of the bladder wall. The medication has the ability to contract the bladder and thereby increase pressure within the urinary tract. Elevation of pressure within the urinary tract could rupture the bladder in clients with these conditions.

238.) Ribavirin (Virazole) is prescribed for the hospitalized child with respiratory syncytial virus (RSV). The nurse prepares to administer this medication via which of the following routes? 1. Orally 2. Via face mask 3. Intravenously 4. Intramuscularly

2. Via face mask Rationale: Ribavirin is an antiviral respiratory medication used mainly in hospitalized children with severe RSV and in high-risk children. Administration is via hood, face mask, or oxygen tent. The medication is most effective if administered within the first 3 days of the infection.

55.) A client who is receiving digoxin (Lanoxin) daily has a serum potassium level of 3.0 mEq/L and is complaining of anorexia. A health care provider prescribes a digoxin level to rule out digoxin toxicity. A nurse checks the results, knowing that which of the following is the therapeutic serum level (range) for digoxin? 1. 3 to 5 ng/mL 2. 0.5 to 2 ng/mL 3. 1.2 to 2.8 ng/mL 4. 3.5 to 5.5 ng/mL

2.) 0.5 to 2 ng/mL Rationale: Therapeutic levels for digoxin range from 0.5 to 2 ng/mL. Therefore, options 1, 3, and 4 are incorrect.

128.) A nurse is providing instructions to an adolescent who has a history of seizures and is taking an anticonvulsant medication. Which of the following statements indicates that the client understands the instructions? 1. "I will never be able to drive a car." 2. "My anticonvulsant medication will clear up my skin." 3. "I can't drink alcohol while I am taking my medication." 4. "If I forget my morning medication, I can take two pills at bedtime."

3. "I can't drink alcohol while I am taking my medication." Rationale: Alcohol will lower the seizure threshold and should be avoided. Adolescents can obtain a driver's license in most states when they have been seizure free for 1 year. Anticonvulsants cause acne and oily skin; therefore a dermatologist may need to be consulted. If an anticonvulsant medication is missed, the health care provider should be notified.

106.) Fluoxetine (Prozac) is prescribed for the client. The nurse reinforces instructions to the client regarding the administration of the medication. Which statement by the client indicates an understanding about administration of the medication? 1. "I should take the medication with my evening meal." 2. "I should take the medication at noon with an antacid." 3. "I should take the medication in the morning when I first arise." 4. "I should take the medication right before bedtime with a snack."

3. "I should take the medication in the morning when I first arise." Rationale: Fluoxetine hydrochloride is administered in the early morning without consideration to meals. **Eliminate options 1, 2, and 4 because they are comparable or alike and indicate taking the medication with an antacid or food.**

197.) Collagenase (Santyl) is prescribed for a client with a severe burn to the hand. The nurse provides instructions to the client regarding the use of the medication. Which statement by the client indicates an accurate understanding of the use of this medication? 1. "I will apply the ointment once a day and leave it open to the air." 2. "I will apply the ointment twice a day and leave it open to the air." 3. "I will apply the ointment once a day and cover it with a sterile dressing." 4. "I will apply the ointment at bedtime and in the morning and cover it with a sterile dressing."

3. "I will apply the ointment once a day and cover it with a sterile dressing." Rationale: Collagenase is used to promote debridement of dermal lesions and severe burns. It is usually applied once daily and covered with a sterile dressing.

42.) A client with a peptic ulcer is diagnosed with a Helicobacter pylori infection. The nurse is reinforcing teaching for the client about the medications prescribed, including clarithromycin (Biaxin), esomeprazole (Nexium), and amoxicillin (Amoxil). Which statement by the client indicates the best understanding of the medication regimen? 1. "My ulcer will heal because these medications will kill the bacteria." 2. "These medications are only taken when I have pain from my ulcer." 3. "The medications will kill the bacteria and stop the acid production." 4. "These medications will coat the ulcer and decrease the acid production in my stomach."

3. "The medications will kill the bacteria and stop the acid production." Rationale: Triple therapy for Helicobacter pylori infection usually includes two antibacterial drugs and a proton pump inhibitor. Clarithromycin and amoxicillin are antibacterials. Esomeprazole is a proton pump inhibitor. These medications will kill the bacteria and decrease acid production.

141.) The nurse has reinforced instructions to a client who has been prescribed cholestyramine (Questran). Which statement by the client indicates a need for further instructions? 1. "I will continue taking vitamin supplements." 2. "This medication will help lower my cholesterol." 3. "This medication should only be taken with water." 4. "A high-fiber diet is important while taking this medication."

3. "This medication should only be taken with water." Rationale: Cholestyramine (Questran) is a bile acid sequestrant used to lower the cholesterol level, and client compliance is a problem because of its taste and palatability. The use of flavored products or fruit juices can improve the taste. Some side effects of bile acid sequestrants include constipation and decreased vitamin absorption. **Note the closed-ended word "only" in option 3**

219.) A health care provider instructs a client with rheumatoid arthritis to take ibuprofen (Motrin). The nurse reinforces the instructions, knowing that the normal adult dose for this client is which of the following? 1. 100 mg orally twice a day 2. 200 mg orally twice a day 3. 400 mg orally three times a day 4. 1000 mg orally four times a day

3. 400 mg orally three times a day Rationale: For acute or chronic rheumatoid arthritis or osteoarthritis, the normal oral adult dose is 400 to 800 mg three or four times daily.

120.) A client is taking lansoprazole (Prevacid) for the chronic management of Zollinger-Ellison syndrome. The nurse advises the client to take which of the following products if needed for a headache? 1. Naprosyn (Aleve) 2. Ibuprofen (Advil) 3. Acetaminophen (Tylenol) 4. Acetylsalicylic acid (aspirin)

3. Acetaminophen (Tylenol) Rationale: Zollinger-Ellison syndrome is a hypersecretory condition of the stomach. The client should avoid taking medications that are irritating to the stomach lining. Irritants would include aspirin and nonsteroidal antiinflammatory drugs (ibuprofen). The client should be advised to take acetaminophen for headache. **Remember that options that are comparable or alike are not likely to be correct. With this in mind, eliminate options 1 and 2 first.**

35.) The client has a PRN prescription for loperamide hydrochloride (Imodium). The nurse understands that this medication is used for which condition? 1. Constipation 2. Abdominal pain 3. An episode of diarrhea 4. Hematest-positive nasogastric tube drainage

3. An episode of diarrhea Rationale: Loperamide is an antidiarrheal agent. It is used to manage acute and also chronic diarrhea in conditions such as inflammatory bowel disease. Loperamide also can be used to reduce the volume of drainage from an ileostomy. It is not used for the conditions in options 1, 2, and 4.

139.) Prednisone is prescribed for a client with diabetes mellitus who is taking Humulin neutral protamine Hagedorn (NPH) insulin daily. Which of the following prescription changes does the nurse anticipate during therapy with the prednisone? 1. An additional dose of prednisone daily 2. A decreased amount of daily Humulin NPH insulin 3. An increased amount of daily Humulin NPH insulin 4. The addition of an oral hypoglycemic medication daily

3. An increased amount of daily Humulin NPH insulin Rationale: Glucocorticoids can elevate blood glucose levels. Clients with diabetes mellitus may need their dosages of insulin or oral hypoglycemic medications increased during glucocorticoid therapy. Therefore the other options are incorrect.

107.) A client receiving a tricyclic antidepressant arrives at the mental health clinic. Which observation indicates that the client is correctly following the medication plan? 1. Reports not going to work for this past week 2. Complains of not being able to "do anything" anymore 3. Arrives at the clinic neat and appropriate in appearance 4. Reports sleeping 12 hours per night and 3 to 4 hours during the day

3. Arrives at the clinic neat and appropriate in appearance Rationale: Depressed individuals will sleep for long periods, are not able to go to work, and feel as if they cannot "do anything." Once they have had some therapeutic effect from their medication, they will report resolution of many of these complaints as well as demonstrate an improvement in their appearance.

174.) A client with portosystemic encephalopathy is receiving oral lactulose (Chronulac) daily. The nurse assesses which of the following to determine medication effectiveness? 1. Lung sounds 2. Blood pressure 3. Blood ammonia level 4. Serum potassium level

3. Blood ammonia level Rationale: Lactulose is a hyperosmotic laxative and ammonia detoxicant. It is used to prevent or treat portosystemic encephalopathy, including hepatic precoma and coma. It also is used to treat constipation. The medication retains ammonia in the colon (decreases the blood ammonia concentration), producing an osmotic effect. It promotes increased peristalsis and bowel evacuation, expelling ammonia from the colon.

69.) A nurse who is administering bethanechol chloride (Urecholine) is monitoring for acute toxicity associated with the medication. The nurse checks the client for which sign of toxicity? 1. Dry skin 2. Dry mouth 3. Bradycardia 4. Signs of dehydration

3. Bradycardia Rationale: Toxicity (overdose) produces manifestations of excessive muscarinic stimulation such as salivation, sweating, involuntary urination and defecation, bradycardia, and severe hypotension. Treatment includes supportive measures and the administration of atropine sulfate subcutaneously or intravenously.

49.) A client has been started on long-term therapy with rifampin (Rifadin). A nurse teaches the client that the medication: 1. Should always be taken with food or antacids 2. Should be double-dosed if one dose is forgotten 3. Causes orange discoloration of sweat, tears, urine, and feces 4. May be discontinued independently if symptoms are gone in 3 months

3. Causes orange discoloration of sweat, tears, urine, and feces Rationale: Rifampin should be taken exactly as directed as part of TB therapy. Doses should not be doubled or skipped. The client should not stop therapy until directed to do so by a health care provider. The medication should be administered on an empty stomach unless it causes gastrointestinal upset, and then it may be taken with food. Antacids, if prescribed, should be taken at least 1 hour before the medication. Rifampin causes orange-red discoloration of body secretions and will permanently stain soft contact lenses.

38.) An older client recently has been taking cimetidine (Tagamet). The nurse monitors the client for which most frequent central nervous system side effect of this medication? 1. Tremors 2. Dizziness 3. Confusion 4. Hallucinations

3. Confusion Rationale: Cimetidine is a histamine 2 (H2)-receptor antagonist. Older clients are especially susceptible to central nervous system side effects of cimetidine. The most frequent of these is confusion. Less common central nervous system side effects include headache, dizziness, drowsiness, and hallucinations.

148.) A client is taking cetirizine hydrochloride (Zyrtec). The nurse checks for which of the following side effects of this medication? 1. Diarrhea 2. Excitability 3. Drowsiness 4. Excess salivation

3. Drowsiness Rationale: A frequent side effect of cetirizine hydrochloride (Zyrtec), an antihistamine, is drowsiness or sedation. Others include blurred vision, hypertension (and sometimes hypotension), dry mouth, constipation, urinary retention, and sweating.

85.) A nurse is monitoring a client receiving baclofen (Lioresal) for side effects related to the medication. Which of the following would indicate that the client is experiencing a side effect? 1. Polyuria 2. Diarrhea 3. Drowsiness 4. Muscular excitability

3. Drowsiness Rationale: Baclofen is a central nervous system (CNS) depressant and frequently causes drowsiness, dizziness, weakness, and fatigue. It can also cause nausea, constipation, and urinary retention. Clients should be warned about the possible reactions. Options 1, 2, and 4 are not side effects.

71.) After kidney transplantation, cyclosporine (Sand immune) is prescribed for a client. Which laboratory result would indicate an adverse effect from the use of this medication? 1. Decreased creatinine level 2. Decreased hemoglobin level 3. Elevated blood urea nitrogen level 4. Decreased white blood cell count

3. Elevated blood urea nitrogen level Rationale: Nephrotoxicity can occur from the use of cyclosporine (Sandimmune). Nephrotoxicity is evaluated by monitoring for elevated blood urea nitrogen (BUN) and serum creatinine levels. Cyclosporine is an immunosuppressant but does not depress the bone marrow.

154.) A nurse is reinforcing dietary instructions to a client who has been prescribed cyclosporine (Sandimmune). Which food item would the nurse instruct the client to avoid? 1. Red meats 2. Orange juice 3. Grapefruit juice 4. Green, leafy vegetables

3. Grapefruit juice Rationale: A compound present in grapefruit juice inhibits metabolism of cyclosporine. As a result, the consumption of grapefruit juice can raise cyclosporine levels by 50% to 100%, thereby greatly increasing the risk of toxicity. Grapefruit juice needs to be avoided. Red meats, orange juice, and green leafy vegetables are acceptable to consume.

97.) Amikacin (Amikin) is prescribed for a client with a bacterial infection. The client is instructed to contact the health care provider (HCP) immediately if which of the following occurs? 1. Nausea 2. Lethargy 3. Hearing loss 4. Muscle aches

3. Hearing loss Rationale: Amikacin (Amikin) is an aminoglycoside. Adverse effects of aminoglycosides include ototoxicity (hearing problems), confusion, disorientation, gastrointestinal irritation, palpitations, blood pressure changes, nephrotoxicity, and hypersensitivity. The nurse instructs the client to report hearing loss to the HCP immediately. Lethargy and muscle aches are not associated with the use of this medication. It is not necessary to contact the HCP immediately if nausea occurs. If nausea persists or results in vomiting, the HCP should be notified. **(most aminoglycoside medication names end in the letters -cin)**

60.) A nurse is planning to administer hydrochlorothiazide (HydroDIURIL) to a client. The nurse understands that which of the following are concerns related to the administration of this medication? 1. Hypouricemia, hyperkalemia 2. Increased risk of osteoporosis 3. Hypokalemia, hyperglycemia, sulfa allergy 4. Hyperkalemia, hypoglycemia, penicillin allergy

3. Hypokalemia, hyperglycemia, sulfa allergy Rationale: Thiazide diuretics such as hydrochlorothiazide are sulfa-based medications, and a client with a sulfa allergy is at risk for an allergic reaction. Also, clients are at risk for hypokalemia, hyperglycemia, hypercalcemia, hyperlipidemia, and hyperuricemia.

130.) The nurse is analyzing the laboratory results of a client with leukemia who has received a regimen of chemotherapy. Which laboratory value would the nurse specifically note as a result of the massive cell destruction that occurred from the chemotherapy? 1. Anemia 2. Decreased platelets 3. Increased uric acid level 4. Decreased leukocyte count

3. Increased uric acid level Rationale: Hyperuricemia is especially common following treatment for leukemias and lymphomas because chemotherapy results in a massive cell kill. Although options 1, 2, and 4 also may be noted, an increased uric acid level is related specifically to cell destruction.

122.) A client who has begun taking fosinopril (Monopril) is very distressed, telling the nurse that he cannot taste food normally since beginning the medication 2 weeks ago. The nurse provides the best support to the client by: 1. Telling the client not to take the medication with food 2. Suggesting that the client taper the dose until taste returns to normal 3. Informing the client that impaired taste is expected and generally disappears in 2 to 3 months 4. Requesting that the health care provider (HCP) change the prescription to another brand of angiotensin-converting enzyme (ACE) inhibitor

3. Informing the client that impaired taste is expected and generally disappears in 2 to 3 months Rationale: ACE inhibitors, such as fosinopril, cause temporary impairment of taste (dysgeusia). The nurse can tell the client that this effect usually disappears in 2 to 3 months, even with continued therapy, and provide nutritional counseling if appropriate to avoid weight loss. Options 1, 2, and 4 are inappropriate actions. Taking this medication with or without food does not affect absorption and action. The dosage should never be tapered without HCP approval and the medication should never be stopped abruptly.

5.) Mafenide acetate (Sulfamylon) is prescribed for the client with a burn injury. When applying the medication, the client complains of local discomfort and burning. Which of the following is the most appropriate nursing action? 1. Notifying the registered nurse 2. Discontinuing the medication 3. Informing the client that this is normal 4. Applying a thinner film than prescribed to the burn site

3. Informing the client that this is normal Rationale: Mafenide acetate is bacteriostatic for gram-negative and gram-positive organisms and is used to treat burns to reduce bacteria present in avascular tissues. The client should be informed that the medication will cause local discomfort and burning and that this is a normal reaction; therefore options 1, 2, and 4 are incorrect

230.) A client is placed on chloral hydrate (Somnote) for short-term treatment. Which nursing action indicates an understanding of the major side effect of this medication? 1. Monitoring neurological signs every 2 hours 2. Monitoring the blood pressure every 4 hours 3. Instructing the client to call for ambulation assistance 4. Lowering the bed and clearing a path to the bathroom at bedtime

3. Instructing the client to call for ambulation assistance Rationale: Chloral hydrate (a sedative-hypnotic) causes sedation and impairment of motor coordination; therefore, safety measures need to be implemented. The client is instructed to call for assistance with ambulation. Options 1 and 2 are not specifically associated with the use of this medication. Although option 4 is an appropriate nursing intervention, it is most important to instruct the client to call for assistance with ambulation.

241.) A client with a history of simple partial seizures is taking clorazepate (Tranxene), and asks the nurse if there is a risk of addiction. The nurse's response is based on the understanding that clorazepate: 1. Is not habit forming, either physically or psychologically 2. Leads to physical tolerance, but only after 10 or more years of therapy 3. Leads to physical and psychological dependence with prolonged high-dose therapy 4. Can result in psychological dependence only, because of the nature of the medication

3. Leads to physical and psychological dependence with prolonged high-dose therapy Rationale: Clorazepate is classified as an anticonvulsant, antianxiety agent, and sedative-hypnotic (benzodiazepine). One of the concerns with clorazepate therapy is that the medication can lead to physical or psychological dependence with prolonged therapy at high doses. For this reason, the amount of medication that is readily available to the client at any one time is restricted. **Eliminate options 2 and 4 first because of the closed-ended word "only"**

52.) A client with tuberculosis is being started on antituberculosis therapy with isoniazid (INH). Before giving the client the first dose, a nurse ensures that which of the following baseline studies has been completed? 1. Electrolyte levels 2. Coagulation times 3. Liver enzyme levels 4. Serum creatinine level

3. Liver enzyme levels Rationale: INH therapy can cause an elevation of hepatic enzyme levels and hepatitis. Therefore, liver enzyme levels are monitored when therapy is initiated and during the first 3 months of therapy. They may be monitored longer in the client who is greater than age 50 or abuses alcohol.

210.) Dantrolene (Dantrium) is prescribed for a client with a spinal cord injury for discomfort resulting from spasticity. The nurse tells the client about the importance of follow-up and the need for which blood study? 1. Creatinine level 2. Sedimentation rate 3. Liver function studies 4. White blood cell count

3. Liver function studies Rationale: Dantrolene can cause liver damage, and the nurse should monitor liver function studies. Baseline liver function studies are done before therapy starts, and regular liver function studies are performed throughout therapy. Dantrolene is discontinued if no relief of spasticity is achieved in 6 weeks.

59.) A client is diagnosed with an acute myocardial infarction and is receiving tissue plasminogen activator, alteplase (Activase, tPA). Which action is a priority nursing intervention? 1. Monitor for renal failure. 2. Monitor psychosocial status. 3. Monitor for signs of bleeding. 4. Have heparin sodium available.

3. Monitor for signs of bleeding. Rationale: Tissue plasminogen activator is a thrombolytic. Hemorrhage is a complication of any type of thrombolytic medication. The client is monitored for bleeding. Monitoring for renal failure and monitoring the client's psychosocial status are important but are not the most critical interventions. Heparin is given after thrombolytic therapy, but the question is not asking about follow-up medications.

151.) A client is being treated for acute congestive heart failure with intravenously administered bumetanide. The vital signs are as follows: blood pressure, 100/60 mm Hg; pulse, 96 beats/min; and respirations, 24 breaths/min. After the initial dose, which of the following is the priority assessment? 1. Monitoring weight loss 2. Monitoring temperature 3. Monitoring blood pressure 4. Monitoring potassium level

3. Monitoring blood pressure Rationale: Bumetanide is a loop diuretic. Hypotension is a common side effect associated with the use of this medication. The other options also require assessment but are not the priority. **priority ABCs—airway, breathing, and circulation**

37.) The client has begun medication therapy with pancrelipase (Pancrease MT). The nurse evaluates that the medication is having the optimal intended benefit if which effect is observed? 1. Weight loss 2. Relief of heartburn 3. Reduction of steatorrhea 4. Absence of abdominal pain

3. Reduction of steatorrhea Rationale: Pancrelipase (Pancrease MT) is a pancreatic enzyme used in clients with pancreatitis as a digestive aid. The medication should reduce the amount of fatty stools (steatorrhea). Another intended effect could be improved nutritional status. It is not used to treat abdominal pain or heartburn. Its use could result in weight gain but should not result in weight loss if it is aiding in digestion.

137.) A nurse is reinforcing instructions for a client regarding intranasal desmopressin acetate (DDAVP). The nurse tells the client that which of the following is a side effect of the medication? 1. Headache 2. Vulval pain 3. Runny nose 4. Flushed skin

3. Runny nose Rationale: Desmopressin administered by the intranasal route can cause a runny or stuffy nose. Headache, vulval pain, and flushed skin are side effects if the medication is administered by the intravenous (IV) route.

111.) A client arrives at the health care clinic and tells the nurse that he has been doubling his daily dosage of bupropion hydrochloride (Wellbutrin) to help him get better faster. The nurse understands that the client is now at risk for which of the following? 1. Insomnia 2. Weight gain 3. Seizure activity 4. Orthostatic hypotension

3. Seizure activity Rationale: Bupropion does not cause significant orthostatic blood pressure changes. Seizure activity is common in dosages greater than 450 mg daily. Bupropion frequently causes a drop in body weight. Insomnia is a side effect, but seizure activity causes a greater client risk.

98.) The nurse is assigned to care for a client with cytomegalovirus retinitis and acquired immunodeficiency syndrome who is receiving foscarnet. The nurse should check the latest results of which of the following laboratory studies while the client is taking this medication? 1. CD4 cell count 2. Serum albumin 3. Serum creatinine 4. Lymphocyte count

3. Serum creatinine Rationale: Foscarnet is toxic to the kidneys. Serum creatinine is monitored before therapy, two to three times per week during induction therapy, and at least weekly during maintenance therapy. Foscarnet may also cause decreased levels of calcium, magnesium, phosphorus, and potassium. Thus these levels are also measured with the same frequency.

211.) A client with epilepsy is taking the prescribed dose of phenytoin (Dilantin) to control seizures. A phenytoin blood level is drawn, and the results reveal a level of 35 mcg/ml. Which of the following symptoms would be expected as a result of this laboratory result? 1. Nystagmus 2. Tachycardia 3. Slurred speech 4. No symptoms, because this is a normal therapeutic level

3. Slurred speech Rationale: The therapeutic phenytoin level is 10 to 20 mcg/mL. At a level higher than 20 mcg/mL, involuntary movements of the eyeballs (nystagmus) appear. At a level higher than 30 mcg/mL, ataxia and slurred speech occur.

46.) A postoperative client has received a dose of naloxone hydrochloride for respiratory depression shortly after transfer to the nursing unit from the postanesthesia care unit. After administration of the medication, the nurse checks the client for: 1. Pupillary changes 2. Scattered lung wheezes 3. Sudden increase in pain 4. Sudden episodes of diarrhea

3. Sudden increase in pain Rationale: Naloxone hydrochloride is an antidote to opioids and may also be given to the postoperative client to treat respiratory depression. When given to the postoperative client for respiratory depression, it may also reverse the effects of analgesics. Therefore, the nurse must check the client for a sudden increase in the level of pain experienced. Options 1, 2, and 4 are not associated with this medication.

245.) A client taking carbamazepine (Tegretol) asks the nurse what to do if he misses one dose. The nurse responds that the carbamazepine should be: 1. Withheld until the next scheduled dose 2. Withheld and the health care provider is notified immediately 3. Taken as long as it is not immediately before the next dose 4. Withheld until the next scheduled dose, which should then be doubled

3. Taken as long as it is not immediately before the next dose Rationale: Carbamazepine is an anticonvulsant that should be taken around the clock, precisely as directed. If a dose is omitted, the client should take the dose as soon as it is remembered, as long as it is not immediately before the next dose. The medication should not be double dosed. If more than one dose is omitted, the client should call the health care provider.

229.) A client who is taking lithium carbonate (Lithobid) is scheduled for surgery. The nurse informs the client that: 1. The medication will be discontinued a week before the surgery and resumed 1 week postoperatively. 2. The medication is to be taken until the day of surgery and resumed by injection immediately postoperatively. 3. The medication will be discontinued 1 to 2 days before the surgery and resumed as soon as full oral intake is allowed. 4. The medication will be discontinued several days before surgery and resumed by injection in the immediate postoperative period.

3. The medication will be discontinued 1 to 2 days before the surgery and resumed as soon as full oral intake is allowed. Rationale: The client who is on lithium carbonate must be off the medication for 1 to 2 days before a scheduled surgical procedure and can resume the medication when full oral intake is prescribed after the surgery. **lithium carbonate is an oral medication and is not given as an injection**

The client being discharged after sustaining an acute myocardial infarction is prescribed the ACE inhibitor lisinopril. Which instruction should the nurse include when teaching about this medication? 1. Instruct the client to monitor the blood pressure weekly. 2. Encourage the client to take the medication on an empty stomach. 3. Discuss the need to rise slowly from lying to a standing position. 4. Teach the client to take the medication at night only.

3. This medication causes orthostatic hypotension, and the client should be instructed to rise slowly from lying to sitting to standing position to prevent falls and injury.

The nurse is preparing to administer Plavix, an antiplatelet medication, to the patient with CAD. The client asks the nurse, "Why am I getting on this medication?" Which statement by the nurse would be most appropriate? 1. "It will help decrease your chance of developing DVT." 2. "Plavix will help decrease your LDL cholesterol levels in about 1 month." 3. "This medication will help prevent your blood from clotting in the arteries." 4. "This medication will help decrease your blood pressure if you take it daily."

3. This medication works in the arteries to prevent platelet aggregation and is prescribed for a client diagnosed with arteriosclerosis.

22.) A nurse is caring for a client after thyroidectomy and notes that calcium gluconate is prescribed for the client. The nurse determines that this medication has been prescribed to: 1. Treat thyroid storm. 2. Prevent cardiac irritability. 3. Treat hypocalcemic tetany. 4. Stimulate the release of parathyroid hormone.

3. Treat hypocalcemic tetany. Rationale: Hypocalcemia can develop after thyroidectomy if the parathyroid glands are accidentally removed or injured during surgery. Manifestations develop 1 to 7 days after surgery. If the client develops numbness and tingling around the mouth, fingertips, or toes or muscle spasms or twitching, the health care provider is notified immediately. Calcium gluconate should be kept at the bedside.

27.) Sildenafil (Viagra) is prescribed to treat a client with erectile dysfunction. A nurse reviews the client's medical record and would question the prescription if which of the following is noted in the client's history? 1. Neuralgia 2. Insomnia 3. Use of nitroglycerin 4. Use of multivitamins

3. Use of nitroglycerin Rationale: Sildenafil (Viagra) enhances the vasodilating effect of nitric oxide in the corpus cavernosum of the penis, thus sustaining an erection. Because of the effect of the medication, it is contraindicated with concurrent use of organic nitrates and nitroglycerin. Sildenafil is not contraindicated with the use of vitamins. Neuralgia and insomnia are side effects of the medication.

The client diagnosed with CAD is instructed to take 81 mg of aspirin daily. Which statement best describes the scientific rationale for prescribing this medication? 1. This medication will help thin the client's blood. 2. Daily aspirin will decrease the incidence of angina. 3. This medication will prevent platelet aggregation. 4. Baby aspirin will not cause gastric distress.

3. When a baby aspirin is taken daily, it helps prevent platelet aggregation, which, in turn, helps the blood pass through the narrowed arteries more easily.

103.) A nurse is caring for a hospitalized client who has been taking clozapine (Clozaril) for the treatment of a schizophrenic disorder. Which laboratory study prescribed for the client will the nurse specifically review to monitor for an adverse effect associated with the use of this medication? 1. Platelet count 2. Cholesterol level 3. White blood cell count 4. Blood urea nitrogen level

3. White blood cell count Rationale: Hematological reactions can occur in the client taking clozapine and include agranulocytosis and mild leukopenia. The white blood cell count should be checked before initiating treatment and should be monitored closely during the use of this medication. The client should also be monitored for signs indicating agranulocytosis, which may include sore throat, malaise, and fever. Options 1, 2, and 4 are unrelated to this medication.

When dobutamine is administered with the b-adrenergic blocking drugs, the nurse is aware of an increased rate for.... 1. Seizures 2. Arrhythmias 3. Hypotension 4. HTN

4

When norepinephrine is transmitted in the sympathetic nervous system, which of the following occurs? 1. Heart rate slows 2. BP lowers 3. GI system speeds up 4. Bronchi relax

4

Which of the following adrenergic receptors is responsible for increased heart rate and increased force of myocardial contraction? 1. a1 receptors 2. a2 receptors 3. b2 receptors 4. b1 receptors

4

Which of the following are the common adverse reactions the nurse would expect with the administration of the adrenergic drugs? 1. Bradycardia, lethargy, bronchial constriction 2. Increase in appetite, nervousness, drowsiness 3. Anorexia, vomiting, hypotension 4. Headache, nervousness, nausea

4

118.) A nurse is caring for an older client with a diagnosis of myasthenia gravis and has reinforced self-care instructions. Which statement by the client indicates that further teaching is necessary? 1. "I rest each afternoon after my walk." 2. "I cough and deep breathe many times during the day." 3. "If I get abdominal cramps and diarrhea, I should call my doctor." 4. "I can change the time of my medication on the mornings that I feel strong."

4. "I can change the time of my medication on the mornings that I feel strong." Rationale: The client with myasthenia gravis should be taught that timing of anticholinesterase medication is critical. It is important to instruct the client to administer the medication on time to maintain a chemical balance at the neuromuscular junction. If not given on time, the client may become too weak to swallow. Options 1, 2, and 3 include the necessary information that the client needs to understand to maintain health with this neurological degenerative disease.

54.) A nurse reinforces discharge instructions to a postoperative client who is taking warfarin sodium (Coumadin). Which statement, if made by the client, reflects the need for further teaching? 1. "I will take my pills every day at the same time." 2. "I will be certain to avoid alcohol consumption." 3. "I have already called my family to pick up a Medic-Alert bracelet." 4. "I will take Ecotrin (enteric-coated aspirin) for my headaches because it is coated."

4. "I will take Ecotrin (enteric-coated aspirin) for my headaches because it is coated." Rationale: Ecotrin is an aspirin-containing product and should be avoided. Alcohol consumption should be avoided by a client taking warfarin sodium. Taking prescribed medication at the same time each day increases client compliance. The Medic-Alert bracelet provides health care personnel emergency information.

61.) A home health care nurse is visiting a client with elevated triglyceride levels and a serum cholesterol level of 398 mg/dL. The client is taking cholestyramine (Questran). Which of the following statements, if made by the client, indicates the need for further education? 1. "Constipation and bloating might be a problem." 2. "I'll continue to watch my diet and reduce my fats." 3. "Walking a mile each day will help the whole process." 4. "I'll continue my nicotinic acid from the health food store."

4. "I'll continue my nicotinic acid from the health food store." Rationale: Nicotinic acid, even an over-the-counter form, should be avoided because it may lead to liver abnormalities. All lipid-lowering medications also can cause liver abnormalities, so a combination of nicotinic acid and cholestyramine resin is to be avoided. Constipation and bloating are the two most common side effects. Walking and the reduction of fats in the diet are therapeutic measures to reduce cholesterol and triglyceride levels.

218.) A film-coated form of diflunisal has been prescribed for a client for the treatment of chronic rheumatoid arthritis. The client calls the clinic nurse because of difficulty swallowing the tablets. Which initial instruction should the nurse provide to the client? 1. "Crush the tablets and mix them with food." 2. "Notify the health care provider for a medication change." 3. "Open the tablet and mix the contents with food." 4. "Swallow the tablets with large amounts of water or milk."

4. "Swallow the tablets with large amounts of water or milk." Rationale: Diflunisal may be given with water, milk, or meals. The tablets should not be crushed or broken open. Taking the medication with a large amount of water or milk should be tried before contacting the health care provider.

198.) Coal tar has been prescribed for a client with a diagnosis of psoriasis, and the nurse provides instructions to the client about the medication. Which statement by the client indicates a need for further instructions? 1. "The medication can cause phototoxicity." 2. "The medication has an unpleasant odor." 3. "The medication can stain the skin and hair." 4. "The medication can cause systemic effects."

4. "The medication can cause systemic effects." Rationale: Coal tar is used to treat psoriasis and other chronic disorders of the skin. It suppresses DNA synthesis, mitotic activity, and cell proliferation. It has an unpleasant odor, can frequently stain the skin and hair, and can cause phototoxicity. Systemic toxicity does not occur. **The name of the medication will assist in eliminating options 2 and 3**

226.) A client receiving lithium carbonate (Lithobid) complains of loose, watery stools and difficulty walking. The nurse would expect the serum lithium level to be which of the following? 1. 0.7 mEq/L 2. 1.0 mEq/L 3. 1.2 mEq/L 4. 1.7 mEq/L

4. 1.7 mEq/L Rationale: The therapeutic serum level of lithium ranges from 0.6 to 1.2 mEq/L. Serum lithium levels above the therapeutic level will produce signs of toxicity.

56.) Heparin sodium is prescribed for the client. The nurse expects that the health care provider will prescribe which of the following to monitor for a therapeutic effect of the medication? 1. Hematocrit level 2. Hemoglobin level 3. Prothrombin time (PT) 4. Activated partial thromboplastin time (aPTT)

4. Activated partial thromboplastin time (aPTT) Rationale: The PT will assess for the therapeutic effect of warfarin sodium (Coumadin) and the aPTT will assess the therapeutic effect of heparin sodium. Heparin sodium doses are determined based on these laboratory results. The hemoglobin and hematocrit values assess red blood cell concentrations.

The client newly diagnosed with CAD is being prescribed a daily aspirin. The client tells the nurse, "I had a bad case of gastritis last year." Which intervention should the nurse implement first? 1. Ask the client if he or she informed the HCP of the gastritis. 2. Explain that regular aspirin could cause gastric upset. 3. Instruct the client to take an enteric-coated aspirin. 4. Determine if the client is taking any antiulcer medication.

4. Assessment is the first part of the nursing process, and determining if the client is taking any antiulcer medication is the first question the nurse should ask the client.

The client calls the clinic and says, "I am having chest pain. I think I am having another heart attack." Which intervention should the nurse implement first? 1. Call 911 emergency medical services. 2. Instruct the client to take an aspirin. 3. Determine if the client is at home alone. 4. Ask if the client has any sublingual NTG.

4. Because the client has had one myocardial infarction, the client may have sublingual NTG in a pocket and can take it immediately. If the client does not have any on the body, then the nurse should determine if there is anyone in the home that can help the client.

237.) A client who is on lithium carbonate (Lithobid) will be discharged at the end of the week. In formulating a discharge teaching plan, the nurse will instruct the client that it is most important to: 1. Avoid soy sauce, wine, and aged cheese. 2. Have the lithium level checked every week. 3. Take medication only as prescribed because it can become addicting. 4. Check with the psychiatrist before using any over-the-counter (OTC) medications or prescription medications.

4. Check with the psychiatrist before using any over-the-counter (OTC) medications or prescription medications. Rationale: Lithium is the medication of choice to treat manic-depressive illness. Many OTC medications interact with lithium, and the client is instructed to avoid OTC medications while taking lithium. Lithium is not addicting, and, although serum lithium levels need to be monitored, it is not necessary to check these levels every week. A tyramine-free diet is associated with monoamine oxidase inhibitors.

19.) Tamoxifen is prescribed for the client with metastatic breast carcinoma. The nurse understands that the primary action of this medication is to: 1. Increase DNA and RNA synthesis. 2. Promote the biosynthesis of nucleic acids. 3. Increase estrogen concentration and estrogen response. 4. Compete with estradiol for binding to estrogen in tissues containing high concentrations of receptors.

4. Compete with estradiol for binding to estrogen in tissues containing high concentrations of receptors. Rationale: Tamoxifen is an antineoplastic medication that competes with estradiol for binding to estrogen in tissues containing high concentrations of receptors. Tamoxifen is used to treat metastatic breast carcinoma in women and men. Tamoxifen is also effective in delaying the recurrence of cancer following mastectomy. Tamoxifen reduces DNA synthesis and estrogen response.

217.) A health care provider prescribes auranofin (Ridaura) for a client with rheumatoid arthritis. Which of the following would indicate to the nurse that the client is experiencing toxicity related to the medication? 1. Joint pain 2. Constipation 3. Ringing in the ears 4. Complaints of a metallic taste in the mouth

4. Complaints of a metallic taste in the mouth Rationale: Ridaura is the one gold preparation that is given orally rather than by injection. Gastrointestinal reactions including diarrhea, abdominal pain, nausea, and loss of appetite are common early in therapy, but these usually subside in the first 3 months of therapy. Early symptoms of toxicity include a rash, purple blotches, pruritus, mouth lesions, and a metallic taste in the mouth.

124.) A client with chronic renal failure is receiving ferrous sulfate (Feosol). The nurse monitors the client for which common side effect associated with this medication? 1. Diarrhea 2. Weakness 3. Headache 4. Constipation

4. Constipation Rationale: Feosol is an iron supplement used to treat anemia. Constipation is a frequent and uncomfortable side effect associated with the administration of oral iron supplements. Stool softeners are often prescribed to prevent constipation. **Focus on the name of the medication. Recalling that oral iron can cause constipation will easily direct you to the correct option.**

132.) The client with non-Hodgkin's lymphoma is receiving daunorubicin (DaunoXome). Which of the following would indicate to the nurse that the client is experiencing a toxic effect related to the medication? 1. Fever 2. Diarrhea 3. Complaints of nausea and vomiting 4. Crackles on auscultation of the lungs

4. Crackles on auscultation of the lungs Rationale: Cardiotoxicity noted by abnormal electrocardiographic findings or cardiomyopathy manifested as congestive heart failure is a toxic effect of daunorubicin. Bone marrow depression is also a toxic effect. Nausea and vomiting are frequent side effects associated with the medication that begins a few hours after administration and lasts 24 to 48 hours. Fever is a frequent side effect, and diarrhea can occur occasionally. The other options, however, are not toxic effects. **keep in mind that the question is asking about a toxic effect and think: ABCs—airway, breathing, and circulation**

117.) A nurse has given the client taking ethambutol (Myambutol) information about the medication. The nurse determines that the client understands the instructions if the client immediately reports: 1. Impaired sense of hearing 2. Distressing gastrointestinal side effects 3. Orange-red discoloration of body secretions 4. Difficulty discriminating the color red from green

4. Difficulty discriminating the color red from green Rationale: Ethambutol causes optic neuritis, which decreases visual acuity and the ability to discriminate between the colors red and green. This poses a potential safety hazard when driving a motor vehicle. The client is taught to report this symptom immediately. The client is also taught to take the medication with food if gastrointestinal upset occurs. Impaired hearing results from antitubercular therapy with streptomycin. Orange-red discoloration of secretions occurs with rifampin (Rifadin).

203.) A nurse is preparing to give the postcraniotomy client medication for incisional pain. The family asks the nurse why the client is receiving codeine sulfate and not "something stronger." In formulating a response, the nurse incorporates the understanding that codeine: 1. Is one of the strongest opioid analgesics available 2. Cannot lead to physical or psychological dependence 3. Does not cause gastrointestinal upset or constipation as do other opioids 4. Does not alter respirations or mask neurological signs as do other opioids

4. Does not alter respirations or mask neurological signs as do other opioids Rationale: Codeine sulfate is the opioid analgesic often used for clients after craniotomy. It is frequently combined with a nonopioid analgesic such as acetaminophen for added effect. It does not alter the respiratory rate or mask neurological signs as do other opioids. Side effects of codeine include gastrointestinal upset and constipation. The medication can lead to physical and psychological dependence with chronic use. It is not the strongest opioid analgesic available.

159.) A nurse is caring for a client receiving morphine sulfate subcutaneously for pain. Because morphine sulfate has been prescribed for this client, which nursing action would be included in the plan of care? 1. Encourage fluid intake. 2. Monitor the client's temperature. 3. Maintain the client in a supine position. 4. Encourage the client to cough and deep breathe.

4. Encourage the client to cough and deep breathe. Rationale: Morphine sulfate suppresses the cough reflex. Clients need to be encouraged to cough and deep breathe to prevent pneumonia. **ABCs—airway, breathing, and circulation**

196.) A client has been prescribed amikacin (Amikin). Which of the following priority baseline functions should be monitored? 1. Apical pulse 2. Liver function 3. Blood pressure 4. Hearing acuity

4. Hearing acuity Rationale: Amikacin (Amikin) is an antibiotic. This medication can cause ototoxicity and nephrotoxicity; therefore, hearing acuity tests and kidney function studies should be performed before the initiation of therapy. Apical pulse, liver function studies, and blood pressure are not specifically related to the use of this medication.

192.) A nurse is collecting medication information from a client, and the client states that she is taking garlic as an herbal supplement. The nurse understands that the client is most likely treating which of the following conditions? 1. Eczema 2. Insomnia 3. Migraines 4. Hyperlipidemia

4. Hyperlipidemia Rationale: Garlic is an herbal supplement that is used to treat hyperlipidemia and hypertension. An herbal supplement that may be used to treat eczema is evening primrose. Insomnia has been treated with both valerian root and chamomile. Migraines have been treated with feverfew.

76.) Carbidopa-levodopa (Sinemet) is prescribed for a client with Parkinson's disease, and the nurse monitors the client for adverse reactions to the medication. Which of the following indicates that the client is experiencing an adverse reaction? 1. Pruritus 2. Tachycardia 3. Hypertension 4. Impaired voluntary movements

4. Impaired voluntary movements Rationale: Dyskinesia and impaired voluntary movement may occur with high levodopa dosages. Nausea, anorexia, dizziness, orthostatic hypotension, bradycardia, and akinesia (the temporary muscle weakness that lasts 1 minute to 1 hour, also known as the "on-off phenomenon") are frequent side effects of the medication.

212.) Mannitol (Osmitrol) is being administered to a client with increased intracranial pressure following a head injury. The nurse assisting in caring for the client knows that which of the following indicates the therapeutic action of this medication? 1. Prevents the filtration of sodium and water through the kidneys 2. Prevents the filtration of sodium and potassium through the kidneys 3. Decreases water loss by promoting the reabsorption of sodium and water in the loop of Henle 4. Induces diuresis by raising the osmotic pressure of glomerular filtrate, thereby inhibiting tubular reabsorption of water and solutes

4. Induces diuresis by raising the osmotic pressure of glomerular filtrate, thereby inhibiting tubular reabsorption of water and solutes Rationale: Mannitol is an osmotic diuretic that induces diuresis by raising the osmotic pressure of glomerular filtrate, thereby inhibiting tubular reabsorption of water and solutes. It is used to reduce intracranial pressure in the client with head trauma.

225.) A nursing student is assigned to care for a client with a diagnosis of schizophrenia. Haloperidol (Haldol) is prescribed for the client, and the nursing instructor asks the student to describe the action of the medication. Which statement by the nursing student indicates an understanding of the action of this medication? 1. It is a serotonin reuptake blocker. 2. It inhibits the breakdown of released acetylcholine. 3. It blocks the uptake of norepinephrine and serotonin. 4. It blocks the binding of dopamine to the postsynaptic dopamine receptors in the brain.

4. It blocks the binding of dopamine to the postsynaptic dopamine receptors in the brain. Rationale: Haloperidol acts by blocking the binding of dopamine to the postsynaptic dopamine receptors in the brain. Imipramine hydrochloride (Tofranil) blocks the reuptake of norepinephrine and serotonin. Donepezil hydrochloride (Aricept) inhibits the breakdown of released acetylcholine. Fluoxetine hydrochloride (Prozac) is a potent serotonin reuptake blocker.

119.) A client with diabetes mellitus who has been controlled with daily insulin has been placed on atenolol (Tenormin) for the control of angina pectoris. Because of the effects of atenolol, the nurse determines that which of the following is the most reliable indicator of hypoglycemia? 1. Sweating 2. Tachycardia 3. Nervousness 4. Low blood glucose level

4. Low blood glucose level Rationale: β-Adrenergic blocking agents, such as atenolol, inhibit the appearance of signs and symptoms of acute hypoglycemia, which would include nervousness, increased heart rate, and sweating. Therefore, the client receiving this medication should adhere to the therapeutic regimen and monitor blood glucose levels carefully. Option 4 is the most reliable indicator of hypoglycemia.

205.) A nurse is assisting in preparing to administer acetylcysteine (Mucomyst) to a client with an overdose of acetaminophen (Tylenol). The nurse prepares to administer the medication by: 1. Administering the medication subcutaneously in the deltoid muscle 2. Administering the medication by the intramuscular route in the gluteal muscle 3. Administering the medication by the intramuscular route, mixed in 10 mL of normal saline 4. Mixing the medication in a flavored ice drink and allowing the client to drink the medication through a straw

4. Mixing the medication in a flavored ice drink and allowing the client to drink the medication through a straw Rationale: Because acetylcysteine has a pervasive odor of rotten eggs, it must be disguised in a flavored ice drink. It is consumed preferably through a straw to minimize contact with the mouth. It is not administered by the intramuscular or subcutaneous route. **Knowing that the medication is a solution that is also used for nebulization treatments will assist you to select the option that indicates an oral route**

143.) A client has just taken a dose of trimethobenzamide (Tigan). The nurse plans to monitor this client for relief of: 1. Heartburn 2. Constipation 3. Abdominal pain 4. Nausea and vomiting

4. Nausea and vomiting Rationale: Trimethobenzamide is an antiemetic agent used in the treatment of nausea and vomiting. The other options are incorrect.

17.) The client with ovarian cancer is being treated with vincristine (Oncovin). The nurse monitors the client, knowing that which of the following indicates a side effect specific to this medication? 1. Diarrhea 2. Hair loss 3. Chest pain 4. Numbness and tingling in the fingers and toes

4. Numbness and tingling in the fingers and toes Rationale: A side effect specific to vincristine is peripheral neuropathy, which occurs in almost every client. Peripheral neuropathy can be manifested as numbness and tingling in the fingers and toes. Depression of the Achilles tendon reflex may be the first clinical sign indicating peripheral neuropathy. Constipation rather than diarrhea is most likely to occur with this medication, although diarrhea may occur occasionally. Hair loss occurs with nearly all the antineoplastic medications. Chest pain is unrelated to this medication.

39.) The client with a gastric ulcer has a prescription for sucralfate (Carafate), 1 g by mouth four times daily. The nurse schedules the medication for which times? 1. With meals and at bedtime 2. Every 6 hours around the clock 3. One hour after meals and at bedtime 4. One hour before meals and at bedtime

4. One hour before meals and at bedtime Rationale: Sucralfate is a gastric protectant. The medication should be scheduled for administration 1 hour before meals and at bedtime. The medication is timed to allow it to form a protective coating over the ulcer before food intake stimulates gastric acid production and mechanical irritation. The other options are incorrect.

2.) Oral iron supplements are prescribed for a 6-year-old child with iron deficiency anemia. The nurse instructs the mother to administer the iron with which best food item? 1. Milk 2. Water 3. Apple juice 4. Orange juice

4. Orange juice Rationale: Vitamin C increases the absorption of iron by the body. The mother should be instructed to administer the medication with a citrus fruit or a juice that is high in vitamin C. Milk may affect absorption of the iron. Water will not assist in absorption. Orange juice contains a greater amount of vitamin C than apple juice.

15.) The client with small cell lung cancer is being treated with etoposide (VePesid). The nurse who is assisting in caring for the client during its administration understands that which side effect is specifically associated with this medication? 1. Alopecia 2. Chest pain 3. Pulmonary fibrosis 4. Orthostatic hypotension

4. Orthostatic hypotension Rationale: A side effect specific to etoposide is orthostatic hypotension. The client's blood pressure is monitored during the infusion. Hair loss occurs with nearly all the antineoplastic medications. Chest pain and pulmonary fibrosis are unrelated to this medication.

222.) A nurse has administered a dose of diazepam (Valium) to a client. The nurse would take which important action before leaving the client's room? 1. Giving the client a bedpan 2. Drawing the shades or blinds closed 3. Turning down the volume on the television 4. Per agency policy, putting up the side rails on the bed

4. Per agency policy, putting up the side rails on the bed Rationale: Diazepam is a sedative-hypnotic with anticonvulsant and skeletal muscle relaxant properties. The nurse should institute safety measures before leaving the client's room to ensure that the client does not injure herself or himself. The most frequent side effects of this medication are dizziness, drowsiness, and lethargy. For this reason, the nurse puts the side rails up on the bed before leaving the room to prevent falls. Options 1, 2, and 3 may be helpful measures that provide a comfortable, restful environment, but option 4 is the one that provides for the client's safety needs.

184.) A nurse preparing a client for surgery reviews the client's medication record. The client is to be nothing per mouth (NPO) after midnight. Which of the following medications, if noted on the client's record, should the nurse question? 1. Cyclobenzaprine (Flexeril) 2. Alendronate (Fosamax) 3. Allopurinol (Zyloprim) 4. Prednisone

4. Prednisone Rationale: Prednisone is a corticosteroid that can cause adrenal atrophy, which reduces the body's ability to withstand stress. Before and during surgery, dosages may be temporarily increased. Cyclobenzaprine is a skeletal muscle relaxant. Alendronate is a bone-resorption inhibitor. Allopurinol is an antigout medication.

189.) Prostaglandin E1 is prescribed for a child with transposition of the great arteries. The mother of the child asks the nurse why the child needs the medication. The nurse tells the mother that the medication: 1. Prevents hypercyanotic (blue or tet) spells 2. Maintains an adequate hormone level 3. Maintains the position of the great arteries 4. Provides adequate oxygen saturation and maintains cardiac output

4. Provides adequate oxygen saturation and maintains cardiac output Rationale: A child with transposition of the great arteries may receive prostaglandin E1 temporarily to increase blood mixing if systemic and pulmonary mixing are inadequate to maintain adequate cardiac output. Options 1, 2, and 3 are incorrect. In addition, hypercyanotic spells occur in tetralogy of Fallot. **Use the ABCs—airway, breathing, and circulation—to answer the question. The correct option addresses circulation**

13.) The client with squamous cell carcinoma of the larynx is receiving bleomycin intravenously. The nurse caring for the client anticipates that which diagnostic study will be prescribed? 1. Echocardiography 2. Electrocardiography 3. Cervical radiography 4. Pulmonary function studies

4. Pulmonary function studies Rationale: Bleomycin is an antineoplastic medication (Chemotheraputic Agents) that can cause interstitial pneumonitis, which can progress to pulmonary fibrosis. Pulmonary function studies along with hematological, hepatic, and renal function tests need to be monitored. The nurse needs to monitor lung sounds for dyspnea and crackles, which indicate pulmonary toxicity. The medication needs to be discontinued immediately if pulmonary toxicity occurs. Options 1, 2, and 3 are unrelated to the specific use of this medication.

144.) A client is taking docusate sodium (Colace). The nurse monitors which of the following to determine whether the client is having a therapeutic effect from this medication? 1. Abdominal pain 2. Reduction in steatorrhea 3. Hematest-negative stools 4. Regular bowel movements

4. Regular bowel movements Rationale: Docusate sodium is a stool softener that promotes the absorption of water into the stool, producing a softer consistency of stool. The intended effect is relief or prevention of constipation. The medication does not relieve abdominal pain, stop gastrointestinal (GI) bleeding, or decrease the amount of fat in the stools.

70.) Oxybutynin chloride (Ditropan XL) is prescribed for a client with neurogenic bladder. Which sign would indicate a possible toxic effect related to this medication? 1. Pallor 2. Drowsiness 3. Bradycardia 4. Restlessness

4. Restlessness Rationale: Toxicity (overdosage) of this medication produces central nervous system excitation, such as nervousness, restlessness, hallucinations, and irritability. Other signs of toxicity include hypotension or hypertension, confusion, tachycardia, flushed or red face, and signs of respiratory depression. Drowsiness is a frequent side effect of the medication but does not indicate overdosage.

66.) Trimethoprim-sulfamethoxazole (TMP-SMZ) is prescribed for a client. A nurse should instruct the client to report which symptom if it developed during the course of this medication therapy? 1. Nausea 2. Diarrhea 3. Headache 4. Sore throat

4. Sore throat Rationale: Clients taking trimethoprim-sulfamethoxazole (TMP-SMZ) should be informed about early signs of blood disorders that can occur from this medication. These include sore throat, fever, and pallor, and the client should be instructed to notify the health care provider if these symptoms occur. The other options do not require health care provider notification.

166.) Alendronate (Fosamax) is prescribed for a client with osteoporosis. The client taking this medication is instructed to: 1. Take the medication at bedtime. 2. Take the medication in the morning with breakfast. 3. Lie down for 30 minutes after taking the medication. 4. Take the medication with a full glass of water after rising in the morning.

4. Take the medication with a full glass of water after rising in the morning. Rationale: Precautions need to be taken with administration of alendronate to prevent gastrointestinal side effects (especially esophageal irritation) and to increase absorption of the medication. The medication needs to be taken with a full glass of water after rising in the morning. The client should not eat or drink anything for 30 minutes following administration and should not lie down after taking the medication.

The client with CAD is prescribed Niacin. The client complains of flushing of the face, neck, and ears. Which priority intervention should the nurse implement? 1. Instruct the client to stop taking the medication immediately. 2. Encourage the client to take the medication with meals only. 3. Discuss that this is a normal side effect and will decrease with time. 4. Tell the client to take 325 mg of aspirin 30 minutes before taking medication.

4. Taking an aspirin prior to the medication will help reduce the flushing of the face, neck, and ears.

177.) A nurse is assisting in caring for a pregnant client who is receiving intravenous magnesium sulfate for the management of preeclampsia and notes that the client's deep tendon reflexes are absent. On the basis of this data, the nurse reports the finding and makes which determination? 1. The magnesium sulfate is effective. 2. The infusion rate needs to be increased. 3. The client is experiencing cerebral edema. 4. The client is experiencing magnesium toxicity.

4. The client is experiencing magnesium toxicity. Rationale: Magnesium toxicity can occur as a result of magnesium sulfate therapy. Signs of magnesium sulfate toxicity relate to the central nervous system depressant effects of the medication and include respiratory depression; loss of deep tendon reflexes; sudden decrease in fetal heart rate or maternal heart rate, or both; and sudden drop in blood pressure. Hyperreflexia indicates increased cerebral edema. An absence of reflexes indicates magnesium toxicity. The therapeutic serum level of magnesium for a client receiving magnesium sulfate ranges from 4 to 7.5 mEq/L (5 to 8 mg/dL).

The nurse is completing A.M. care with a client diagnosed with angina when the client complains of chest pain. The client has a saline lock in the right forearm. Which intervention should the nurse at the bedside implement first? 1. Assess the client's vital signs. 2. Administer sublingual NTG. 3. Administer IV morphine sulfate via saline lock. 4. Administer oxygen via nasal cannula.

4. The nurse would have oxygen at the bedside, and applying it would be the first intervention the nurse could implement at bedside.

99.) The client with acquired immunodeficiency syndrome and Pneumocystis jiroveci infection has been receiving pentamidine isethionate (Pentam 300). The client develops a temperature of 101° F. The nurse does further monitoring of the client, knowing that this sign would most likely indicate: 1. The dose of the medication is too low. 2. The client is experiencing toxic effects of the medication. 3. The client has developed inadequacy of thermoregulation. 4. The result of another infection caused by leukopenic effects of the medication.

4. The result of another infection caused by leukopenic effects of the medication. Rationale: Frequent side effects of this medication include leukopenia, thrombocytopenia, and anemia. The client should be monitored routinely for signs and symptoms of infection. Options 1, 2, and 3 are inaccurate interpretations.

121.) A client who is taking hydrochlorothiazide (HydroDIURIL, HCTZ) has been started on triamterene (Dyrenium) as well. The client asks the nurse why both medications are required. The nurse formulates a response, based on the understanding that: 1. Both are weak potassium-losing diuretics. 2. The combination of these medications prevents renal toxicity. 3. Hydrochlorothiazide is an expensive medication, so using a combination of diuretics is cost-effective. 4. Triamterene is a potassium-sparing diuretic, whereas hydrochlorothiazide is a potassium-losing diuretic.

4. Triamterene is a potassium-sparing diuretic, whereas hydrochlorothiazide is a potassium-losing diuretic. Rationale: Potassium-sparing diuretics include amiloride (Midamor), spironolactone (Aldactone), and triamterene (Dyrenium). They are weak diuretics that are used in combination with potassium-losing diuretics. This combination is useful when medication and dietary supplement of potassium is not appropriate. The use of two different diuretics does not prevent renal toxicity. Hydrochlorothiazide is an effective and inexpensive generic form of the thiazide classification of diuretics. **It is especially helpful to remember that hydrochlorothiazide is a potassium-losing diuretic and triamterene is a potassium-sparing diuretic**

140.) The client has a new prescription for metoclopramide (Reglan). On review of the chart, the nurse identifies that this medication can be safely administered with which condition? 1. Intestinal obstruction 2. Peptic ulcer with melena 3. Diverticulitis with perforation 4. Vomiting following cancer chemotherapy

4. Vomiting following cancer chemotherapy Rationale: Metoclopramide is a gastrointestinal (GI) stimulant and antiemetic. Because it is a GI stimulant, it is contraindicated with GI obstruction, hemorrhage, or perforation. It is used in the treatment of emesis after surgery, chemotherapy, and radiation.

185.) Which of the following herbal therapies would be prescribed for its use as an antispasmodic? Select all that apply. 1.Aloe 2.Kava 3.Ginger 4.Chamomile 5.Peppermint oil

4.Chamomile 5.Peppermint oil Rationale: Chamomile has a mild sedative effect and acts as an antispasmodic and anti-inflammatory. Peppermint oil acts as an antispasmodic and is used for irritable bowel syndrome. Topical aloe promotes wound healing. Aloe taken orally acts as a laxative. Kava has an anxiolytic, sedative, and analgesic effect. Ginger is effective in relieving nausea.

-setron

5-HT2 receptor antagonists (antiemetics)

B. Vastus lateralis ***The preferred intramuscular site for children younger than 2 years is the vastus lateralis (B). (A, C, and D) are not preferred injection sites for the infant at 2 months of age.

A 2-month-old infant is scheduled to receive the first DPT immunization. What is the preferred injection site to administer this immunization? A. Dorsal gluteal B. Vastus lateralis C. Ventral gluteal D. Deltoid

B. Diazepam (Valium) ***Diazepam (Valium) (B) is the drug of choice for treatment of status epilepticus. (A, C, and D) are used for the long-term management of seizure disorders but are not as useful in the emergency management of status epilepticus.

A 6-year-old child is admitted to the emergency department with status epilepticus. His parents report that his seizure disorder has been managed with phenytoin (Dilantin), 50 mg PO bid, for the past year. Which drug should the nurse plan to administer in the emergency department? A. Phenytoin (Dilantin) B. Diazepam (Valium) C. Phenobarbital (Luminal) D. Carbamazepine (Tegretol)

B. This drug is a clot buster that dissolves clots within a coronary artery. ***t-PA, or tissue plasminogen activator, is a coronary-specific fibrinolytic agent that dissolves clots within the coronary arteries (B). This drug is not a calcium channel blocker or nitrate, which would promote vasodilation of the coronary arteries (A). This medication is not an anticoagulant, such as warfarin or heparin, which would prevent new clot formation (C). Volume expansion is not provided by an infusion of TPA and would not necessarily improve myocardial perfusion caused by an increased cardiac output in a client with coronary artery disease (D).

A client being treated for an acute myocardial infarction is to receive the tissue plasminogen activator alteplase (Activase). The nurse would be correct in providing which explanation to the client regarding the purpose of this drug? A. This drug is a nitrate that promotes vasodilation of the coronary arteries. B. This drug is a clot buster that dissolves clots within a coronary artery. C. This drug is a blood thinner that will help prevent the formation of a new clot. D. This drug is a volume expander that improves myocardial perfusion by increasing output.

D. Drink eight glasses of fluid a day. ***Adequate hydration is important for all sulfa drugs because they can crystallize in the urine (D). If possible, the drug should be taken after eating to provide longer intestinal transit time (B). (A) is important for other medications, such as phenytoin (Dilantin), because of the incidence of gingival hyperplasia, and (C) is important for steroid administration, but (D) is most important to stress with this client.

A client is being discharged with a prescription for sulfasalazine (Azulfidine) to treat ulcerative colitis. Which instruction should the nurse provide to this client prior to discharge? A. Maintain good oral hygiene. B. Take the medication 30 minutes before a meal. C. Discontinue use of the drug gradually. D. Drink eight glasses of fluid a day.

A. " My mouth feels sore" ***Stomatitis caused by a thrush infection, which can cause mouth pain, is a sign of superinfection (A). (B, C, and D) are more typical side effects, rather than symptoms, of a superinfection.

A client is receiving anti-infective drug therapy for a postoperative infection. Which complaint should alert the nurse to the possibility that the client has contracted a superinfection? A. " My mouth feels sore" B. "I have a headache." C. "My ears feel plugged up." D. "I feel constipated"

B. Protamine sulfate ***Protamine sulfate (B) is the antagonist for heparin and is given for episodes of acute hemorrhage. (A, C, and D) are not heparin antagonists.

A client receiving a continuous infusion of heparin IV starts to hemorrhage from an arterial access site. Which medication should the nurse anticipate administering to prevent further heparin-induced hemorrhaging? A. Vitamin K1 (AquaMEPHYTON) B. Protamine sulfate C. Warfarin sodium (Coumadin) D. Prothrombin

B. Reduction or elimination of nausea and vomiting ***Ondansetron (Zofran) is a type 3 receptor (5-HT3) antagonist that is recognized for improved control of acute nausea and vomiting associated with chemotherapy (B). 5-HT3 antagonists are most effective when administered IV prior to the induction of the chemotherapeutic agent(s). (A, C, and D) are not therapeutic actions of ondansetron (Zofran).

A client with acute lymphocytic leukemia is to begin chemotherapy today. The health care provider's prescription specifies that ondansetron (Zofran) is to be administered IV 30 minutes prior to the infusion of cisplatin (Platinol). What is the rationale for administering Zofran prior to the chemotherapy induction? A. Promotion of diuresis to prevent nephrotoxicity B. Reduction or elimination of nausea and vomiting C. Prevention of a secondary hyperuricemia D. Reduction in the risk of an allergic reaction

C. Dopamine in the central nervous system is increased. ***Amantadine (Symmetrel) is a dopamine-releasing agent (C); therefore, this medication increases the amount of dopamine present in the central nervous system. Although this medication is also an antiviral agent (A), the antiviral effect is not significant in the treatment of parkinsonism. (B and D) are not affected by amantadine (Symmetrel).

A client with mild Parkinsonism is started on oral amantadine (Symmetrel). Which statement accurately describes the action of this medication? A. Viral organisms that cause Parkinsonism are eliminated. B. Acetylcholine in the myoneural junction is enhanced. C. Dopamine in the central nervous system is increased. D. Norepinephrine release is reduced within the periphery.

C. vitamin A ***This drug is a derivative of vitamin A.

A client with sever acne is seen in the clinic and the HCP prescribes isotretinoin. The nurse reviews the client's medication record and would contact the HCP if the client is also taking which medication? A. digoxin B. phenytoin C. vitamin A D. furosemide

A. Black stools ***Warfarin is an anticoagulant medication that prevents blood clots. Alternatively, it may also increase the risk of bleeding. The nurse should assess for signs of bleeding in the gastrointestinal system, which could manifest as black stools.

A nurse is preparing to administer a dose of warfarin to a patient. Based on the nurse's knowledge of this drug, the nurse knows to monitor for which of the following side effects? A. Black stools B. Constipation C. Abdominal bloating D. Back pain

A. Maintain a drug administration record. ***A written drug administration record (A) provides a consistent plan to ensure safe adherence to multiple medication dosages and times. Although (B) is an important safeguard to monitor for drug interactions, the parents should be given a tool to enhance their confidence and provide a mechanism to ensure accurate and timely medication administration without duplicating or omitting a dose. Using a written record to record medication administration allows more than one person (C) to share the responsibility of giving medications to the child. Although smaller volumes (D) ensure that all the medication is taken, it is more important to maintain an accurate administration schedule.

A pediatric client is discharged home with multiple prescriptions for medications. Which information should the nurse provide that is most helpful to the parents when managing the medication regimens? A. Maintain a drug administration record. B. Fill all prescriptions at one pharmacy. C. Allow one person to give the medications. D. Give all medications in small volumes.

-pril

ACE inhibitors (antihypertensives)

How can supine HTN be minimized when administering midodrine to a patient?

Administer in the morning while patient is in upright position

Because the sympathetic nervous system is activated by the precursor of epinephrine, another name for the sympathetic nervous system is the ___________ ____________

Adrenergic Branch

C. Anaphylactic ***A patient may have an adverse reaction to a medication, which can occur as a result of various types of interactions, including cytotoxic response or an infection. An anaphylactic reaction is manifested as difficulty breathing and wheezing, facial edema, and flushing of the skin, which represent a hypersensitivity of the patient's body to the medication.

After starting an IV dose of sulfamethoxazole (Bactrim®), the nurse notes that the patient is having difficulty breathing, his face is flushed, and he complains of back pain. Which type of hypersensitivity reaction is this patient most likely experiencing? A. Cytotoxic B. Serum sickness C. Anaphylactic D. Infectious

Which drugs are short-acting b2 agonists (SABAs- used for acute respiratory symptom relief)?

Albuterol Epinephrine Bitolterol Levalbuterol Metaproterenol Pirbuterol Terbutaline

-azosin

Alpha Blockers (adrenergic antagonists)

D. Partial thromboplastin time (PTT) ***Heparin therapy is guided by changes in the partial thromboplastin time (PTT) (D). (A, B, and C) are not used to track the therapeutic effect of heparin administration.

Alteration of which laboratory finding represents the achievement of a therapeutic goal for heparin administration? A. Prothrombin time (PT) B. Fibrin split products C. Platelet count D. Partial thromboplastin time (PTT)

-micin

Aminoglycosides (antibiotics)

-mycin

Aminoglycosides/Macrolides (antibiotics)

C. 20 ***500 mg/X mL = 125 mg/5 mL; 125X = 2500; X = 20 mL

Amoxicillin, 500 mg PO every 8 hours, is prescribed for a client with an infection. The drug is available in a suspension of 125 mg/5 mL. How many milliliters should the nurse administer with each dose? A. 10 B. 15 C. 20 D. 25

D. Respirations decrease to 14 breaths/min. ***Hydromorphone (Dilaudid) is an opioid agonist-analgesic of opiate receptors that inhibits ascending pathways and can cause respiratory depression. Older adults are more sensitive to opioids so the "start low and go slow" approach should be taken (D). (A) lists common side effects of opioids, particularly the opiates, which are usually harmless and often transient . (B) is within the normal range (2 to 6 cm). The suture site may be red and swollen as an inflammatory response, but no action is required if the skin around the incision is a normal color and temperature (C).

An older client who had a colon resection yesterday is receiving a constant dose of hydromorphone (Dilaudid) via a patient-controlled analgesia (PCA) pump. Which assessment finding is most significant and requires that the nurse intervene? A. The client is drowsy and complains of pruritus. B. Pupils are 3 mm; PERRLA. C. The area around the sutures is reddened and swollen. D. Respirations decrease to 14 breaths/min.

-sartan

Angiotensin-II receptor antagonists

-triptan

Anti-migraines

-azole

Antifungals

-vastatin

Antilipemics (anti-cholesterol)

-stine

Antineoplastics (anti-tumor)

-quine

Antiparasitics

rifa-

Antituberculars

-cyclovir

Antivirals

-vir

Antivirals

-circlovir-

Antivirls

Which drugs are long-acting b2 agonists (LABAs- used for long-term respiratory management)?

Arformoterol Formoterol Indacaterol Salmeterol

Which intervention is most important for the nurse implement for a client is receiving lispro Humalog insulin? -Check blood glucose levels every six hours -provide meals at the same time that insulin is given -assess for hypoglycemia between meals -keeping oral liquid or glucose source available

B Rationale Humalog is a rapid acting insulin 15 minutes, so meals should be readily available time administration of lispro (B). Although glucose blood levels a, monitoring for hypoglycemia see, and keeping emergency source of glucose available B should be implemented for client receiving any form of insulin those who take these are at greatest risk for rapid hypoglycemia shortly after it is administered

-barbital

Barbiturates (sedative-hypnotics)

-iam

Benzodiazepines (anxiolytics)

-pam

Benzodiazepines (anxiolytics)

-lol

Beta Blockers (adrenergic antagonists)

-dronate

Biphosphonates

-terol

Bronchodilators

Adrenergic drugs are classified as pregnancy category __________ and are used with extreme caution during pregnancy

C

-calci-

Calcium & Vitamin D Supplements

-dipine

Calcium Channel Blockers

Ephedra should not be taken with...

Cardiac glycosides Halothane Guanethidine MAOI antidepressants Oxytoxin With St John's wort

This type of shock occurs when cardiac output is insufficient and perfusion to vital organs cannot be maintained. Usually occurs as a result of MI, ventricular arrhythmias, CHF, or severe cardiomyopathy

Cardiogenic shock

cef-

Cephalosporins (antibiotics)

ceph-

Cephalosporins (antibiotics)

-stigmine

Cholinergics

Adrenergic drugs are used cautiously in patients with...

Coronary insufficiency Cardiac arrhythmias Angina pectoris Diabetes Hyperthyroidism Occlusive vascular disease Prostatic Hypertrophy

-lone

Corticosteroids (anti-inflammatory)

-sone

Corticosteroids (anti-inflammatory)

In explaining the benefits of the combination anti-infective drug code tri-moxazole/TMP-SMZ (bactrium) to a client receiving the medication for a urinary tract infection, more rationale to the nurse provide? -Each drug could cause damage to the kidneys if taken separately -one drug reduces the risk of side effects caused by the drug -while one drug provide releif, the other fights the infection -the two drugs work together to reduce resistance of the bacterial infection of symptoms

D rationale the combination of drugs and bacteria work synergistically to reduce bacterial resistance enterprise D, thereby increasing the drug therapeutic benefits answer choices a B &C are incorrect rationales

Before administering a laxative to a bedfast client, it is most important for the nurse to perform what assessment? a-Observe the skin integrity of the client's rectal and sacral areas b-assess the client strength in moving and turning in the bed c-evaluate the client's ability to recognize the urge to defecate d-determine the frequency and consistency of bowel movements

D rationale the nurse should ensure that the client is not experiencing diarrhea (answer choice D) before administering a laxative, which will increase the frequency of bowel movements. answer choices A,B,C R4 and assessments for the provision of care when the client experience is a valve, but are of less priority then answer choice D which provides data about the possible need to hold medication

This type of shock occurs when there are changes to the blood vessels causing dilation but no additional blood volume. The body is redistributed within the body. Septic, anaphylactic, and neurogenic shock fall under this category

Distributive Shock

B. Increase in urine output ***Intropin activates dopamine receptors in the kidney and dilates blood vessels to improve renal perfusion, so an increase in urine output (B) indicates an increase in glomerular filtration caused by increased arterial blood pressure. (A) is related to fluid retention but is not an indicator of a therapeutic response to dopamine therapy. (C) is not related to the vasopressor effect of dopamine therapy. Dopamine increases cardiac output, which increases a client's blood pressure, not (D).

Dopamine (Intropin) is administered to a client who is hypotensive. Which finding should the nurse identify as a therapeutic response? A. Gain in weight B. Increase in urine output C. Improved gastric motility D. Decrease in blood pressure

This herb has been used to relieve cold symptoms, improve respiratory function, and as an adjunct in weight loss. It has been shown to cause stroke or heart attack though, so it is typically not recommended to take this herb

Ephedra

-ergot-

Ergotamines (anti-migraine)

-floxacin

Fluoroquinolones (Antibiotics)

A. Decrease in level of chest pain ***Nitroglycerin reduces myocardial oxygen consumption, which decreases ischemia and reduces chest pain (A). (B, C, and D) are not expected outcomes of sublingual nitroglycerin.

Following the administration of sublingual nitroglycerin, which assessment finding indicates that the medication was effective? A. Decrease in level of chest pain B. Clear bilateral breath sounds C. Increase in blood pressure D. Increase in urinary output

-ane

General anesthetics

-tidine

H2 receptor antagonists (anti-ulcer)

-navir

HIV/AIDS antivirals

Norepinephrine is C/I in patients who are...

Hypotensive from blood volume deficits

A. Hypotension ***Nifedipine (Procardia) reduces peripheral vascular resistance and nitrates produce vasodilation, so concurrent use of nitrates with nifedipine can cause hypotension with the initial administration of these agents (A). (B, C, and D) are not side effects of this treatment regimen.

In addition to nitrate therapy, a client is receiving nifedipine (Procardia), 10 mg PO every 6 hours. The nurse should plan to observe for which common side effect of this treatment regimen? A. Hypotension B. Hyperkalemia C. Hypokalemia D. Seizures

Adrenergic drugs produce what type of effects on the heart?

Increase in heart rate

Patients with diabetes will need an ________ dose of insulin while taking adrenergic drugs

Increased

If an adrenergic drug is taken in conjunction with an oxytoxin, what may occur as a result?

Increased risk of HTN

If dobutamine is administered with b-adrenergic blocking drugs, what may occur as a result?

Increased risk of HTN

If metaraminol is taken in conjunction with an digoxin, what may occur as a result?

Increased risk of cardiac arrhythmias

If an adrenergic drug is taken in conjunction with an phenytoin (Dilantin), what may occur as a result?

Increased risk of seizures, hypotension, and bradycardia

The sympathetic nervous system is regulated by ____________ control and it is known as the "fight-or-flight" response

Involuntary

D. total cholesterol level ***Isotretinoin can elevate triglyceride levels. They should be measure before treatment and peridoically thereafter until the effect on the triglycerides is know.

Isotretinoin is prescribed for a client with severe acne. Before administration of this medication, the nurse anticipates that which laboratory test will be prescribed? A. K levels B. triglyceride levels C. Hemoglobin A1C D. total cholesterol level

-caine

Local anesthetics

This drug should only be given to patients whose lives are impaired despite standard treatment offered

Midodrine

Adrenergic drugs are useful in improving hemodynamic status by improving ____________ contractility and increasing heart rate

Myocardial

-profen

NSAIDS (anti-inflammatory)

Epineprhine is C/I in patients with...

Narrow-angle glaucoma As a local anesthetic adjunct in fingers and toes

-zoline

Nasal decongestants

This type of shock occurs when there is interference with the PNS control of blood vessels usually due to a spinal injury

Neurogenic Shock

-curium

Neuromuscular Blockers

-curonium

Neuromuscular Blockers

If an adrenergic drug is ___________, it means that it acts on both alpha receptors and beta receptors

Nonselective

This neurotransmitter is the primary one of the sympathetic nervous system and the substance that keeps the nerve impulse going from the brain to the target organ.

Norepinephrine

This type of shock occurs when obstruction of blood flow results in inadequate tissue perfusion. Usually occurs as a result of pericardial tamponade, restrictive pericarditis, and severe cardiac valve dysfunction

Obstructive Shock

Epinephrine may temporarily increase tremors and rigidity in older adults with __________________ disease

Parkinson's

-vudine

Phenothiazines (antipsychotics, antiemeitics)

Dopamine is C/I in patients with...

Pheochromocytoma (Adrenal gland tumor) Unmanaged arthritis Ventricular fibrillation

-thiazide

Potassium-losing diuretics

-actone

Potassium-sparing diuretics

-prazole

Proton Pump Inhibitors (anti-ulcers)

182.) A client with angina pectoris is experiencing chest pain that radiates down the left arm. The nurse administers a sublingual nitroglycerin tablet to the client. The client's pain is unrelieved, and the nurse determines that the client needs another nitroglycerin tablet. Which of the following vital signs is most important for the nurse to check before administering the medication? 1. Temperature 2. Respirations 3. Blood pressure 4. Radial pulse rate

Rationale: Nitroglycerin acts directly on the smooth muscle of the blood vessels, causing relaxation and dilation. As a result, hypotension can occur. The nurse would check the client's blood pressure before administering the second nitroglycerin tablet. Although the respirations and apical pulse may be checked, these vital signs are not affected as a result of this medication. The temperature also is not associated with the administration of this medication.

134.) A nurse reinforces instructions to a client who is taking levothyroxine (Synthroid). The nurse tells the client to take the medication: 1. With food 2. At lunchtime 3. On an empty stomach 4. At bedtime with a snack

Rationale: Oral doses of levothyroxine (Synthroid) should be taken on an empty stomach to enhance absorption. Dosing should be done in the morning before breakfast. **Note that options 1, 2, and 4 are comparable or alike in that these options address administering the medication with food.**

206.) A client is receiving baclofen (Lioresal) for muscle spasms caused by a spinal cord injury. The nurse monitors the client, knowing that which of the following is a side effect of this medication? 1. Muscle pain 2. Hypertension 3. Slurred speech 4. Photosensitivity

Rationale: Side effects of baclofen include drowsiness, dizziness, weakness, and nausea. Occasional side effects include headache, paresthesia of the hands and feet, constipation or diarrhea, anorexia, hypotension, confusion, and nasal congestion. Paradoxical central nervous system excitement and restlessness can occur, along with slurred speech, tremor, dry mouth, nocturia, and impotence. **Option 3 is most closely associated with a neurological disorder**

A. tinnitus ***Salicylic acid is absorbed readily through the skin, and systemic toxicity (salicylism) can result. Symptoms include tinnitu, dizziness, hyperpnea, and psychological disturbances. Constipation and diarrhea are not associated with salicylism.

Salicylic acid is prescribed for a client with a diagnosis of psoriasis. The nurse monitors the client, knowing that which finding indicates the presence of systemic toxicity from this medication? A. tinnitus B. diarrhea C. constipation D. decreased respirations

If an adrenergic drug is ___________, it means that it acts on alpha receptors or beta receptors

Selective

This type of shock occurs when circulatory insufficiency results from overwhelming infection

Septic Shock

What is the suggested dosing schedule for midodrine?

Shortly before arising in the morning, midday and late afternoon Should not be administered after 6PM

-ine

Stimulants

sulf-

Sulfonamides (antibiotics)

What is a major adverse reaction of taking midodrine?

Supine HTN

The autonomic nervous system is divided into what 2 sections?

Sympathetic and Parasympathetic

The adrenergic drugs produce pharmacological effects similar to the effects that occur in the body when the _____________ ___________ (norepinephrine) and the _______________ (epinephrine) are stimulated

Sympathetic nerves/ Medulla

Adrenergic drugs are also known as _______________ which means to mimic the actions of the sympathetic nervous system

Sympathomimetic

Isoproterenol C/I in patients with...

Tachyarrhythmias, tachycardia, or heart block caused by digitalis toxicity, ventricular arrhythmias, and angina pectoris

-cycline

Tetracyclines (Antibiotics)

D. at least 30 minutes before exposure to the sun ***Sunscreens are most effective when applied at least 30 minutes before exposure to the sun so that they can penetrate the skin. All sunscreens should be reapplied after swimming.

The camp nurse asks the children preparing to swim in the lake if they have applied sunscreen. The nurse reminds the children that chemical sunscreens are most effective when applied at which times? A. immediately before swimming B. 5 minutes before exposure to the sun C. immediately before exposure to the sun D. at least 30 minutes before exposure to the sun

D. Contact the health care provider to clarify the prescription. ***Lovenox is a low-molecular-weight heparin that can only be administered subcutaneously, so the nurse should contact the health care provider to clarify the route of administration (D). (A and B) are important nursing interventions but not necessary to the administration of this medication. The client should be instructed about medication side effects (C), but this is of lower priority than obtaining a correct prescription.

The health care provider has prescribed a low-molecular-weight heparin, enoxaparin (Lovenox) prefilled syringe, 30 mg/0.3 mL IV every 12 hours, for a client following hip replacement. Prior to administering the first dose, which intervention is most important for the nurse to implement? A. Assess the client's IV site for signs of inflammation. B. Evaluate the client's degree of mobility. C. Instruct the client regarding medication side effects. D. Contact the health care provider to clarify the prescription.

B. refrigerate the insulin

The home care nurse visits a client recently diagnosed with diabetes mellitus who is taking Hummulin NPH insulin daily. The client asks the nurse how to store the unopened vials of insulin. The nurse should tell the client to take which action? A. freeze the insulin B. refrigerate the insulin C. store the insulin in a dark, dry place D. keep the insulin at room temperature

A. Thirty minutes after the dose is administered ***Peak drug serum levels are achieved 30 minutes after the IV administration of aminoglycosides, so (A) is the optimum time to get a peak level. (B, C, and D) are not appropriate times associated with peak levels for gentamicin.

The nurse plans to draw blood samples for the determination of peak and trough levels of gentamicin sulfate (Garamycin) in a client receiving IV doses of this medication. When should the nurse plan to obtain the peak level? A. Thirty minutes after the dose is administered B. Immediately before giving the next dose C. When the next electrolyte levels are drawn D. Sixty minutes after the dose is administered

-ase

Thrombolytics (clot-busters)

What is the purpose of stimulating the sympathetic nerves?

To divert blood flow to the vital organs so that the body can deal with a stressful situation

C. Compare the client's blood pressure before and after the client takes the medication. ***Therapeutic effects are the expected or predictable physiologic responses to a medication. An antihypertensive medication is administered to lower blood pressure, so to determine if the therapeutic effect has been achieved, the nurse should compare the client's blood pressure before and after the client takes the medication (C). (A and B) provide data related to the side effect of hypotension, which may occur following the administration of an antihypertensive medication. (D) provides useful data but does not evaluate the medication's effectiveness.

To evaluate whether the administration of an antihypertensive medication has caused a therapeutic effect, which action should the nurse implement? A. Ask the client about the onset of any dizziness since taking the medication. B. Measure the client's blood pressure while the client is lying, sitting, and then standing. C. Compare the client's blood pressure before and after the client takes the medication. D. Interview the client about any past or recent history of high blood pressure.

-triptyline

Tricyclics (antidepressants)

A. Severe acute anaphylactic response ***Anaphylaxis related to penicillin can cause a life-threatening allergic response characterized by bronchospasm, laryngeal edema, and a precipitous drop in blood pressure. This client's ingestion of penicillin and presenting clinical picture indicates the client is having an acute reaction (A) with respiratory difficulty. (B, C, and D) are other physiologic responses to medications, but immediate action is required for a potential loss of airway, breathing, and circulation (A).

Two hours after taking the first dose of penicillin, a client arrives at the emergency department complaining of feeling ill, exhibiting hives, having difficulty breathing, and experiencing hypotension. These findings are consistent with which client response that requires immediate action? A. Severe acute anaphylactic response B. Side reaction that should resolve C. Idiosyncratic reaction D. Cumulative drug response

________________ are drugs that raise the blood pressure because of their ability to constrict blood vessels

Vasopressors

Adrenergic drugs produce what type of effects in the CNS?

Wakefulness, quick reaction to stimuli, quickened reflexes

This adrenergic receptor works on peripheral blood vessels and causes vasoconstriction of peripheral blood vessels

a1

This adrenergic receptor works on the presynaptic neuron and regulates the release of neurotransmitters and decreases tone, motility, and secretions of the GI tract

a2

Examples of Antivirals

abacivir, zanamivir

Examples of Antivirals

acyclovir, valacyclovir

Examples of Bronchodilators

albuterol, bitoiterol, levalbuterol, pirbuterol

Examples of Potassium-sparing diuretics

aldactone, spironolactone

Examples of Biphosphonates

alendronate, etidronate, pamidronate, risedronate

Examples of Benzodiazepines (anxiolytics)

alprazolam, midazolam

Examples of Tricyclics (antidepressants)

amitriptyline, nortriptyline, protriptyline

Examples of Calcium Channel Blockers

amlodipine, felodipine, isradipine, nifedipine

Examples of Barbiturates (sedative-hypnotics)

amobarbital, pentoarbial, secobarbital

Examples of Stimulants

amphetamine, caffeine, terbutaline, theophyline

Examples of HIV/AIDS antivirals

amprenavir, indinavir, nelfinavir, ritonavir

Examples of Beta Blockers (adrenergic antagonists)

atenolol, propanolol, sodalol

Examples of Antilipemics (anti-cholesterol)

atorvastatin, lovastatin, simvastain

This adrenergic receptor works on the myocardium and increases heart rate and the force of myocardial contraction

b1 (1 heart)

Examples of Calcium & Vitamin D Supplements

calciferol, calcitrol, ergocalciferol

Examples of Angiotensin-II receptor antagonists

candesartan, losartan, telmisartan, valsartan

Examples of Antineoplastics (anti-tumor)

carmustine, lomustine, vinblastine, vincristine

Examples of Cephalosporins

cefadroxil, cefactor, celfixime, ceftibuten

Examples of Cepalosporins

cephalexin, cephapirin, cephradine

Examples of H2 receptor antagonists (anti-ulcer)

cimetidine, famotidine, nizatdine, ranitidine

Examples of Fluoroquinolones (Antibiotics)

ciprofloxacin, gatifloxacin, levofloxacin

Examples of Corticosteroids (anti-inflammatory)

cortisone, dexamethasone, prednisone

Examples of General Anesthetics

cyclohexane, ethane, fluorane

Examples of Tetracyclines (Antibiotics)

demeclocycline, doxycycline, minocycline

Examples of Benzodiazepines (anxiolytics)

diazepam, lorazepam

Examples of 5-HT2 receptor antagonists (antiemetics)

dolansetron, granetron, ondansetron

Examples of Alpha blockers (adrenergic antagonists)

doxazosin, prazosin, terazosin

Examples of Thrombolytics (clot-busters)

eminase, retavase, streptokinase

Examples of Ergotamines (anti-migraine)

ergotamine, dihydroxyergotamine

Examples of Aminoglycosides/Macrolides (antibiotics)

erythromycin, tobramycin, vancomycin

Examples of Neuromuscular Blockers

famciclovir, ganciclovir

Examples of NSAIDS (anti-inflammatory)

fenoprofen, iburprofen, ketoprofen

Examples Aminoglycosides (antibiotics)

gentamicin

Examples Proton Pump Inhibitors (anti-ulcers)

lansoprazole, omeprazole, pantoprazole

Examples of Corticosteroids (anti-inflammatory)

methylprednisolone, prednisolone, triamcinolone

Examples of Anti-migraines

naratiptan, rizatriptan

Examples of Cholinergics

neostigmine, physostigmine, pyridostigmine

Examples of Nasal Decongestants

oxymetazoline, zylometazoline

Examples of Neuromuscular Blockers

pancuroniu, pipecuronium, rocuronium

Client takes nonsteroidal anti-inflammatory drugs every day for rheumatoid arthritis is being treated for anemia which intervention is most important for the nurse to include any plan of care observe for gastrointestinal bleeding monitor liver function test results protect skin from bruising offered dietary selections rich in iron rationale

rationale the client is at risk for gastrointestinal bleeding duty history of NSAID a .new onset anemia indicates bleeding may be present. NSAIDS May impact liver function b, but this is of less immediacy than a. C is a basic core measure but not one of high priority for this client. D often useful for persons with anemia but is of less priority than a

Examples of Antituberculars

rifamate, rifampin, rifapentine rifater

Examples of Sulfonamides (antibiotics)

sulfadiazine, sulfamethazole, sulfisoxazole

A patient has been prescribed midodrine. Which of the following is an adverse effect of midodrine that the nurse should monitor for? 1. Supine HTN 2. Tachycardia 3. Orthostatic hypotension 4. Respiratory distress

1

At what intervals would the nurse monitor the blood pressure of a patient administered norepinephrine? 1. Continuously 2. Every 30 mins 3. Every hour 4. Every 4 hours

1

Which of the following adrenergic receptors is responsible for decreased tone, motility, and secretions of the GI tract? 1. a2 receptors 2. b1 receptors 3. a1 receptors 4. b2 receptors

1

Which of the following adrenergic receptors is responsible for the vasoconstriction of peripheral blood vessels? 1. Alpha receptors 2. a2 receptors 3. Beta receptors 4. b2 receptors

1

Which of the following is stimulated during the body's fight-or-flight response to a stressful condition? 1. Sympathetic nervous system 2. Parasympathetic nervous system 3. CNS 4. Somatic nervous system

1

Which of the following is the primary neurotransmitter of the sympathetic nervous system? 1. Norepinephrine 2. Dopamine 3. Serotonin 4. Acetylcholine

1

220.) A adult client with muscle spasms is taking an oral maintenance dose of baclofen (Lioresal). The nurse reviews the medication record, expecting that which dose should be prescribed? 1. 15 mg four times a day 2. 25 mg four times a day 3. 30 mg four times a day 4. 40 mg four times a day

1. 15 mg four times a day Rationale: Baclofen is dispensed in 10- and 20-mg tablets for oral use. Dosages are low initially and then gradually increased. Maintenance doses range from 15 to 20 mg administered three or four times a day.

145.) A nurse has a prescription to give a client albuterol (Proventil HFA) (two puffs) and beclomethasone dipropionate (Qvar) (nasal inhalation, two puffs), by metered-dose inhaler. The nurse administers the medication by giving the: 1. Albuterol first and then the beclomethasone dipropionate 2. Beclomethasone dipropionate first and then the albuterol 3. Alternating a single puff of each, beginning with the albuterol 4. Alternating a single puff of each, beginning with the beclomethasone dipropionate

1. Albuterol first and then the beclomethasone dipropionate Rationale: Albuterol is a bronchodilator. Beclomethasone dipropionate is a glucocorticoid. Bronchodilators are always administered before glucocorticoids when both are to be given on the same time schedule. This allows for widening of the air passages by the bronchodilator, which then makes the glucocorticoid more effective.

100.) Saquinavir (Invirase) is prescribed for the client who is human immunodeficiency virus seropositive. The nurse reinforces medication instructions and tells the client to: 1. Avoid sun exposure. 2. Eat low-calorie foods. 3. Eat foods that are low in fat. 4. Take the medication on an empty stomach.

1. Avoid sun exposure. Rationale: Saquinavir (Invirase) is an antiretroviral (protease inhibitor) used with other antiretroviral medications to manage human immunodeficiency virus infection. Saquinavir is administered with meals and is best absorbed if the client consumes high-calorie, high-fat meals. Saquinavir can cause photosensitivity, and the nurse should instruct the client to avoid sun exposure.

32.) Desmopressin acetate (DDAVP) is prescribed for the treatment of diabetes insipidus. The nurse monitors the client after medication administration for which therapeutic response? 1. Decreased urinary output 2. Decreased blood pressure 3. Decreased peripheral edema 4. Decreased blood glucose level

1. Decreased urinary output Rationale: Desmopressin promotes renal conservation of water. The hormone carries out this action by acting on the collecting ducts of the kidney to increase their permeability to water, which results in increased water reabsorption. The therapeutic effect of this medication would be manifested by a decreased urine output. Options 2, 3, and 4 are unrelated to the effects of this medication.

105.) A nurse is collecting data from a client and the client's spouse reports that the client is taking donepezil hydrochloride (Aricept). Which disorder would the nurse suspect that this client may have based on the use of this medication? 1. Dementia 2. Schizophrenia 3. Seizure disorder 4. Obsessive-compulsive disorder

1. Dementia Rationale: Donepezil hydrochloride is a cholinergic agent used in the treatment of mild to moderate dementia of the Alzheimer type. It enhances cholinergic functions by increasing the concentration of acetylcholine. It slows the progression of Alzheimer's disease. Options 2, 3, and 4 are incorrect.

193.) Sodium hypochlorite (Dakin's solution) is prescribed for a client with a leg wound containing purulent drainage. The nurse is assisting in developing a plan of care for the client and includes which of the following in the plan? 1. Ensure that the solution is freshly prepared before use. 2. Soak a sterile dressing with solution and pack into the wound. 3. Allow the solution to remain in the wound following irrigation. 4. Apply the solution to the wound and on normal skin tissue surrounding the wound.

1. Ensure that the solution is freshly prepared before use. Rationale: Dakin solution is a chloride solution that is used for irrigating and cleaning necrotic or purulent wounds. It can be used for packing necrotic wounds. It cannot be used to pack purulent wounds because the solution is inactivated by copious pus. It should not come into contact with healing or normal tissue, and it should be rinsed off immediately if used for irrigation. Solutions are unstable and the nurse must ensure that the solution has been prepared fresh before use. **Eliminate options 2 and 3 first because they are comparable or alike. It makes sense to ensure that the solution is freshly prepared; therefore, select option 1**

167.) A nurse prepares to reinforce instructions to a client who is taking allopurinol (Zyloprim). The nurse plans to include which of the following in the instructions? 1. Instruct the client to drink 3000 mL of fluid per day. 2. Instruct the client to take the medication on an empty stomach. 3. Inform the client that the effect of the medication will occur immediately. 4. Instruct the client that, if swelling of the lips occurs, this is a normal expected response.

1. Instruct the client to drink 3000 mL of fluid per day. Rationale: Allopurinol (Zyloprim) is an antigout medication used to decrease uric acid levels. Clients taking allopurinol are encouraged to drink 3000 mL of fluid a day. A full therapeutic effect may take 1 week or longer. Allopurinol is to be given with or immediately following meals or milk to prevent gastrointestinal irritation. If the client develops a rash, irritation of the eyes, or swelling of the lips or mouth, he or she should contact the health care provider because this may indicate hypersensitivity.

113.) A nurse is reinforcing discharge instructions to a client receiving sulfisoxazole. Which of the following would be included in the plan of care for instructions? 1. Maintain a high fluid intake. 2. Discontinue the medication when feeling better. 3. If the urine turns dark brown, call the health care provider immediately. 4. Decrease the dosage when symptoms are improving to prevent an allergic response.

1. Maintain a high fluid intake. Rationale: Each dose of sulfisoxazole should be administered with a full glass of water, and the client should maintain a high fluid intake. The medication is more soluble in alkaline urine. The client should not be instructed to taper or discontinue the dose. Some forms of sulfisoxazole cause the urine to turn dark brown or red. This does not indicate the need to notify the health care provider.

109.) A client taking buspirone (BuSpar) for 1 month returns to the clinic for a follow-up visit. Which of the following would indicate medication effectiveness? 1. No rapid heartbeats or anxiety 2. No paranoid thought processes 3. No thought broadcasting or delusions 4. No reports of alcohol withdrawal symptoms

1. No rapid heartbeats or anxiety Rationale: Buspirone hydrochloride is not recommended for the treatment of drug or alcohol withdrawal, paranoid thought disorders, or schizophrenia (thought broadcasting or delusions). Buspirone hydrochloride is most often indicated for the treatment of anxiety and aggression.

235.) A tricyclic antidepressant is administered to a client daily. The nurse plans to monitor for the common side effects of the medication and includes which of the following in the plan of care? 1. Offer hard candy or gum periodically. 2. Offer a nutritious snack between meals. 3. Monitor the blood pressure every 2 hours. 4. Review the white blood cell (WBC) count results daily.

1. Offer hard candy or gum periodically. Rationale: Dry mouth is a common side effect of tricyclic antidepressants. Frequent mouth rinsing with water, sucking on hard candy, and chewing gum will alleviate this common side effect. It is not necessary to monitor the blood pressure every 2 hours. In addition, it is not necessary to check the WBC daily. Weight gain is a common side effect and frequent snacks will aggravate this problem.

92.) In monitoring a client's response to disease-modifying antirheumatic drugs (DMARDs), which findings would the nurse interpret as acceptable responses? Select all that apply. 1. Symptom control during periods of emotional stress 2. Normal white blood cell counts, platelet, and neutrophil counts 3. Radiological findings that show nonprogression of joint degeneration 4. An increased range of motion in the affected joints 3 months into therapy 5. Inflammation and irritation at the injection site 3 days after injection is given 6. A low-grade temperature upon rising in the morning that remains throughout the day

1. Symptom control during periods of emotional stress 2. Normal white blood cell counts, platelet, and neutrophil counts 3. Radiological findings that show nonprogression of joint degeneration 4. An increased range of motion in the affected joints 3 months into therapy Rationale: Because emotional stress frequently exacerbates the symptoms of rheumatoid arthritis, the absence of symptoms is a positive finding. DMARDs are given to slow progression of joint degeneration. In addition, the improvement in the range of motion after 3 months of therapy with normal blood work is a positive finding. Temperature elevation and inflammation and irritation at the medication injection site could indicate signs of infection.

The nurse is administering 0900 medications to the following clients. Which client should the nurse question administering the medication? 1. The client receiving a calcium channel blocker who drank a glass of grapefruit juice. 2. The client receiving a beta blocker who has an apical pulse of 62 bpm. 3. The client receiving a NTG patch who has a blood pressure of 148/92. 4. The client receiving an antiplatelet medication who has a platelet count of 150,000.

1. The client receiving a calcium channel blocker (CCB) should avoid grapefruit juice because it can cause the CCB to rise to toxic levels.

The client is complaining of severe chest pain radiating down the left arm and is nauseated and diaphoretic. The HCP suspects the client is having a myocardial infarction (MI) and has ordered morphine sulfate (MS), a narcotic analgesic, for the pain. Which interventions should the nurse implement? (Select all that apply) 1. Instruct the client not to get up out of the bed without notifying the nurse. 2. Administer the morphine sulfate intramuscularly in the ventral gluteal muscle. 3. Dilute the morphine sulfate to a 10-mL bolus with normal saline. 4. Administer the morphine sulfate slowly over 5 minutes. 5. Question the order because morphine sulfate should not be administered to a patient with a myocardial infarction.

1. The client should not get out of the bed without assistance due to the drowsiness the client will experience after receiving MS. Also, the client is having chest pain and should not get out of the bed without assistance. 3. MS is the drug of choice for chest pain, and it is administered IV so that it acts as soon as possible, within 10-15 minutes. IV push medications should be diluted to help decrease the pain when it is administered and to prevent irritation to the vein. 4. An IV push also allows the nurse to inject the medication more accurately over the 5-minute administration time.

221.) A nurse is reviewing the health care provider's prescriptions for an adult client who has been admitted to the hospital following a back injury. Carisoprodol (Soma) is prescribed for the client to relieve the muscle spasms; the health care provider has prescribed 350 mg to be administered four times a day. When preparing to give this medication, the nurse determines that this dosage is: 1. The normal adult dosage 2. A lower than normal dosage 3. A higher than normal dosage 4. A dosage requiring further clarification

1. The normal adult dosage Rationale: The normal adult dosage for carisoprodol is 350 mg orally three or four times daily.

The client with CAD is prescribed cholestyramine, a bile-acid sequestrant. Which intervention should the nurse implement when administering the medication? 1. Administer the medication with fruit juice. 2. Instruct the client to decrease fiber when taking the medication. 3. Monitor the cholesterol level before giving medication. 4. Assess the client for upper-abdominal discomfort.

1. This medication should be administered with water, fruit juice, soup, or pulpy fruit (applesauce, pineapple) to reduce the risk of esophageal irritation and impintervention.

21.) A nurse is assisting with caring for a client with cancer who is receiving cisplatin. Select the adverse effects that the nurse monitors for that are associated with this medication. Select all that apply. 1. Tinnitus 2. Ototoxicity 3. Hyperkalemia 4. Hypercalcemia 5. Nephrotoxicity 6. Hypomagnesemia

1. Tinnitus 2. Ototoxicity 5. Nephrotoxicity 6. Hypomagnesemia Rationale: Cisplatin is an alkylating medication. Alkylating medications are cell cycle phase-nonspecific medications that affect the synthesis of DNA by causing the cross-linking of DNA to inhibit cell reproduction. Cisplatin may cause ototoxicity, tinnitus, hypokalemia, hypocalcemia, hypomagnesemia, and nephrotoxicity. Amifostine (Ethyol) may be administered before cisplatin to reduce the potential for renal toxicity.

8.) A client with severe acne is seen in the clinic and the health care provider (HCP) prescribes isotretinoin. The nurse reviews the client's medication record and would contact the (HCP) if the client is taking which medication? 1. Vitamin A 2. Digoxin (Lanoxin) 3. Furosemide (Lasix) 4. Phenytoin (Dilantin)

1. Vitamin A Rationale: Isotretinoin is a metabolite of vitamin A and can produce generalized intensification of isotretinoin toxicity. Because of the potential for increased toxicity, vitamin A supplements should be discontinued before isotretinoin therapy. Options 2, 3, and 4 are not contraindicated with the use of isotretinoin.

79.) Ibuprofen (Advil) is prescribed for a client. The nurse tells the client to take the medication: 1. With 8 oz of milk 2. In the morning after arising 3. 60 minutes before breakfast 4. At bedtime on an empty stomach

1. With 8 oz of milk Rationale: Ibuprofen is a nonsteroidal anti-inflammatory drug (NSAID). NSAIDs should be given with milk or food to prevent gastrointestinal irritation. Options 2, 3, and 4 are incorrect.

24.) A nurse is reinforcing teaching for a client regarding how to mix regular insulin and NPH insulin in the same syringe. Which of the following actions, if performed by the client, indicates the need for further teaching? 1. Withdraws the NPH insulin first 2. Withdraws the regular insulin first 3. Injects air into NPH insulin vial first 4. Injects an amount of air equal to the desired dose of insulin into the vial

1. Withdraws the NPH insulin first Rationale: When preparing a mixture of regular insulin with another insulin preparation, the regular insulin is drawn into the syringe first. This sequence will avoid contaminating the vial of regular insulin with insulin of another type. Options 2, 3, and 4 identify the correct actions for preparing NPH and regular insulin.

6.) The burn client is receiving treatments of topical mafenide acetate (Sulfamylon) to the site of injury. The nurse monitors the client, knowing that which of the following indicates that a systemic effect has occurred? 1.Hyperventilation 2.Elevated blood pressure 3.Local pain at the burn site 4.Local rash at the burn site

1.Hyperventilation Rationale: Mafenide acetate is a carbonic anhydrase inhibitor and can suppress renal excretion of acid, thereby causing acidosis. Clients receiving this treatment should be monitored for signs of an acid-base imbalance (hyperventilation). If this occurs, the medication should be discontinued for 1 to 2 days. Options 3 and 4 describe local rather than systemic effects. An elevated blood pressure may be expected from the pain that occurs with a burn injury.

A 65 YO patient has been prescribed isoproterenol. Which of the following should the nurse report immediately to the doctor? 1. Feeling of nausea 2. Changes in pulse rate 3. Severe headache 4. Urinary urgency

2

For which of the following patients is isoproterenol C/I? 1. Patient with narrow-angle glaucoma 2. Patient with tachycardia 3. Patient with hypotension 4. Patient with pheochromocytoma

2

If a patient uses an auto-inject epineprhine device after an allergic reaction, the best disposal would be... 1. Container the injector is packaged in 2. Held until seen by the emergency dept 3. Hard plastic container 4. Sharps box or needle container

2

The adrenergic branch of the autonomic nervous system is also known by which of the following names? 1. Parasympathetic nervous system 2. Sympathetic nervous system 3. CNS 4. Somatic nervous system

2

The doctor prescribes norepinephrine, a potent vasopressor, to be administered to a patient in shock. The rate of administration of the IV fluid containing norepinephrine is: 1. Maintained at a set rate of infusion 2. Adjusted per protocol to maintain the patient's blood pressure 3. Given at a rate not to exceed 5 mg/min 4. Discontinued when BP is 100 mmHg systolic

2

What is the transmitting substance in the sympathetic branch of the nervous system? 1. Serotonin 2. Norepinephrine 3. Dopamine 4. Acetylcholine

2

194.) A nurse provides instructions to a client regarding the use of tretinoin (Retin-A). Which statement by the client indicates the need for further instructions? 1. "Optimal results will be seen after 6 weeks." 2. "I should apply a very thin layer to my skin." 3. "I should wash my hands thoroughly after applying the medication." 4. "I should cleanse my skin thoroughly before applying the medication."

2. "I should apply a very thin layer to my skin." Rationale: Tretinoin is applied liberally to the skin. The hands are washed thoroughly immediately after applying. Therapeutic results should be seen after 2 to 3 weeks but may not be optimal until after 6 weeks. The skin needs to be cleansed thoroughly before applying the medication.

215.) A client with rheumatoid arthritis is taking acetylsalicylic acid (aspirin) on a daily basis. Which medication dose should the nurse expect the client to be taking? 1. 1 g daily 2. 4 g daily 3. 325 mg daily 4. 1000 mg daily

2. 4 g daily Rationale: Aspirin may be used to treat the client with rheumatoid arthritis. It may also be used to reduce the risk of recurrent transient ischemic attack (TIA) or brain attack (stroke) or reduce the risk of myocardial infarction (MI) in clients with unstable angina or a history of a previous MI. The normal dose for clients being treated with aspirin to decrease thrombosis and MI is 300 to 325 mg/day. Clients being treated to prevent TIAs are usually prescribed 1.3 g/day in two to four divided doses. Clients with rheumatoid arthritis are treated with 3.6 to 5.4 g/day in divided doses. **Eliminate options 1 and 4 because they are alike**

64.) Nalidixic acid (NegGram) is prescribed for a client with a urinary tract infection. On review of the client's record, the nurse notes that the client is taking warfarin sodium (Coumadin) daily. Which prescription should the nurse anticipate for this client? 1. Discontinuation of warfarin sodium (Coumadin) 2. A decrease in the warfarin sodium (Coumadin) dosage 3. An increase in the warfarin sodium (Coumadin) dosage 4. A decrease in the usual dose of nalidixic acid (NegGram)

2. A decrease in the warfarin sodium (Coumadin) dosage Rationale: Nalidixic acid can intensify the effects of oral anticoagulants by displacing these agents from binding sites on plasma protein. When an oral anticoagulant is combined with nalidixic acid, a decrease in the anticoagulant dosage may be needed.

88.) Dantrolene sodium (Dantrium) is prescribed for a client experiencing flexor spasms, and the client asks the nurse about the action of the medication. The nurse responds, knowing that the therapeutic action of this medication is which of the following? 1. Depresses spinal reflexes 2. Acts directly on the skeletal muscle to relieve spasticity 3. Acts within the spinal cord to suppress hyperactive reflexes 4. Acts on the central nervous system (CNS) to suppress spasms

2. Acts directly on the skeletal muscle to relieve spasticity Rationale: Dantrium acts directly on skeletal muscle to relieve muscle spasticity. The primary action is the suppression of calcium release from the sarcoplasmic reticulum. This in turn decreases the ability of the skeletal muscle to contract. **Options 1, 3, and 4 are all comparable or alike in that they address CNS suppression and the depression of reflexes. Therefore, eliminate these options.**

74.) A client with myasthenia gravis is receiving pyridostigmine (Mestinon). The nurse monitors for signs and symptoms of cholinergic crisis caused by overdose of the medication. The nurse checks the medication supply to ensure that which medication is available for administration if a cholinergic crisis occurs? 1. Vitamin K 2. Atropine sulfate 3. Protamine sulfate 4. Acetylcysteine (Mucomyst)

2. Atropine sulfate Rationale: The antidote for cholinergic crisis is atropine sulfate. Vitamin K is the antidote for warfarin (Coumadin). Protamine sulfate is the antidote for heparin, and acetylcysteine (Mucomyst) is the antidote for acetaminophen (Tylenol).

86.) A nurse is reinforcing discharge instructions to a client receiving baclofen (Lioresal). Which of the following would the nurse include in the instructions? 1. Restrict fluid intake. 2. Avoid the use of alcohol. 3. Stop the medication if diarrhea occurs. 4. Notify the health care provider if fatigue occurs.

2. Avoid the use of alcohol. Rationale: Baclofen is a central nervous system (CNS) depressant. The client should be cautioned against the use of alcohol and other CNS depressants, because baclofen potentiates the depressant activity of these agents. Constipation rather than diarrhea is an adverse effect of baclofen. It is not necessary to restrict fluids, but the client should be warned that urinary retention can occur. Fatigue is related to a CNS effect that is most intense during the early phase of therapy and diminishes with continued medication use. It is not necessary that the client notify the health care provider if fatigue occurs.

170.) Atenolol hydrochloride (Tenormin) is prescribed for a hospitalized client. The nurse should perform which of the following as a priority action before administering the medication? 1. Listen to the client's lung sounds. 2. Check the client's blood pressure. 3. Check the recent electrolyte levels. 4. Assess the client for muscle weakness.

2. Check the client's blood pressure. Rationale: Atenolol hydrochloride is a beta-blocker used to treat hypertension. Therefore the priority nursing action before administration of the medication is to check the client's blood pressure. The nurse also checks the client's apical heart rate. If the systolic blood pressure is below 90 mm Hg or the apical pulse is 60 beats per minute or lower, the medication is withheld and the registered nurse and/or health care provider is notified. The nurse would check baseline renal and liver function tests. The medication may cause weakness, and the nurse would assist the client with activities if weakness occurs. **Beta-blockers have "-lol" at the end of the medication name**

34.) A client with Crohn's disease is scheduled to receive an infusion of infliximab (Remicade). The nurse assisting in caring for the client should take which action to monitor the effectiveness of treatment? 1. Monitoring the leukocyte count for 2 days after the infusion 2. Checking the frequency and consistency of bowel movements 3. Checking serum liver enzyme levels before and after the infusion 4. Carrying out a Hematest on gastric fluids after the infusion is completed

2. Checking the frequency and consistency of bowel movements Rationale: The principal manifestations of Crohn's disease are diarrhea and abdominal pain. Infliximab (Remicade) is an immunomodulator that reduces the degree of inflammation in the colon, thereby reducing the diarrhea. Options 1, 3, and 4 are unrelated to this medication.

63.) A client with coronary artery disease complains of substernal chest pain. After checking the client's heart rate and blood pressure, a nurse administers nitroglycerin, 0.4 mg, sublingually. After 5 minutes, the client states, "My chest still hurts." Select the appropriate actions that the nurse should take. Select all that apply. 1. Call a code blue. 2. Contact the registered nurse. 3. Contact the client's family. 4. Assess the client's pain level. 5. Check the client's blood pressure. 6. Administer a second nitroglycerin, 0.4 mg, sublingually.

2. Contact the registered nurse. 4. Assess the client's pain level. 5. Check the client's blood pressure. 6. Administer a second nitroglycerin, 0.4 mg, sublingually. Rationale: The usual guideline for administering nitroglycerin tablets for a hospitalized client with chest pain is to administer one tablet every 5 minutes PRN for chest pain, for a total dose of three tablets. The registered nurse should be notified of the client's condition, who will then notify the health care provider as appropriate. Because the client is still complaining of chest pain, the nurse would administer a second nitroglycerin tablet. The nurse would assess the client's pain level and check the client's blood pressure before administering each nitroglycerin dose. There are no data in the question that indicate the need to call a code blue. In addition, it is not necessary to contact the client's family unless the client has requested this.

41.) The client has been taking omeprazole (Prilosec) for 4 weeks. The ambulatory care nurse evaluates that the client is receiving optimal intended effect of the medication if the client reports the absence of which symptom? 1. Diarrhea 2. Heartburn 3. Flatulence 4. Constipation

2. Heartburn Rationale: Omeprazole is a proton pump inhibitor classified as an antiulcer agent. The intended effect of the medication is relief of pain from gastric irritation, often called heartburn by clients. Omeprazole is not used to treat the conditions identified in options 1, 3, and 4.

101.) Ketoconazole is prescribed for a client with a diagnosis of candidiasis. Select the interventions that the nurse includes when administering this medication. Select all that apply. 1. Restrict fluid intake. 2. Instruct the client to avoid alcohol. 3. Monitor hepatic and liver function studies. 4. Administer the medication with an antacid. 5. Instruct the client to avoid exposure to the sun. 6. Administer the medication on an empty stomach.

2. Instruct the client to avoid alcohol. 3. Monitor hepatic and liver function studies. 5. Instruct the client to avoid exposure to the sun. Rationale: Ketoconazole is an antifungal medication. It is administered with food (not on an empty stomach) and antacids are avoided for 2 hours after taking the medication to ensure absorption. The medication is hepatotoxic and the nurse monitors liver function studies. The client is instructed to avoid exposure to the sun because the medication increases photosensitivity. The client is also instructed to avoid alcohol. There is no reason for the client to restrict fluid intake. In fact, this could be harmful to the client.

65.) A nurse is reinforcing discharge instructions to a client receiving sulfisoxazole. Which of the following should be included in the list of instructions? 1. Restrict fluid intake. 2. Maintain a high fluid intake. 3. If the urine turns dark brown, call the health care provider (HCP) immediately. 4. Decrease the dosage when symptoms are improving to prevent an allergic response.

2. Maintain a high fluid intake. Rationale: Each dose of sulfisoxazole should be administered with a full glass of water, and the client should maintain a high fluid intake. The medication is more soluble in alkaline urine. The client should not be instructed to taper or discontinue the dose. Some forms of sulfisoxazole cause urine to turn dark brown or red. This does not indicate the need to notify the HCP.

161.) A nurse is caring for a client with severe back pain, and codeine sulfate has been prescribed for the client. Which of the following would the nurse include in the plan of care while the client is taking this medication? 1. Restrict fluid intake. 2. Monitor bowel activity. 3. Monitor for hypertension. 4. Monitor peripheral pulses.

2. Monitor bowel activity. Rationale: While the client is taking codeine sulfate, an opioid analgesic, the nurse would monitor vital signs and monitor for hypotension. The nurse should also increase fluid intake, palpate the bladder for urinary retention, auscultate bowel sounds, and monitor the pattern of daily bowel activity and stool consistency (codeine can cause constipation). The nurse should monitor respiratory status and initiate breathing and coughing exercises. In addition, the nurse monitors the effectiveness of the pain medication.

227.) When teaching a client who is being started on imipramine hydrochloride (Tofranil), the nurse would inform the client that the desired effects of the medication may: 1. Start during the first week of administration 2. Not occur for 2 to 3 weeks of administration 3. Start during the second week of administration 4. Not occur until after a month of administration

2. Not occur for 2 to 3 weeks of administration Rationale: The therapeutic effects of administration of imipramine hydrochloride may not occur for 2 to 3 weeks after the antidepressant therapy has been initiated. Therefore options 1, 3, and 4 are incorrect.

47.) A client has been taking isoniazid (INH) for 2 months. The client complains to a nurse about numbness, paresthesias, and tingling in the extremities. The nurse interprets that the client is experiencing: 1. Hypercalcemia 2. Peripheral neuritis 3. Small blood vessel spasm 4. Impaired peripheral circulation

2. Peripheral neuritis Rationale: A common side effect of the TB drug INH is peripheral neuritis. This is manifested by numbness, tingling, and paresthesias in the extremities. This side effect can be minimized by pyridoxine (vitamin B6) intake. Options 1, 3, and 4 are incorrect.

186.) A nurse prepares to administer sodium polystyrene sulfonate (Kayexalate) to a client. Before administering the medication, the nurse reviews the action of the medication and understands that it: 1. Releases bicarbonate in exchange for primarily sodium ions 2. Releases sodium ions in exchange for primarily potassium ions 3. Releases potassium ions in exchange for primarily sodium ions 4. Releases sodium ions in exchange for primarily bicarbonate ions

2. Releases sodium ions in exchange for primarily potassium ions Rationale: Sodium polystyrene sulfonate is a cation exchange resin used in the treatment of hyperkalemia. The resin either passes through the intestine or is retained in the colon. It releases sodium ions in exchange for primarily potassium ions. The therapeutic effect occurs 2 to 12 hours after oral administration and longer after rectal administration.

The client diagnosed with coronary artery disease is prescribed atorvastatin, an HMG-CoA reductase inhibitor. Which statement by the client warrants the nurse notifying the health-care provider? 1. "I really haven't changed my diet, but I am taking my medication every day." 2. "I am feeling pretty good except I am having muscle pain all over my body." 3. "I am swimming at the local pool about three times a week for 30 minutes" 4. "I am taking this medication first thing in the morning with a bowl of oatmeal."

2. Statins can cause muscle injury, which leads to myosititis, fatal rhabdomyolysis, or myopathy. Muscle pain or tenderness should be reported to the HCP immediately; usually the medication is discontinued.

188.) The nurse should anticipate that the most likely medication to be prescribed prophylactically for a child with spina bifida (myelomeningocele) who has a neurogenic bladder would be: 1. Prednisone 2. Sulfisoxazole 3. Furosemide (Lasix) 4. Intravenous immune globulin (IVIG)

2. Sulfisoxazole Rationale: A neurogenic bladder prevents the bladder from completely emptying because of the decrease in muscle tone. The most likely medication to be prescribed to prevent urinary tract infection would be an antibiotic. A common prescribed medication is sulfisoxazole. Prednisone relieves allergic reactions and inflammation rather than preventing infection. Furosemide promotes diuresis and decreases edema caused by congestive heart failure. IVIG assists with antibody production in immunocompromised clients.

147.) A client with a prescription to take theophylline (Theo-24) daily has been given medication instructions by the nurse. The nurse determines that the client needs further information about the medication if the client states that he or she will: 1. Drink at least 2 L of fluid per day. 2. Take the daily dose at bedtime. 3. Avoid changing brands of the medication without health care provider (HCP) approval. 4. Avoid over-the-counter (OTC) cough and cold medications unless approved by the HCP.

2. Take the daily dose at bedtime. Rationale: The client taking a single daily dose of theophylline, a xanthine bronchodilator, should take the medication early in the morning. This enables the client to have maximal benefit from the medication during daytime activities. In addition, this medication causes insomnia. The client should take in at least 2 L of fluid per day to decrease viscosity of secretions. The client should check with the physician before changing brands of the medication. The client also checks with the HCP before taking OTC cough, cold, or other respiratory preparations because they could cause interactive effects, increasing the side effects of theophylline and causing dysrhythmias.

45.) A client has a prescription to take guaifenesin (Humibid) every 4 hours, as needed. The nurse determines that the client understands the most effective use of this medication if the client states that he or she will: 1. Watch for irritability as a side effect. 2. Take the tablet with a full glass of water. 3. Take an extra dose if the cough is accompanied by fever. 4. Crush the sustained-release tablet if immediate relief is needed.

2. Take the tablet with a full glass of water. Rationale: Guaifenesin is an expectorant. It should be taken with a full glass of water to decrease viscosity of secretions. Sustained-release preparations should not be broken open, crushed, or chewed. The medication may occasionally cause dizziness, headache, or drowsiness as side effects. The client should contact the health care provider if the cough lasts longer than 1 week or is accompanied by fever, rash, sore throat, or persistent headache.

214.) A health care provider initiates carbidopa/levodopa (Sinemet) therapy for the client with Parkinson's disease. A few days after the client starts the medication, the client complains of nausea and vomiting. The nurse tells the client that: 1. Taking an antiemetic is the best measure to prevent the nausea. 2. Taking the medication with food will help to prevent the nausea. 3. This is an expected side effect of the medication and will decrease over time. 4. The nausea and vomiting will decrease when the dose of levodopa is stabilized.

2. Taking the medication with food will help to prevent the nausea. Rationale: If carbidopa/levodopa is causing nausea and vomiting, the nurse would tell the client that taking the medication with food will prevent the nausea. Additionally, the client should be instructed not to take the medication with a high-protein meal because the high-protein will affect absorption. Antiemetics from the phenothiazine class should not be used because they block the therapeutic action of dopamine. **eliminate options 3 and 4 because they are comparable or alike**

57.) A nurse is monitoring a client who is taking propranolol (Inderal LA). Which data collection finding would indicate a potential serious complication associated with propranolol? 1. The development of complaints of insomnia 2. The development of audible expiratory wheezes 3. A baseline blood pressure of 150/80 mm Hg followed by a blood pressure of 138/72 mm Hg after two doses of the medication 4. A baseline resting heart rate of 88 beats/min followed by a resting heart rate of 72 beats/min after two doses of the medication

2. The development of audible expiratory wheezes Rationale: Audible expiratory wheezes may indicate a serious adverse reaction, bronchospasm. β-Blockers may induce this reaction, particularly in clients with chronic obstructive pulmonary disease or asthma. Normal decreases in blood pressure and heart rate are expected. Insomnia is a frequent mild side effect and should be monitored.

28.) The health care provider (HCP) prescribes exenatide (Byetta) for a client with type 1 diabetes mellitus who takes insulin. The nurse knows that which of the following is the appropriate intervention? 1. The medication is administered within 60 minutes before the morning and evening meal. 2. The medication is withheld and the HCP is called to question the prescription for the client. 3. The client is monitored for gastrointestinal side effects after administration of the medication. 4. The insulin is withdrawn from the Penlet into an insulin syringe to prepare for administration.

2. The medication is withheld and the HCP is called to question the prescription for the client. Rationale: Exenatide (Byetta) is an incretin mimetic used for type 2 diabetes mellitus only. It is not recommended for clients taking insulin. Hence, the nurse should hold the medication and question the HCP regarding this prescription. Although options 1 and 3 are correct statements about the medication, in this situation the medication should not be administered. The medication is packaged in prefilled pens ready for injection without the need for drawing it up into another syringe.

83.) The client has been on treatment for rheumatoid arthritis for 3 weeks. During the administration of etanercept (Enbrel), it is most important for the nurse to check: 1. The injection site for itching and edema 2. The white blood cell counts and platelet counts 3. Whether the client is experiencing fatigue and joint pain 4. A metallic taste in the mouth, with a loss of appetite

2. The white blood cell counts and platelet counts Rationale: Infection and pancytopenia are side effects of etanercept (Enbrel). Laboratory studies are performed before and during drug treatment. The appearance of abnormal white blood cell counts and abnormal platelet counts can alert the nurse to a potentially life-threatening infection. Injection site itching is a common occurrence following administration. A metallic taste with loss of appetite are not common signs of side effects of this medication.

The client with a serum cholesterol level of 320 mg/dL is taking the antihyperlipidemic medication Zetia. Which statement by the client indicates the client needs more teaching concerning this medication? 1. "This medication helps decrease the absorption of cholesterol in my intestines." 2. "I cannot take this medication with any other cholesterol-lowering medication." 3. "I need to eat a low-fat, low-cholesterol diet even when taking the medication." 4. It will take a few months for my cholesterol level to get down to normal levels."

2. This is not a true statement; therefore, the client needs more teaching. Zetia acts by decreasing cholesterol absorption in the intestine and is used together with statins to help lower cholesterol in clients whose cholesterol levels cannot be controlled by taking statins alone.

126.) A nurse is caring for a client who has been prescribed furosemide (Lasix) and is monitoring for adverse effects associated with this medication. Which of the following should the nurse recognize as a potential adverse effect Select all that apply. 1. Nausea 2. Tinnitus 3. Hypotension 4. Hypokalemia 5. Photosensitivity 6. Increased urinary frequency

2. Tinnitus 3. Hypotension 4. Hypokalemia Rationale: Furosemide is a loop diuretic; therefore, an expected effect is increased urinary frequency. Nausea is a frequent side effect, not an adverse effect. Photosensitivity is an occasional side effect. Adverse effects include tinnitus (ototoxicity), hypotension, and hypokalemia and occur as a result of sudden volume depletion.

7.) Isotretinoin is prescribed for a client with severe acne. Before the administration of this medication, the nurse anticipates that which laboratory test will be prescribed? 1. Platelet count 2. Triglyceride level 3. Complete blood count 4. White blood cell count

2. Triglyceride level Rationale: Isotretinoin can elevate triglyceride levels. Blood triglyceride levels should be measured before treatment and periodically thereafter until the effect on the triglycerides has been evaluated. Options 1, 3, and 4 do not need to be monitored specifically during this treatment.

A nurse is required to administer metaraminol for a patient who is taking digoxin. The patient is at an increased risk for which of the following? 1. Epigastric distress 2. Pheochromocytoma 3. Cardiac arrhythmias 4. Decrease in BP

3

Drugs that produce activity similar to the neurotransmitter norepinephrine are known as which of the following? 1. Sympatholytics 2. Antiadrenergic 3. Sympathomimetics 4. Anticholinergic drugs

3

Shock is described as... 1. Result of blood 2. Compensation for bodily assault 3. Inadequate tissue perfusion 4. The "flight of fight" response

3

164.) A client receives a prescription for methocarbamol (Robaxin), and the nurse reinforces instructions to the client regarding the medication. Which client statement would indicate a need for further instructions? 1. "My urine may turn brown or green." 2. "This medication is prescribed to help relieve my muscle spasms." 3. "If my vision becomes blurred, I don't need to be concerned about it." 4. "I need to call my doctor if I experience nasal congestion from this medication."

3. "If my vision becomes blurred, I don't need to be concerned about it." Rationale: The client needs to be told that the urine may turn brown, black, or green. Other adverse effects include blurred vision, nasal congestion, urticaria, and rash. The client needs to be instructed that, if these adverse effects occur, the health care provider needs to be notified. The medication is used to relieve muscle spasms.

11.) The health care provider has prescribed silver sulfadiazine (Silvadene) for the client with a partial-thickness burn, which has cultured positive for gram-negative bacteria. The nurse is reinforcing information to the client about the medication. Which statement made by the client indicates a lack of understanding about the treatments? 1. "The medication is an antibacterial." 2. "The medication will help heal the burn." 3. "The medication will permanently stain my skin." 4. "The medication should be applied directly to the wound."

3. "The medication will permanently stain my skin." Rationale: Silver sulfadiazine (Silvadene) is an antibacterial that has a broad spectrum of activity against gram-negative bacteria, gram-positive bacteria, and yeast. It is applied directly to the wound to assist in healing. It does not stain the skin.

191.) A child is brought to the emergency department for treatment of an acute asthma attack. The nurse prepares to administer which of the following medications first? 1. Oral corticosteroids 2. A leukotriene modifier 3. A β2 agonist 4. A nonsteroidal anti-inflammatory

3. A β2 agonist Rationale: In treating an acute asthma attack, a short acting β2 agonist such as albuterol (Proventil HFA) will be given to produce bronchodilation. Options 1, 2, and 4 are long-term control (preventive) medications.

233.) Diphenhydramine hydrochloride (Benadryl) is used in the treatment of allergic rhinitis for a hospitalized client with a chronic psychotic disorder. The client asks the nurse why the medication is being discontinued before hospital discharge. The nurse responds, knowing that: 1. Allergic symptoms are short in duration. 2. This medication promotes long-term extrapyramidal symptoms. 3. Addictive properties are enhanced in the presence of psychotropic medications. 4. Poor compliance causes this medication to fail to reach its therapeutic blood level.

3. Addictive properties are enhanced in the presence of psychotropic medications. Rationale: The addictive properties of diphenhydramine hydrochloride are enhanced when used with psychotropic medications. Allergic symptoms may not be short term and will occur if allergens are present in the environment. Poor compliance may be a problem with psychotic clients but is not the subject of the question. Diphenhydramine hydrochloride may be used for extrapyramidal symptoms and mild medication-induced movement disorders.

207.) A client is suspected of having myasthenia gravis, and the health care provider administers edrophonium (Enlon) to determine the diagnosis. After administration of this medication, which of the following would indicate the presence of myasthenia gravis? 1. Joint pain 2. A decrease in muscle strength 3. An increase in muscle strength 4. Feelings of faintness, dizziness, hypotension, and signs of flushing in the client

3. An increase in muscle strength Rationale: Edrophonium is a short-acting acetylcholinesterase inhibitor used as a diagnostic agent. When a client with suspected myasthenia gravis is given the medication intravenously, an increase in muscle strength would be seen in 1 to 3 minutes. If no response occurs, another dose is given over the next 2 minutes, and muscle strength is again tested. If no increase in muscle strength occurs with this higher dose, the muscle weakness is not caused by myasthenia gravis. Clients receiving injections of this medication commonly demonstrate a drop of blood pressure, feel faint and dizzy, and are flushed.

96.) The nurse is caring for a postrenal transplant client taking cyclosporine (Sandimmune, Gengraf, Neoral). The nurse notes an increase in one of the client's vital signs, and the client is complaining of a headache. What is the vital sign that is most likely increased? 1. Pulse 2. Respirations 3. Blood pressure 4. Pulse oximetry

3. Blood pressure Rationale: Hypertension can occur in a client taking cyclosporine (Sandimmune, Gengraf, Neoral), and because this client is also complaining of a headache, the blood pressure is the vital sign to be monitoring most closely. Other adverse effects include infection, nephrotoxicity, and hirsutism. Options 1, 2, and 4 are unrelated to the use of this medication.

153.) A client is diagnosed with pulmonary embolism and is to be treated with streptokinase (Streptase). A nurse would report which priority data collection finding to the registered nurse before initiating this therapy? 1. Adventitious breath sounds 2. Temperature of 99.4° F orally 3. Blood pressure of 198/110 mm Hg 4. Respiratory rate of 28 breaths/min

3. Blood pressure of 198/110 mm Hg Rationale: Thrombolytic therapy is contraindicated in a number of preexisting conditions in which there is a risk of uncontrolled bleeding, similar to the case in anticoagulant therapy. Thrombolytic therapy also is contraindicated in severe uncontrolled hypertension because of the risk of cerebral hemorrhage. Therefore the nurse would report the results of the blood pressure to the registered nurse before initiating therapy. The findings in options 1, 2, and 4 may be present in the client with pulmonary embolism.

1) A nurse is caring for a client with hyperparathyroidism and notes that the client's serum calcium level is 13 mg/dL. Which medication should the nurse prepare to administer as prescribed to the client? 1. Calcium chloride 2. Calcium gluconate 3. Calcitonin (Miacalcin) 4. Large doses of vitamin D

3. Calcitonin (Miacalcin) Rationale: The normal serum calcium level is 8.6 to 10.0 mg/dL. This client is experiencing hypercalcemia. Calcium gluconate and calcium chloride are medications used for the treatment of tetany, which occurs as a result of acute hypocalcemia. In hypercalcemia, large doses of vitamin D need to be avoided. Calcitonin, a thyroid hormone, decreases the plasma calcium level by inhibiting bone resorption and lowering the serum calcium concentration.

16.) The clinic nurse is reviewing a teaching plan for the client receiving an antineoplastic medication. When implementing the plan, the nurse tells the client: 1. To take aspirin (acetylsalicylic acid) as needed for headache 2. Drink beverages containing alcohol in moderate amounts each evening 3. Consult with health care providers (HCPs) before receiving immunizations 4. That it is not necessary to consult HCPs before receiving a flu vaccine at the local health fair

3. Consult with health care providers (HCPs) before receiving immunizations Rationale: Because antineoplastic medications lower the resistance of the body, clients must be informed not to receive immunizations without a HCP's approval. Clients also need to avoid contact with individuals who have recently received a live virus vaccine. Clients need to avoid aspirin and aspirin-containing products to minimize the risk of bleeding, and they need to avoid alcohol to minimize the risk of toxicity and side effects.

150.) A client complaining of not feeling well is seen in a clinic. The client is taking several medications for the control of heart disease and hypertension. These medications include a β-blocker, digoxin (Lanoxin), and a diuretic. A tentative diagnosis of digoxin toxicity is made. Which of the following assessment data would support this diagnosis? 1. Dyspnea, edema, and palpitations 2. Chest pain, hypotension, and paresthesia 3. Double vision, loss of appetite, and nausea 4. Constipation, dry mouth, and sleep disorder

3. Double vision, loss of appetite, and nausea Rationale: Double vision, loss of appetite, and nausea are signs of digoxin toxicity. Additional signs of digoxin toxicity include bradycardia, difficulty reading, visual alterations such as green and yellow vision or seeing spots or halos, confusion, vomiting, diarrhea, decreased libido, and impotence. **gastrointestinal (GI) and visual disturbances occur with digoxin toxicity**

138.) A daily dose of prednisone is prescribed for a client. A nurse reinforces instructions to the client regarding administration of the medication and instructs the client that the best time to take this medication is: 1. At noon 2. At bedtime 3. Early morning 4. Anytime, at the same time, each day

3. Early morning Rationale: Corticosteroids (glucocorticoids) should be administered before 9:00 AM. Administration at this time helps minimize adrenal insufficiency and mimics the burst of glucocorticoids released naturally by the adrenal glands each morning. **Note the suffix "-sone," and recall that medication names that end with these letters are corticosteroids.**

244.) A client has a prescription for valproic acid (Depakene) orally once daily. The nurse plans to: 1. Administer the medication with an antacid. 2. Administer the medication with a carbonated beverage. 3. Ensure that the medication is administered at the same time each day. 4. Ensure that the medication is administered 2 hours before breakfast only, when the client's stomach is empty.

3. Ensure that the medication is administered at the same time each day. Rationale: Valproic acid is an anticonvulsant, antimanic, and antimigraine medication. It may be administered with or without food. It should not be taken with an antacid or carbonated beverage because these products will affect medication absorption. The medication is administered at the same time each day to maintain therapeutic serum levels. **Use general pharmacology guidelines to assist in eliminating options 1 and 2. Eliminate option 4 because of the closed-ended word "only."**

243.) A hospitalized client is having the dosage of clonazepam (Klonopin) adjusted. The nurse should plan to: 1. Weigh the client daily. 2. Observe for ecchymosis. 3. Institute seizure precautions. 4. Monitor blood glucose levels.

3. Institute seizure precautions. Rationale: Clonazepam is a benzodiazepine used as an anticonvulsant. During initial therapy and during periods of dosage adjustment, the nurse should initiate seizure precautions for the client. Options 1, 2, and 4 are not associated with the use of this medication.

201.) A nurse is preparing to administer eardrops to an infant. The nurse plans to: 1. Pull up and back on the ear and direct the solution onto the eardrum. 2. Pull down and back on the ear and direct the solution onto the eardrum. 3. Pull down and back on the ear and direct the solution toward the wall of the canal. 4. Pull up and back on the ear lobe and direct the solution toward the wall of the canal.

3. Pull down and back on the ear and direct the solution toward the wall of the canal. Rationale: When administering eardrops to an infant, the nurse pulls the ear down and straight back. In the adult or a child older than 3 years, the ear is pulled up and back to straighten the auditory canal. The medication is administered by aiming it at the wall of the canal rather than directly onto the eardrum.

95.) The nurse is reviewing the results of serum laboratory studies drawn on a client with acquired immunodeficiency syndrome who is receiving didanosine (Videx). The nurse interprets that the client may have the medication discontinued by the health care provider if which of the following significantly elevated results is noted? 1. Serum protein 2. Blood glucose 3. Serum amylase 4. Serum creatinine

3. Serum amylase Rationale: Didanosine (Videx) can cause pancreatitis. A serum amylase level that is increased 1.5 to 2 times normal may signify pancreatitis in the client with acquired immunodeficiency syndrome and is potentially fatal. The medication may have to be discontinued. The medication is also hepatotoxic and can result in liver failure.

187.) A clinic nurse prepares to administer an MMR (measles, mumps, rubella) vaccine to a child. How is this vaccine best administered? 1. Intramuscularly in the deltoid muscle 2. Subcutaneously in the gluteal muscle 3. Subcutaneously in the outer aspect of the upper arm 4. Intramuscularly in the anterolateral aspect of the thigh

3. Subcutaneously in the outer aspect of the upper arm Rationale: The MMR vaccine is administered subcutaneously in the outer aspect of the upper arm. The gluteal muscle is most often used for intramuscular injections. The MMR vaccine is not administered by the intramuscular route.

58.) Isosorbide mononitrate (Imdur) is prescribed for a client with angina pectoris. The client tells the nurse that the medication is causing a chronic headache. The nurse appropriately suggests that the client: 1. Cut the dose in half. 2. Discontinue the medication. 3. Take the medication with food. 4. Contact the health care provider (HCP).

3. Take the medication with food. Rationale: Isosorbide mononitrate is an antianginal medication. Headache is a frequent side effect of isosorbide mononitrate and usually disappears during continued therapy. If a headache occurs during therapy, the client should be instructed to take the medication with food or meals. It is not necessary to contact the HCP unless the headaches persist with therapy. It is not appropriate to instruct the client to discontinue therapy or adjust the dosages.

The nurse is teaching the client diagnosed with angina about sublingual nitroglycerin (NTG), a coronary vasodilator. Which statement indicates the client needs more medication teaching? 1. "I will always carry my nitroglycerin in a dark-colored bottle." 2. "If I have chest pain, I will put a tablet underneath my tongue." 3. If my pain is not relieved with one tablet, I will get medical help." 4. "I should expect to get a headache after taking my nitroglycerin."

3. The client should put one tablet under the tongue every 5 minutes and, if the chest pain is not relieved after taking three tablets, the client should seek medical attention. This statement indicates the client needs more teaching about the medication.

The nurse is preparing to administer nitroglycerin, a coronary vasodilator transdermal patch, to the patient diagnosed with a myocardial infarction. Which intervention should the nurse implement? 1. Question applying the patch if the client's BP is less than 110/70. 2. Use nonsterile gloves when applying the transdermal patch. 3. Date and time the transdermal patch prior to applying to the client's skin. 4. Place the transdermal patch on the site where the old patch was removed.

3. The nurse should remove the old patch, wash the client's skin, note the date and time the new patch is applied, and apply it to a new area that is not hairy.

80.) A nurse is caring for a client who is taking phenytoin (Dilantin) for control of seizures. During data collection, the nurse notes that the client is taking birth control pills. Which of the following information should the nurse provide to the client? 1. Pregnancy should be avoided while taking phenytoin (Dilantin). 2. The client may stop taking the phenytoin (Dilantin) if it is causing severe gastrointestinal effects. 3. The potential for decreased effectiveness of the birth control pills exists while taking phenytoin (Dilantin). 4. The increased risk of thrombophlebitis exists while taking phenytoin (Dilantin) and birth control pills together.

3. The potential for decreased effectiveness of the birth control pills exists while taking phenytoin (Dilantin). Rationale: Phenytoin (Dilantin) enhances the rate of estrogen metabolism, which can decrease the effectiveness of some birth control pills. Options 1, 2, are 4 are not accurate.

129.) Megestrol acetate (Megace), an antineoplastic medication, is prescribed for the client with metastatic endometrial carcinoma. The nurse reviews the client's history and contacts the registered nurse if which diagnosis is documented in the client's history? 1. Gout 2. Asthma 3. Thrombophlebitis 4. Myocardial infarction

3. Thrombophlebitis Rationale: Megestrol acetate (Megace) suppresses the release of luteinizing hormone from the anterior pituitary by inhibiting pituitary function and regressing tumor size. Megestrol is used with caution if the client has a history of thrombophlebitis. **megestrol acetate is a hormonal antagonist enzyme and that a side effect is thrombotic disorders**

81.) A client with trigeminal neuralgia is being treated with carbamazepine (Tegretol). Which laboratory result would indicate that the client is experiencing an adverse reaction to the medication? 1. Sodium level, 140 mEq/L 2. Uric acid level, 5.0 mg/dL 3. White blood cell count, 3000 cells/mm3 4. Blood urea nitrogen (BUN) level, 15 mg/dL

3. White blood cell count, 3000 cells/mm3 Rationale: Adverse effects of carbamazepine (Tegretol) appear as blood dyscrasias, including aplastic anemia, agranulocytosis, thrombocytopenia, leukopenia, cardiovascular disturbances, thrombophlebitis, dysrhythmias, and dermatological effects. Options 1, 2, and 4 identify normal laboratory values.

62.) A client is on nicotinic acid (niacin) for hyperlipidemia and the nurse provides instructions to the client about the medication. Which statement by the client would indicate an understanding of the instructions? 1. "It is not necessary to avoid the use of alcohol." 2. "The medication should be taken with meals to decrease flushing." 3. "Clay-colored stools are a common side effect and should not be of concern." 4. "Ibuprofen (Motrin) taken 30 minutes before the nicotinic acid should decrease the flushing."

4. "Ibuprofen (Motrin) taken 30 minutes before the nicotinic acid should decrease the flushing." Rationale: Flushing is a side effect of this medication. Aspirin or a nonsteroidal anti-inflammatory drug can be taken 30 minutes before taking the medication to decrease flushing. Alcohol consumption needs to be avoided because it will enhance this side effect. The medication should be taken with meals, this will decrease gastrointestinal upset. Taking the medication with meals has no effect on the flushing. Clay-colored stools are a sign of hepatic dysfunction and should be immediately reported to the health care provider (HCP).

87.) A client with acute muscle spasms has been taking baclofen (Lioresal). The client calls the clinic nurse because of continuous feelings of weakness and fatigue and asks the nurse about discontinuing the medication. The nurse should make which appropriate response to the client? 1. "You should never stop the medication." 2. "It is best that you taper the dose if you intend to stop the medication." 3. "It is okay to stop the medication if you think that you can tolerate the muscle spasms." 4. "Weakness and fatigue commonly occur and will diminish with continued medication use."

4. "Weakness and fatigue commonly occur and will diminish with continued medication use." Rationale: The client should be instructed that symptoms such as drowsiness, weakness, and fatigue are more intense in the early phase of therapy and diminish with continued medication use. The client should be instructed never to withdraw or stop the medication abruptly, because abrupt withdrawal can cause visual hallucinations, paranoid ideation, and seizures. It is best for the nurse to inform the client that these symptoms will subside and encourage the client to continue the use of the medication.

The elderly client diagnosed with CAD has been taking aspirin daily for more than a year. Which data warrant notifying the health-care provider? 1. The client has lost 5 pounds in the last month. 2. The client has trouble hearing low tones. 3. The client reports having a funny taste in the mouth. 4. The client has hard, dark, tarry stools.

4. A complication of long-term aspirin use is gastric bleeding, which could result in dark, tarry stools. This data would warrant further intervention.

163.) A client with trigeminal neuralgia tells the nurse that acetaminophen (Tylenol) is taken on a frequent daily basis for relief of generalized discomfort. The nurse reviews the client's laboratory results and determines that which of the following indicates toxicity associated with the medication? 1. Sodium of 140 mEq/L 2. Prothrombin time of 12 seconds 3. Platelet count of 400,000 cells/mm3 4. A direct bilirubin level of 2 mg/dL

4. A direct bilirubin level of 2 mg/dL Rationale: In adults, overdose of acetaminophen (Tylenol) causes liver damage. Option 4 is an indicator of liver function and is the only option that indicates an abnormal laboratory value. The normal direct bilirubin is 0 to 0.4 mg/dL. The normal platelet count is 150,000 to 400,000 cells/mm3. The normal prothrombin time is 10 to 13 seconds. The normal sodium level is 135 to 145 mEq/L.

75.) A client with myasthenia gravis becomes increasingly weak. The health care provider prepares to identify whether the client is reacting to an overdose of the medication (cholinergic crisis) or increasing severity of the disease (myasthenic crisis). An injection of edrophonium (Enlon) is administered. Which of the following indicates that the client is in cholinergic crisis? 1. No change in the condition 2. Complaints of muscle spasms 3. An improvement of the weakness 4. A temporary worsening of the condition

4. A temporary worsening of the condition Rationale: An edrophonium (Enlon) injection, a cholinergic drug, makes the client in cholinergic crisis temporarily worse. This is known as a negative test. An improvement of weakness would occur if the client were experiencing myasthenia gravis. Options 1 and 2 would not occur in either crisis.

181.) A client is taking ticlopidine hydrochloride (Ticlid). The nurse tells the client to avoid which of the following while taking this medication? 1. Vitamin C 2. Vitamin D 3. Acetaminophen (Tylenol) 4. Acetylsalicylic acid (aspirin)

4. Acetylsalicylic acid (aspirin) Rationale: Ticlopidine hydrochloride is a platelet aggregation inhibitor. It is used to decrease the risk of thrombotic strokes in clients with precursor symptoms. Because it is an antiplatelet agent, other medications that precipitate or aggravate bleeding should be avoided during its use. Therefore, aspirin or any aspirin-containing product should be avoided.

242.) A client who was started on anticonvulsant therapy with clonazepam (Klonopin) tells the nurse of increasing clumsiness and unsteadiness since starting the medication. The client is visibly upset by these manifestations and asks the nurse what to do. The nurse's response is based on the understanding that these symptoms: 1. Usually occur if the client takes the medication with food 2. Are probably the result of an interaction with another medication 3. Indicate that the client is experiencing a severe untoward reaction to the medication 4. Are worse during initial therapy and decrease or disappear with long-term use

4. Are worse during initial therapy and decrease or disappear with long-term use Rationale: Drowsiness, unsteadiness, and clumsiness are expected effects of the medication during early therapy. They are dose related and usually diminish or disappear altogether with continued use of the medication. It does not indicate that a severe side effect is occurring. It is also unrelated to interaction with another medication. The client is encouraged to take this medication with food to minimize gastrointestinal upset. **Eliminate options 2 and 3 first because they are comparable or alike and because of the word "severe" in option 3**

4.) The camp nurse asks the children preparing to swim in the lake if they have applied sunscreen. The nurse reminds the children that chemical sunscreens are most effective when applied: 1. Immediately before swimming 2. 15 minutes before exposure to the sun 3. Immediately before exposure to the sun 4. At least 30 minutes before exposure to the sun

4. At least 30 minutes before exposure to the sun Rationale: Sunscreens are most effective when applied at least 30 minutes before exposure to the sun so that they can penetrate the skin. All sunscreens should be reapplied after swimming or sweating.

178.) Methylergonovine (Methergine) is prescribed for a client with postpartum hemorrhage caused by uterine atony. Before administering the medication, the nurse checks which of the following as the important client parameter? 1. Temperature 2. Lochial flow 3. Urine output 4. Blood pressure

4. Blood pressure Rationale: Methylergonovine is an ergot alkaloid used for postpartum hemorrhage. It stimulates contraction of the uterus and causes arterial vasoconstriction. Ergot alkaloids are avoided in clients with significant cardiovascular disease, peripheral disease, hypertension, eclampsia, or preeclampsia. These conditions are worsened by the vasoconstrictive effects of the ergot alkaloids. The nurse would check the client's blood pressure before administering the medication and would follow agency protocols regarding withholding of the medication. Options 1, 2, and 3 are items that are checked in the postpartum period, but they are unrelated to the use of this medication.

94.) The client with acquired immunodeficiency syndrome has begun therapy with zidovudine (Retrovir, Azidothymidine, AZT, ZDV). The nurse carefully monitors which of the following laboratory results during treatment with this medication? 1. Blood culture 2. Blood glucose level 3. Blood urea nitrogen 4. Complete blood count

4. Complete blood count Rationale: A common side effect of therapy with zidovudine is leukopenia and anemia. The nurse monitors the complete blood count results for these changes. Options 1, 2, and 3 are unrelated to the use of this medication.

155.) Mycophenolate mofetil (CellCept) is prescribed for a client as prophylaxis for organ rejection following an allogeneic renal transplant. Which of the following instructions does the nurse reinforce regarding administration of this medication? 1. Administer following meals. 2. Take the medication with a magnesium-type antacid. 3. Open the capsule and mix with food for administration. 4. Contact the health care provider (HCP) if a sore throat occurs.

4. Contact the health care provider (HCP) if a sore throat occurs. Rationale: Mycophenolate mofetil should be administered on an empty stomach. The capsules should not be opened or crushed. The client should contact the HCP if unusual bleeding or bruising, sore throat, mouth sores, abdominal pain, or fever occurs because these are adverse effects of the medication. Antacids containing magnesium and aluminum may decrease the absorption of the medication and therefore should not be taken with the medication. The medication may be given in combination with corticosteroids and cyclosporine. **neutropenia can occur with this medication**

157.) A client receiving nitrofurantoin (Macrodantin) calls the health care provider's office complaining of side effects related to the medication. Which side effect indicates the need to stop treatment with this medication? 1. Nausea 2. Diarrhea 3. Anorexia 4. Cough and chest pain

4. Cough and chest pain Rationale: Gastrointestinal (GI) effects are the most frequent adverse reactions to this medication and can be minimized by administering the medication with milk or meals. Pulmonary reactions, manifested as dyspnea, chest pain, chills, fever, cough, and the presence of alveolar infiltrates on the x-ray, would indicate the need to stop the treatment. These symptoms resolve in 2 to 4 days following discontinuation of this medication. **Eliminate options 1, 2, and 3 because they are similar GI-related side effects. Also, use the ABCs— airway, breathing, and circulation**

190.) A child is hospitalized with a diagnosis of lead poisoning. The nurse assisting in caring for the child would prepare to assist in administering which of the following medications? 1. Activated charcoal 2. Sodium bicarbonate 3. Syrup of ipecac syrup 4. Dimercaprol (BAL in Oil)

4. Dimercaprol (BAL in Oil) Rationale: Dimercaprol is a chelating agent that is administered to remove lead from the circulating blood and from some tissues and organs for excretion in the urine. Sodium bicarbonate may be used in salicylate poisoning. Syrup of ipecac is used in the hospital setting in poisonings to induce vomiting. Activated charcoal is used to decrease absorption in certain poisoning situations. Note that dimercaprol is prepared with peanut oil, and hence should be avoided by clients with known or suspected peanut allergy.

223.) A client with a psychotic disorder is being treated with haloperidol (Haldol). Which of the following would indicate the presence of a toxic effect of this medication? 1. Nausea 2. Hypotension 3. Blurred vision 4. Excessive salivation

4. Excessive salivation Rationale: Toxic effects include extrapyramidal symptoms (EPS) noted as marked drowsiness and lethargy, excessive salivation, and a fixed stare. Akathisia, acute dystonias, and tardive dyskinesia are also signs of toxicity. Hypotension, nausea, and blurred vision are occasional side effects.

231.) A client admitted to the hospital gives the nurse a bottle of clomipramine (Anafranil). The nurse notes that the medication has not been taken by the client in 2 months. What behaviors observed in the client would validate noncompliance with this medication? 1. Complaints of hunger 2. Complaints of insomnia 3. A pulse rate less than 60 beats per minute 4. Frequent handwashing with hot, soapy water

4. Frequent handwashing with hot, soapy water Rationale: Clomipramine is commonly used in the treatment of obsessive-compulsive disorder. Handwashing is a common obsessive-compulsive behavior. Weight gain is a common side effect of this medication. Tachycardia and sedation are side effects. Insomnia may occur but is seldom a side effect.

236.) A client is being treated for depression with amitriptyline hydrochloride. During the initial phases of treatment, the most important nursing intervention is: 1. Prescribing the client a tyramine-free diet 2. Checking the client for anticholinergic effects 3. Monitoring blood levels frequently because there is a narrow range between therapeutic and toxic blood levels of this medication 4. Getting baseline postural blood pressures before administering the medication and each time the medication is administered

4. Getting baseline postural blood pressures before administering the medication and each time the medication is administered Rationale: Amitriptyline hydrochloride is a tricyclic antidepressant often used to treat depression. It causes orthostatic changes and can produce hypotension and tachycardia. This can be frightening to the client and dangerous because it can result in dizziness and client falls. The client must be instructed to move slowly from a lying to a sitting to a standing position to avoid injury if these effects are experienced. The client may also experience sedation, dry mouth, constipation, blurred vision, and other anticholinergic effects, but these are transient and will diminish with time.

175.) A nurse notes that a client is receiving lamivudine (Epivir). The nurse determines that this medication has been prescribed to treat which of the following? 1. Pancreatitis 2. Pharyngitis 3. Tonic-clonic seizures 4. Human immunodeficiency virus (HIV) infection

4. Human immunodeficiency virus (HIV) infection Rationale: Lamivudine is a nucleoside reverse transcriptase inhibitor and antiviral medication. It slows HIV replication and reduces the progression of HIV infection. It also is used to treat chronic hepatitis B and is used for prophylaxis in health care workers at risk of acquiring HIV after occupational exposure to the virus. **Note the letters "-vir" in the trade name for this medication**

73.) A client with myasthenia gravis is suspected of having cholinergic crisis. Which of the following indicate that this crisis exists? 1. Ataxia 2. Mouth sores 3. Hypotension 4. Hypertension

4. Hypertension Rationale: Cholinergic crisis occurs as a result of an overdose of medication. Indications of cholinergic crisis include gastrointestinal disturbances, nausea, vomiting, diarrhea, abdominal cramps, increased salivation and tearing, miosis, hypertension, sweating, and increased bronchial secretions.

200.) A client is seen in the clinic for complaints of skin itchiness that has been persistent over the past several weeks. Following data collection, it has been determined that the client has scabies. Lindane is prescribed, and the nurse is asked to provide instructions to the client regarding the use of the medication. The nurse tells the client to: 1. Apply a thick layer of cream to the entire body. 2. Apply the cream as prescribed for 2 days in a row. 3. Apply to the entire body and scalp, excluding the face. 4. Leave the cream on for 8 to 12 hours and then remove by washing.

4. Leave the cream on for 8 to 12 hours and then remove by washing. Rationale: Lindane is applied in a thin layer to the entire body below the head. No more than 30 g (1 oz) should be used. The medication is removed by washing 8 to 12 hours later. Usually, only one application is required.

36.) The client has a PRN prescription for ondansetron (Zofran). For which condition should this medication be administered to the postoperative client? 1. Paralytic ileus 2. Incisional pain 3. Urinary retention 4. Nausea and vomiting

4. Nausea and vomiting Rationale: Ondansetron is an antiemetic used to treat postoperative nausea and vomiting, as well as nausea and vomiting associated with chemotherapy. The other options are incorrect.

44.) A client is receiving acetylcysteine (Mucomyst), 20% solution diluted in 0.9% normal saline by nebulizer. The nurse should have which item available for possible use after giving this medication? 1. Ambu bag 2. Intubation tray 3. Nasogastric tube 4. Suction equipment

4. Suction equipment Rationale: Acetylcysteine can be given orally or by nasogastric tube to treat acetaminophen overdose, or it may be given by inhalation for use as a mucolytic. The nurse administering this medication as a mucolytic should have suction equipment available in case the client cannot manage to clear the increased volume of liquefied secretions.

67.) Phenazopyridine hydrochloride (Pyridium) is prescribed for a client for symptomatic relief of pain resulting from a lower urinary tract infection. The nurse reinforces to the client: 1. To take the medication at bedtime 2. To take the medication before meals 3. To discontinue the medication if a headache occurs 4. That a reddish orange discoloration of the urine may occur

4. That a reddish orange discoloration of the urine may occur Rationale: The nurse should instruct the client that a reddish-orange discoloration of urine may occur. The nurse also should instruct the client that this discoloration can stain fabric. The medication should be taken after meals to reduce the possibility of gastrointestinal upset. A headache is an occasional side effect of the medication and does not warrant discontinuation of the medication.

232.) A client in the mental health unit is administered haloperidol (Haldol). The nurse would check which of the following to determine medication effectiveness? 1. The client's vital signs 2. The client's nutritional intake 3. The physical safety of other unit clients 4. The client's orientation and delusional status

4. The client's orientation and delusional status Rationale: Haloperidol is used to treat clients exhibiting psychotic features. Therefore, to determine medication effectiveness, the nurse would check the client's orientation and delusional status. Vital signs are routine and not specific to this situation. The physical safety of other clients is not a direct assessment of this client. Monitoring nutritional intake is not related to this situation.

160.) Meperidine hydrochloride (Demerol) is prescribed for the client with pain. Which of the following would the nurse monitor for as a side effect of this medication? 1. Diarrhea 2. Bradycardia 3. Hypertension 4. Urinary retention

4. Urinary retention Rationale: Meperidine hydrochloride (Demerol) is an opioid analgesic. Side effects of this medication include respiratory depression, orthostatic hypotension, tachycardia, drowsiness and mental clouding, constipation, and urinary retention.

104.) Disulfiram (Antabuse) is prescribed for a client who is seen in the psychiatric health care clinic. The nurse is collecting data on the client and is providing instructions regarding the use of this medication. Which is most important for the nurse to determine before administration of this medication? 1. A history of hyperthyroidism 2. A history of diabetes insipidus 3. When the last full meal was consumed 4. When the last alcoholic drink was consumed

4. When the last alcoholic drink was consumed Rationale: Disulfiram is used as an adjunct treatment for selected clients with chronic alcoholism who want to remain in a state of enforced sobriety. Clients must abstain from alcohol intake for at least 12 hours before the initial dose of the medication is administered. The most important data are to determine when the last alcoholic drink was consumed. The medication is used with caution in clients with diabetes mellitus, hypothyroidism, epilepsy, cerebral damage, nephritis, and hepatic disease. It is also contraindicated in severe heart disease, psychosis, or hypersensitivity related to the medication.

180.) A health care provider (HCP) writes a prescription for digoxin (Lanoxin), 0.25 mg daily. The nurse teaches the client about the medication and tells the client that it is important to: 1. Count the radial and carotid pulses every morning. 2. Check the blood pressure every morning and evening. 3. Stop taking the medication if the pulse is higher than 100 beats per minute. 4. Withhold the medication and call the HCP if the pulse is less than 60 beats per minute.

4. Withhold the medication and call the HCP if the pulse is less than 60 beats per minute. Rationale: An important component of taking this medication is monitoring the pulse rate; however, it is not necessary for the client to take both the radial and carotid pulses. It is not necessary for the client to check the blood pressure every morning and evening because the medication does not directly affect blood pressure. It is most important for the client to know the guidelines related to withholding the medication and calling the HCP. The client should not stop taking a medication.

B. Take and record radial pulse rate daily. ***Monitoring pulse rate is very important when taking digoxin (Lanoxin) (B). The client should be further instructed to report pulse rates below 60 or greater than 110 beats/min and to withhold the dosage until consulting with the health care provider in such a case. (A and D) are not necessary. (C) is an indication of drug toxicity, and the client should be instructed to report this immediately.

A 67-year-old client is discharged from the hospital with a prescription for digoxin (Lanoxin), 0.25 mg daily. Which instruction should the nurse include in this client's discharge teaching plan? A. Take the medication in the morning before rising. B. Take and record radial pulse rate daily. C. Expect some vision changes caused by the medication. D. Increase intake of foods rich in vitamin K.

A. Hold the theophylline dose and notify the health care provider. ***The therapeutic range for theophylline is 10 to 20 mcg/mL, so the theophylline dose should be held for fear of causing toxicity (A). (B, C, and D) are not indicated actions based on the reported theophylline level.

A client receives a prescription for theophylline (Theo-Dur) PO to be initiated in the morning after the dose of theophylline IV is complete. The nurse determines that a theophylline level drawn yesterday was 22 mcg/mL. Based on this information, which action should the nurse implement? A. Hold the theophylline dose and notify the health care provider. B. Start the client on a half-dose of theophylline PO. C. The theophylline dose can be initiated as planned. D. The client is not ready to be weaned from the IV to the PO route.

B. Obtain the client's blood pressure. ***To determine the most accurate response to antihypertensive therapy, baseline blood pressures should be obtained before an antihypertensive drug is administered and should be compared with orthostatic vital signs to determine whether any side effects are occurring (B). Although (A, C, and D) are required nursing actions prior to giving any drug, the therapeutic response should be determined before another dose is administered.

A client receives an antihypertensive agent daily. Which action is most important for the nurse to implement prior to administering the medication? A. Verify the expiration date. B. Obtain the client's blood pressure. C. Determine the client's history of adverse reactions. D. Review the client's medical record for a change in drug route.

C. Instruct the client to obtain a stool specimen to be taken to the laboratory for analysis. ***Antibiotics, such as Zyvox, can cause pseudomembranous colitis, resulting in severe watery diarrhea. The prescriber should be notified, and a stool specimen (C) should be obtained and analyzed for this complication. Severe diarrhea is not an indication of drug toxicity, so (A) is not warranted. Although gastrointestinal disturbance can be an adverse effect of Zyvox (B), a stool specimen should be obtained because the client reports the diarrhea is severe. Antidiarrheal medications (D) are contraindicated in the presence of this colitis and should not be started until this potential complication is ruled out.

A client taking linezolid (Zyvox) at home for an infected foot ulcer calls the home care nurse to report the onset of watery diarrhea. Which intervention should the nurse implement? A. Schedule appointments to obtain blood samples for drug peak and trough levels. B. Reassure the client that this is an expected side effect that will resolve in a few days. C. Instruct the client to obtain a stool specimen to be taken to the laboratory for analysis. D. Advise the client to begin taking an over-the-counter antidiarrheal agent.

A. Fatigue and muscle weakness ***Thiazide diuretics, such as HCTZ, cause potassium wasting in the urine, so the client should be instructed to report fatigue and muscle weakness (A), which are characteristic of hypokalemia. Although (B, C, and D) should be reported, they are not indicative of hypokalemia, which is a side effect of HCTZ that can cause cardiac dysrhythmias.

A client who is hypertensive receives a prescription for hydrochlorothiazide (HCTZ). When teaching about the side effects of this drug, which symptoms are most important for the nurse to instruct the client to report? A. Fatigue and muscle weakness B. Anxiety and heart palpitations C. Abdominal cramping and diarrhea D. Confusion and personality changes

B. Multiple drugs prevent the development of resistant organisms. ***A multidrug regimen is prescribed for a client with HIV and TB to prevent the development of resistance of the tubercle bacilli (B). Although antitubercular medications can inhibit some antiretrovirals (A), a multidrug regimen is needed to inhibit the proliferation of the virulent tubercle bacilli. The duration of antitubercular therapy is typically 6 to 9 months and is not shortened (C) by the use of multiple medications. A client who is receiving HIV and TB therapy is at an increased risk of adverse reactions (D) because of the complex medication regimens and complications secondary to immunosuppression.

A client with HIV who was recently diagnosed with tuberculosis (TB) asks the nurse, "Why do I need to take all of these medications for TB?" What information should the nurse provide? A. Antiretroviral medications decrease the efficacy of the TB drugs. B. Multiple drugs prevent the development of resistant organisms. C. Duration of the medication regimen is shortened. D. Potential adverse drug reactions are minimized.

A. Oral contraceptives may not be effective. ***Certain antibiotics, such as tetracycline (Vibramycin), decrease the effectiveness of oral contraceptives (A). (B, C, and D) do not convey accurate information related to client teaching about this medication.

A female client is receiving tetracycline (Vibramycin) for acne. Which client teaching should the nurse include? A. Oral contraceptives may not be effective. B. Drinking cranberry juice will promote healing. C. Breast tenderness may occur as a side effect. D. The urine will turn a red-orange color.

A. Advise the client to take the medication in the morning, rather than at bedtime. ***Daily doses of long-term corticosteroid therapy should be administered in the morning (A) to coincide with the body's normal secretion of cortisol. Clients receiving long-term corticosteroids need to increase their intake of calcium, which generally means an increase in dairy products (B). Corticosteroids can often cause gastrointestinal distress and should be administered with meals (C). The client has established a safe routine by taking the medication with a snack, but the routine will be more effective if done in the morning (D).

A female client who has started long-term corticosteroid therapy tells the nurse that she is careful to take her daily dose at bedtime with a snack of crackers and milk. Which is the best response by the nurse? A. Advise the client to take the medication in the morning, rather than at bedtime. B. Teach the client that dairy products should not be taken with her medication. C. Tell the client that absorption is improved when taken on an empty stomach. D. Affirm that the client has a safe and effective routine for taking the medication.

D. Develop a teaching plan for the client to self- adjust the dose of medication in response to symptoms ***Maintaining optimal dosage for cholinesterase inhibitors can be challenging for clients with myasthenia gravis. Clients should be taught to recognize signs of overmedication and undermedication so that they can modify the dosage themselves (D) based on a prescribed sliding scale. (A, B, and C) do not adequately address the client's concerns.

A female client with myasthenia gravis is taking a cholinesterase inhibitor and asks the nurse what can be done to remedy her fatigue and difficulty swallowing. What action should the nurse implement? A. Explore a plan for development of coping strategies for the symptoms with the client. B. Explain to the client that the dosage is too high, so she should skip every other dose of medication. C. Advise the client to contact her health care provider because of the development of tolerance to the medication. D. Develop a teaching plan for the client to self- adjust the dose of medication in response to symptoms

A. Avoid alcohol consumption. ***Clients should be instructed to avoid alcohol and products containing alcohol (A) while taking metronidazole (Flagyl) because of the possibility of a disulfiram (Antabuse)-like reaction. (B) helps prevent the development of Flagyl-resistant T. vaginalis. To prevent reinfection, clients should abstain from sexual contact or use a barrier contraceptive (C) while taking Flagyl, and their partner(s) should be treated concurrently (D). The most important instruction for client well-being is (A).

A female client with trichomoniasis (Trichomonas vaginalis) receives a prescription for metronidazole (Flagyl). Which instruction is most important for the nurse to include this client's teaching plan? A. Avoid alcohol consumption. B. Complete the medication regimen. C. Use a barrier contraceptive method. D. Treat partner(s) concurrently.

C. Pharmacodynamic tolerance requires increased drug levels to achieve the same effect. ***Pharmacodynamic tolerance explains the client's need for an increased drug level to produce effects that formerly occurred at lower drug levels (C). Tolerance can occur with opioids (A) during shorter periods of use. Although a withdrawal syndrome can occur if the client develops a dependency (B), this does not address the client's immediate concern of drug effectiveness. Although a stable serum drug level provides effective pain management (D), the client's complaint is consistent with a tolerance to his current pain management regimen.

A male client who has chronic back pain is on long-term pain medication management and asks the nurse why his pain relief therapy is not as effective as it was 2 months ago. How should the nurse respond? A. The phenomenon occurs when opiates are used for more than 6 months to relieve pain. B. Withdrawal occurs if the drug is not tapered slowly while being discontinued. C. Pharmacodynamic tolerance requires increased drug levels to achieve the same effect. D. A consistent dosage with around-the-clock administration is the most effective.

D. There is no safety evidence of this medication during pregnancy, so it should be avoided ***When working with a pregnant client, the nurse must be familiar with those situations and medications that can be harmful to the pregnant mother and/or her unborn baby. An example is corticosteroid use, which has not been shown to be safe during pregnancy. The nurse should be aware of this and counsel the client against using this type of drug.

A nurse is caring for a pregnant patient who needs treatment for rosacea. The patient asks the nurse about using topical corticosteroids for treatment. Which of the following information should the nurse provide this patient? A. The patient can safely use this type of medication B. The patient can only use this medication in areas away from the abdomen C. This medication causes teratogenic effects and should be avoided D. There is no safety evidence of this medication during pregnancy, so it should be avoided

A. Sucralfate (Carafate) ***Sucralfate (Carafate) (A) is used to treat duodenal ulcers and will bind with tetracycline hydrochloride (Sumycin), inhibiting this antibiotic's absorption. (B, C, and D) have no drug interaction properties that prohibit concurrent use with tetracycline hydrochloride (Sumycin).

During the initial nursing assessment history, a client tells the nurse that he is taking tetracycline hydrochloride (Sumycin) for urethritis. Which medication taken concurrently with Sumycin could interfere with its absorption? A. Sucralfate (Carafate) B. Hydrochlorothiazide (Diuril) C. Acetaminophen (Tylenol) D. Phenytoin (Dilantin)

This system activates the production of hormones and the release of epinephrine and norepinephrine

Hypothalamic-pituitary-adrenal system (HPA)

If an adrenergic drug is taken in conjunction with an antidepressant, what may occur as a result?

Increased sympathomimetic effect

Adrenergic drugs produce what type of effects on metabolism?

Increased use of glucose and liberation of fatty acids from adipose tissue

A. Report vaginal itching or discharge. C. Protect skin from natural and artificial ultraviolet light. D. Avoid driving until response to medication is known. F. Use a nonhormonal method of contraception if sexually active. ***Correct selections are (A, C, D, and F). Adverse effects of tetracyclines include superinfections, photosensitivity, and decreased efficacy of oral contraceptives. Therefore, the client should report vaginal itching or discharge (A), protect the skin from ultraviolet light (C), and use a nonhormonal method of contraception (F) while on the medication. Minocycline (Minocin) is known to cause dizziness and ataxia, so until the client's response to the medication is known, driving (D) should be avoided. Tetracyclines should be taken around the clock (B) but exhibit decreased absorption when taken with antacids, so (E) is contraindicated.

Minocycline (Minocin), 50 mg PO every 8 hours, is prescribed for an adolescent girl diagnosed with acne. The nurse discusses self-care with the client while she is taking the medication. Which teaching points should be included in the discussion? (Select all that apply.) A. Report vaginal itching or discharge. B. Take the medication at 0800, 1500, and 2200 hours. C. Protect skin from natural and artificial ultraviolet light. D. Avoid driving until response to medication is known. E. Take with an antacid tablet to prevent nausea. F. Use a nonhormonal method of contraception if sexually active.

A female client with multiple sclerosis reports having less fatigue and improved memory since she began using the herbal supplement, ginkgo biloba. Which information is most important for the nurse to include in the teaching plans for this client? a-Aspirin and nonsteroidal anti-inflammatory drugs interact with ginkgo b-nausea and diarrhea can occur when using this supplement c-anxiety and headaches increased with use of ginkgo d-ginkgo biloba use should be limited and not taken during pregnancy

Rationale ginkgo biloba has blood thinning properties and should not be used in taking aspirin or unsafe which increased the risk for bleeding. nausea diarrhea anxiety and headaches are also side effects of supplements but they do not pose the same risk as a. Although D is accurate A has a higher priority

Client being treated with Haldol for schizophrenia is complaining of jaw tightness & a stiff neck. Which interventions should the nurse impliment? -give PRN those of diphenhydramine Benadryl -assess client other sensory hallucinations -massage neck until muscles begin to relax -obtain a 12 lead EKG

Rationale jaw tightness and a stiff neck are signs of tardive dyskinesia, a serious side effect of health all that can be reversed with Benadryl answer choice A. the client complaints are not considered hallucinations b. Massaging the neck does not eliminate the symptoms. A 12 lead EKG is not indicated because the pain is not cardiac in origin.

Adrenergic drugs produce what type of effects in the autonomic nervous system?

Relaxation of smooth muscles of the bronchi Constriction of blood vessels Dilation of coronary blood vessels Decrease in GI motility

C. 75-year-old woman with liver disease ***Impaired hepatic metabolic pathways for drug and chemical degradation place (C) at greatest risk for adverse reactions to medications based on advancing age and liver disease. (A and D) have no predisposing factors, such as genetics, pathophysiologic dysfunction, or drug allergies, that would increase the risk for cumulative toxicity or adverse drug reactions. (B) is at risk for dose-related adverse reactions but is at less risk than (C).

The charge nurse is reviewing the admission history and physical data for four clients newly admitted to the unit. Which client is at greatest risk for adverse reactions to medications? A. 30-year-old man with a fracture B. 7-year-old child with an ear infection C. 75-year-old woman with liver disease D. 50-year-old man with an upper respiratory tract infection

C. Myelosuppression ***Myelosuppression (C) is the highest priority complication that can potentially affect clients managed with carbamazepine (Tegretol) therapy. The client requires close monitoring for this condition by weekly laboratory testing. Hepatic function may be altered (D), but this complication does not have as great a potential for occurrence as (C). (A and B) are not typical complications of carbamazepine (Tegretol) therapy.

The health care provider prescribes carbamazepine (Tegretol) for a child whose tonic-clonic seizures have been poorly controlled. The nurse informs the mother that the child must have blood tests every week. The mother asks why so many blood tests are necessary. Which complication is assessed through frequent laboratory testing that the nurse should explain to this mother? A. Nephrotoxicity B. Ototoxicity C. Myelosuppression D. Hepatotoxicity

C. Urine output ***The effectiveness of the diuresis is best measured by urine output (C). Mannitol, an osmotic diuretic, is given during cisplatin (Platinol) therapy to promote diuresis and reduce the risk of nephrotoxicity and ototoxicity associated with this chemotherapeutic agent. (A, B, and D) do not provide information about the risk for nephrotoxicity and ototoxicity related to Platinol administration.

The health care provider prescribes cisplatin (Platinol) to be administered in 5% dextrose and 0.45% normal saline with mannitol (Osmitrol) added. Which assessment parameters would be most helpful to the nurse in evaluating the effectiveness of the Osmitrol therapy? A. Oral temperature B. Blood cultures C. Urine output D. Liver enzyme levels

A. Naloxone hydrochloride (naracan) ***Naloxone is an opioid antidote used in opioid overdose (A) to reverse CNS and respiratory depression. Atropine (B) is used for bradycardia, intestinal hypertonicity and hypermotility, muscarinic agonist poisoning, peptic ulcer disease, and biliary colic. Vitamin K (C) is used to manage warfarin overdose and vitamin K deficiency in newborns. Flumazenil (D) reduces the sedative effects of benzodiazepines following general anesthesia or overdose.

The nurse is assessing a stuporuous client in the emergency department who is suspected of overdosing with opioids. Which agent should the nurse prepare to administer if the client becomes comatose? A. Naloxone hydrochloride (naracan) B. Atropine Sulfate C. Vitamin K D. Romazicon

A. Decrease the oral secretions. ***Atropine sulfate (Atropine), an anticholinergic agent, is given to decrease oral secretions during a surgical procedure (A). (B, C, and D) are not actions of anticholinergic agents.

The nurse is preparing a child for transport to the operating room for an emergency appendectomy. The anesthesiologist prescribes atropine sulfate (Atropine), IM STAT. What is the primary purpose for administering this drug to the child at this time? A. Decrease the oral secretions. B. Reduce the child's anxiety C. Potentiate the opioid effects D. Prevent possible peritonitis

B. Risk for infection ***Corticosteroids depress the immune system, placing the client at risk for infection (B). Although (A, C, and D) reflect diagnostic statements that may be applicable to this client, only (B) is directly related to the administration of this medication.

The nurse is preparing a plan of care for a client receiving the glucocorticoid methylprednisolone (Solu-Medrol). Which nursing diagnosis reflects a problem related to this medication that should be included in the care plan? A. Ineffective airway clearance B. Risk for infection C. Deficient fluid volume D. Impaired gas exchange

D. "I will take the medication every day before breakfast." ***The enzyme that helps metabolize cholesterol is activated at night, so this medication should be taken with the evening meal (D). (A, B, and C) reflect correct information about lovastatin.

The nurse is evaluating a client's understanding of the prescribed antilipemic drug lovastatin (Mevacor). Which client statement indicates that further teaching is needed? A. "My bowel habits should not be affected by this drug." B. "This medication should be taken once a day only." C. "I will still need to follow a low-cholesterol diet." D. "I will take the medication every day before breakfast."

A. Avoid ingesting any alcohol or acetaminophen (Tylenol). ***Combining hepatotoxic drugs, such as acetaminophen and alcohol, increases the risk of liver damage, so (A) is an important discharge instruction. Although clients who receive hepatotoxic drugs should be screened for any changes in serum liver function test (LFT) results, (B) is not indicated. Rest is advantageous during an infectious process, but activity restriction (C) is unnecessary. A client who is receiving a hepatotoxic drug should report any hepatotoxic symptoms, such as jaundice, dark urine, or light-colored stools, but an increased appetite (D) does not need medical attention.

The nurse is providing discharge instructions to a client who has received a prescription for an antibiotic that is hepatotoxic. Which information should the nurse include in the instructions? A. Avoid ingesting any alcohol or acetaminophen (Tylenol). B. Schedule a follow-up visit for a liver biopsy in 1 month. C. Activities that are strenuous should be avoided. D. Notify the health care provider of any increase in appetite.

A. Hypokalemia ***Low potassium levels enhance the effects of neuromuscular blocking agents, so the health care provider should be informed of the client's hypokalemia (A). (B, C, and D) are of concern but do not enhance the effects of neuromuscular blocking agents.

The nurse is reviewing a client's laboratory results before a procedure in which a neuromuscular blocking agent is a standing order. Which finding should the nurse report to the health care provider? A. Hypokalemia B. Hyponatremia C. Hypercalcemia D. Hypomagnesemia

D. Oral forms of drugs must pass through the liver first, where more of the dose is metabolized. ***Oral doses of medication are usually larger than parenteral doses to compensate for the first-pass effect in the liver after oral administration (D), which metabolizes more of the drug's dose before affecting its therapeutic response. Although recommended dose ranges for adults should be individualized, a client's pain should be controlled at discharge, not (A or C). (B) is inaccurate information to convey to the client.

The nurse is reviewing prescribed medications with a female client who is preparing for discharge. The client asks the nurse why the oral dose of an opioid analgesic is higher than the IV dose that she received during hospitalization. Which response is best for the nurse to provide? A. A higher dose of analgesic medication may be needed after discharge. B. An error in the dose calculation may have occurred when the prescribed dose was converted. C. The doses should be the same unless the pain is not well controlled. D. Oral forms of drugs must pass through the liver first, where more of the dose is metabolized.

C. Instruct the laboratory to draw the trough immediately before the next scheduled dose. ***The best time to draw a trough is the closest time to the next administration (C). (A) will provide a peak level. (B) will not provide the most accurate trough level. The medication is given before peak and trough levels are obtained (D).

The nurse is scheduling a client's antibiotic peak and trough levels with the laboratory personnel. What is the best schedule for drawing the trough level? A. Give the dose of medication, and call the laboratory to draw the trough STAT. B. Arrange for the laboratory to draw the trough 1 hour after the dose is given. C. Instruct the laboratory to draw the trough immediately before the next scheduled dose. D. Give the first dose of medication after the laboratory reports that the trough has been drawn.

A. withdraws the NPH insulin first

The nurse is teaching a client how to mix regular insulin and NPH insulin in the same syringe. Which action, if performed by the client, indicates the need for further teaching? A. withdraws the NPH insulin first B. withdraws the regular insulin first C. injects air into NPH insulin vial first D. injects an amount of air equal to the desired dose of insulin into each vial

C. Low serum potassium level ***Hypokalemia (C) predisposes the client on digoxin to digoxin toxicity, which usually presents as abdominal pain, anorexia, nausea, vomiting, visual disturbances, bradycardia, and atrioventricular (AV) dissociation. Assessment of serum potassium levels with prompt correction of hypokalemia are important interventions for the client taking digoxin. (A, B, and D) are not relevant.

When caring for a client on digoxin (Lanoxin) therapy, the nurse knows to be alert for digoxin (Lanoxin) toxicity. Which finding would predispose this client to developing digoxin toxicity? A. Low serum sodium level B. High serum sodium level C. Low serum potassium level D. High serum potassium level

B. Assess respiratory status and breath sounds often. ***The client should be assessed often for signs of respiratory complications (B). The client with myasthenia gravis is at greatest risk for life-threatening respiratory complications because of the weakness of the diaphragm and ancillary respiratory muscles caused by the disease process. Cholinergic agents used to reduce muscle weakness can also cause hypersalivation, increased respiratory secretions, and possible bronchoconstriction. Although (A, C, and D) reflect helpful interventions, they do not have the priority of (B) in caring for the client with myasthenia gravis.

When providing nursing care for a client receiving pyridostigmine bromide (Mestinon) for myasthenia gravis, which nursing intervention has the highest priority? A. Monitor the client frequently for urinary retention. B. Assess respiratory status and breath sounds often. C. Monitor blood pressure each shift to screen for hypertension. D. Administer most medications after meals to decrease gastrointestinal irritation.

C. Client education ***The client's educational level (C) is the most important factor when planning teaching to ensure a client's compliance with taking a prescribed drug. (A and D) are physiologic responses that do not relate to a client's compliance. Although maturity level and age (B) contribute to compliance, the client's basic understanding of instructions, which is best indicated by educational level, is more significant.

Which factor is most important to ensure compliance when planning to teach a client about a drug regimen? A. Genetics B. Client age C. Client education D. Absorption rate

E. All of the above

Which of the following agents would increase sedation caused by morphine? A. ethanol B. diazepam C. chlorpromazine D. clomipramine E. All of the above

D. Decreased urine output ***Lasix is a diuretic medication that can be given to induce elimination of excess fluid from the body. Lasix is typically used when a patient has excess fluid because of such diseases as heart failure or when pulmonary edema is present. Lasix should not be used when a patient has decreased urine output as a method to get the patient to urinate.

Which of the following is considered a contraindication for administration of Furosemide [Lasix®]? A. 4+ pitting edema in the lower extremities B. Hypertension C. Facial swelling D. Decreased urine output

C. Activated partial thromboplastin time (aPTT) ***The laboratory value that measures heparin's therapeutic anticoagulation time is the aPTT (C). (A) should be checked before the administration of digoxin. (B) is valuable information but not a parameter measured for heparin therapy. (D) is evaluated during anticoagulation therapy using sodium warfarin (Coumadin).

Which parameter is most important for the nurse to check prior to administering a subcutaneous injection of heparin? A. Heart rate B. Urinary output C. Activated partial thromboplastin time (aPTT) D. Prothrombin time (PT) and international normalized ratio (INR)

A. "Are you having difficulty hearing?" ***Complications of gentamicin sulfate (Garamycin) therapy include ototoxicity, nephrotoxicity, and neurotoxicity. Determining if the client is hard of hearing (A) prior to initiation of this aminoglycoside will be helpful as the treatment progresses and ototoxicity is identified as a possible complication. Information obtained in (B, C, and D) are important elements of any medical history, but they do not have the priority of (A) when assessing for complications of aminoglycoside therapy.

Which question should the nurse ask a client prior to the initiation of treatment with IV infusions of gentamicin sulfate (Garamycin)? A. "Are you having difficulty hearing?" B. "Have you ever been diagnosed with cancer?" C. "Do you have any type of diabetes mellitus?" D. "Have you ever had anemia?"

208.) A client with myasthenia gravis verbalizes complaints of feeling much weaker than normal. The health care provider plans to implement a diagnostic test to determine if the client is experiencing a myasthenic crisis and administers edrophonium (Enlon). Which of the following would indicate that the client is experiencing a myasthenic crisis? 1. Increasing weakness 2. No change in the condition 3. An increase in muscle spasms 4. A temporary improvement in the condition

auto-define "A client with myasthen..." Rationale: Edrophonium (Enlon) is administered to determine whether the client is reacting to an overdose of a medication (cholinergic crisis) or to an increasing severity of the disease (myasthenic crisis). When the edrophonium (Enlon) injection is given and the condition improves temporarily, the client is in myasthenic crisis. This is known as a positive test. Increasing weakness would occur in cholinergic crisis. Options 2 and 3 would not occur in either crisis.

This adrenergic receptor works on the peripheral blood vessels and bronchial smooth muscles. It causes vasodilation of peripheral vessels and causes bonchodilation

b2 (2 lungs)

Examples of ACE inhibitors (antihypertensives)

benazepril, captopril, moexipril, quinapril

Examples of Potassium-losing diuretics

benzthiazide, hydrochlorothiazide

Examples of Local anesthetics

bupinvacaine, cocaine, ifocaine, xylocaine

Examples of Antifungals

butoconazole, econazole, fluconazole

Examples of Antiparasitics

chloroquine, hydroxychloroquine, mfloquine

Examples of Phenothiazines (antipsychotics, antiemeitics)

chlorpromazine, perphenazine, prochlorperazine


Kaugnay na mga set ng pag-aaral

Lab Simulation 9-1: Work with a Failover Cluster

View Set

MicroBio Ch. 5 - Viruses & Prions

View Set

SAFE Act and CSBS/AARMR Model State Law

View Set

Trench Warfare and Weapons of WWI

View Set